Download as pdf or txt
Download as pdf or txt
You are on page 1of 275

AUSTRALIAN MATHEMATICS

COMPETITION 1978-1984
M

BOOK 1 WJ ATKINS, JD EDWARDS, DJ KING,


PJ O'HALLORAN & PJ TAYLOR A
T
AUSTRALIAN MATHEMATICS
COMPETITION 1978-1984

BOOK 1 WJ ATKINS, JD EDWARDS, DJ KING,


PJ O'HALLORAN & PJ TAYLOR
Published by

AMT P u b l i s h i n g

Australian Mathematics Trust


170 Haydon Drive
Bruce ACT 2617
AUSTRALIA

Copyright ©1986 AMT Publishing


First reprint 2000
Second reprint 2004
Third reprint 2014
PDF edition 2018

Telephone: +61 2 6201 5137


www.amt.edu.au
AMTT Limited ACN 083 950 341
National Library of Australia Card Number and ISSN
Australian Mathematics Trust Enrichment Series ISSN 1326-0170
Australian Mathematics Competition Book 1 1978-1984
ISBN 978-1-876420-06-2
AUSTRALIAN MATHEMATICS TRUST

E N R I C H M E N T S E R I E S

EDITORIAL COMMITTEE

MIKE CLAPPER, Australian Mathematics Trust AUSTRALIA


WARREN J ATKINS, Newcastle AUSTRALIA
ED J BARBEAU, Toronto CANADA
GEORGE BERZSENYI, Shorewood USA
RON DUNKLEY, Waterloo CANADA
NIKOLAY KONSTANTINOV, Moscow RUSSIA
ANDY LIU, Edmonton CANADA
JORDAN B TABOV, Sofia BULGARIA
PETER J TAYLOR, Canberra AUSTRALIA
JOHN WEBB, Cape Town SOUTH AFRICA
The books in this series are selected for their motivating, interesting
and stimulating sets of quality problems, with a lucid expository
style in their solutions. Typically, the problems have occurred in either
national or international contests at the secondary school level.
They are intended to be sufficiently detailed at an elementary level
for the mathematically inclined or interested to understand but, at
the same time, be interesting and sometimes challenging to the
undergraduate and the more advanced mathematician. It is believed
that these mathematics competition problems are a positive influence
on the learning and enrichment of mathematics.
AUSTRALIAN MATHEMATICS TRUST

E N R I C H M E N T S E R I E S

B OO K S IN THE SERIES

1 AUSTRALIAN MATHEMATICS COMPETITION 1978–1984 BOOK 1


WJ Atkins, JD Edwards, DJ King, PJ O’Halloran & PJ Taylor
2 MATHEMATICAL TOOLCHEST
AW Plank & NH Williams
3 TOURNAMENT OF TOWNS QUESTIONS AND SOLUTIONS 1984–1989 BOOK 2
PJ Taylor
4 AUSTRALIAN MATHEMATICS COMPETITION 1985–1991 BOOK 2
PJ O’Halloran, G Pollard & PJ Taylor
5 PROBLEM SOLVING VIA THE AMC
W Atkins
6 TOURNAMENT OF TOWNS QUESTIONS AND SOLUTIONS 1980–1984 BOOK 1
PJ Taylor
7 TOURNAMENT OF TOWNS QUESTIONS AND SOLUTIONS 1989–1993 BOOK 3
PJ Taylor
8 ASIAN PACIFIC MATHEMATICS OLYMPIAD 1989-2000
H Lausch & C Bosch Giral
9 METHODS OF PROBLEM SOLVING BOOK 1
JB s & PJ Taylor
10 CHALLENGE! 1991-1995
JB Henry, J Dowsey, AR Edwards, LJ Mottershead,
A Nakos & G Vardaro
11 USSR MATHEMATICAL OLYMPIADS 1989-1992
AM Slinko
12 AUSTRALIAN MATHEMATICAL OLYMPIADS 1979-1995
H Lausch & PJ Taylor
13 CHINESE MATHEMATICS COMPETITIONS AND OLYMPIADS 1981-1993
A Liu
14 POLISH & AUSTRIAN MATHEMATICAL OLYMPIADS 1981– 1995
ME Kuczma & E Windischbacher
15 INTERNATIONAL MATHEMATICS TOURNAMENT OF TOWNS 1993–1997 BOOK 4
PJ Taylor & AM Storozhev
16 AUSTRALIAN MATHEMATICS COMPETITION 1992–1998 BOOK 3
WJ Atkins, JE Munro & PJ Taylor
17 SEEKING SOLUTIONS
JC Burns
18 101 PROBLEMS IN ALGEBRA
T Andreescu & Z Feng
19 METHODS OF PROBLEM SOLVING BOOK 2
JB Tabov & PJ Taylor
20 HUNGARY-ISRAEL MATHEMATICS COMPETITION: THE FIRST TWELVE YEARS
S Gueron
21 BULGARIAN MATHEMATICS COMPETITION 1992-2001
BJ Lazarov, JB Tabov, PJ Taylor & A Storozhev
22 CHINESE MATHEMATICS COMPETITIONS AND OLYMPIADS 1993-2001 BOOK 2
A Liu
23 INTERNATIONAL MATHEMATICS TOURNAMENT OF TOWNS 1997-2002 BOOK 5
AM Storozhev
24 AUSTRALIAN MATHEMATICS COMPETITION BOOK 1999-2005 BOOK 4
WJ Atkins & PJ Taylor
25 CHALLENGE! 1999-2006 BOOK 2
JB Henry, J Dowsey, AR Edwards, LJ Mottershead, A Nakos, G Vardaro
& PJ Taylor
26 INTERNATIONAL MATHEMATICS TOURNAMENT OF TOWNS 2002-2007 BOOK 6
PJ Taylor
27 INTERNATIONAL MATHEMATICAL TALENT SEARCH PART 1
G Berzsenyi
28 INTERNATIONAL MATHEMATICAL TALENT SEARCH PART 2
G Berzsenyi
29 AUSTRALIAN MATHEMATICAL OLYMPIADS 1996-2011 BOOK 2
H Lausch, A Di Pasquale, DC Hunt & PJ Taylor
30 METHODS OF PROBLEM SOLVING BOOK 3
JB Tabov, EM Kolev & PJ Taylor
31 AUSTRALIAN MATHEMATICS COMPETITION 2006-2012 BOOK 5
WJ Atkins & PJ Taylor
32 AUSTRALIAN INTERMEDIATE MATHEMATICS OLYMPIADS 1999-2013
JB Henry & KL McAvaney
33 PROBLEM SOLVING TACTICS
A Di Pasquale, N Do & D Mathews
34 CHINESE MATHEMATICS COMPETITIONS AND OLYMPIADS 2001-2009 BOOK 3
Y Fu, Z Li & A Liu
PREFACE
Preface to the First Printing
Since its beginning in 1978 the Australian Mathematics Competition has become
one of the great mathematics competitions in the world. In 1985 there were 305,000
entrants from 80% of Australian secondary schools with a further 32,000 entrants from
12 other countries, mainly in the South Pacific region. Large print and Braille versions
are produced for visually handicapped students and a French version for French
speaking students in the region.
The Competition consists of 3 sets of papers, a Junior Division for years 7 and 8,
Intermediate Division for years 9 and 10 and Senior Division for Years 11 and 12. Each
paper contains 30 multiple-choice questions which have to be attempted within 75
minutes.
There are many reasons why the Competition has had such a dramatic impact within
Australia. One is the unique triumvirate of sponsors from the educational sector
(Canberra CAE), the private sector (Westpac Banking Corporation) and the professional
sector (Canberra Mathematical Association).
Another significant factor has been the quality of the questions. As Professor Paul
Halmos, eminent mathematician and editor of the American Mathematical Monthly
wrote “I don’t know who designed them (the questions) but please transmit my
congratulations: they are intelligent questions, well put, and have the property of being
challenging and interesting, which is very unusual in competitions of this sort. I insist
that mathematics consists of ideas, not techniques, but that point is hard to make at
the school level…the questions come as close to doing so as can be done.”
Questions within each paper range from those covering important basic mathematical
skills to those which are of a deeper, more challenging nature, requiring insight,
creativity and a healthy measure of mental agility. Such questions serve their purpose
if students and teachers puzzle for a day or two over the solutions.
As a result of requests and encouragement from many teachers, students and their
parents, this book has brought together the hundreds of questions that have been set
in the AMC papers from 1978 to 1984. The questions have been grouped together
within topics and in general are sequenced in order of difficulty. The editors believe
that the book will become an extremely useful aid for mathematics teachers and a
learning vehicle for students.
The book includes the solutions of the questions. In many cases alternative solutions
are given, and no doubt many of these will be different from the reader’s solutions.
The solutions are not only intended as a way of showing how to solve the problems
but also to draw attention to the various approaches that can be adopted. The correct
response rates for all the year levels at which each question has been set, have also
been included.
Many people have contributed to the production of this book. Clearly, it would not have
been possible without the efforts of the members of the AMC Problems Committee. We
estimate that on average each question on a competition paper has required eight hours
of mathematicians’ time. This time is spent posing the problem, removing ambiguities,
searching for plausable distractors and ascertaining its suitability and placement in the
paper. This has happened under the guidance of two chairmen, Warren Atkins and
Peter Taylor, and involved, in addition to the editors, the time and talents of Geoff Ball,
George Berzsenyi, John Blake, Malcolm Brooks, Bob Bryce, John Carty, Bruce Devlin,
Kevin Friel, Chris Harman, Pierre-Olivier Legrand, John Mack, Bob McCreddin, John
Munro, Ron Scoins, Anne Street, Robin Thornely, Dennis Thorpe, Martin Ward and Jim
Williams. Helping them have been moderators in all parts of Australia and the Pacific
region who have taken on the task of fine-tuning the questions.
A further group of 1,325,216 has given purpose to the whole enterprise. This is the
number of entrants throughout Australia and the Pacific region who have sat for the
AMC papers represented here. We thank them and their teachers for the interest they
have shown.
Others have helped with the nuts and bolts of bringing this project to fruition. Bronwyn
Edwards, Susan Alosi, Lisa Pederson, Leanne Pilley, Clarice McLean, Charlie Samuels,
Susan Stewart, John Salmond and Adrian Young have given invaluable assistance in
the preparation of the manuscript. The Competition’s supremely capable and efficient
Manager, Sally Bakker, has provided exactly the right combination of support and
prodding to ensure that the publishing of the book has proceeded smoothly. Anne’s
help in keeping Alanna entertained has been above and beyond the call of duty.
Declan King, Peter O’Halloran
Canberra, November 1985
Preface to the Second Printing
As the two Chairmen of the Australian Problems Committee during the period covered
by this book we have re-typeset this book in LaTex and put it together as a publication
in the Australian Mathematics Trust’s Enrichment Series.
Since the book’s original printing as All the Best from the Australian Mathematics
Competition, the competition and related activities have seen many changes, mostly for
the good, but also regrettably the death of our colleague Peter O’Halloran in 1994. A
tribute to Peter appears in the latest printing of the second book in the series.
Current information about the competition may be found on the Australian
Mathematics Trust’s website at www.amt.edu.au
Warren Atkins, Peter Taylor
December 1999
Preface to the Third Printing
This is similar to the second printing, but makes corrections to some minor errors in
the second printing.

Preface to the Fourth Printing


I would like to thank Stephen Dutch for pointing out a number of errors which have
been corrected in this printing.
Peter Taylor
August 2014
ACKNOWLEDGEMENTS
We acknowledge those mathematicians and teachers who were on the Australian
Mathematics Competition Problems Committees during the years from 1978 to 1984.
They are the people who have produced the motivating and challenging questions in
this book and so have enriched the learning of mathematics.

Members of the AMC Problems Committee (1978 to 1984)

Mr W J Atkins Canberra CAE, ACT (1978-1979, 1981-1984)


Mr G Ball University of Sydney, NSW (1981-1982)
Prof J Blake University of Wollongong, NSW (1980-1984)
Dr M S Brooks Canberra CAE, ACT (1978-1979)
Dr R Bryce Australian National University, ACT (1980-1984)
Mr J Carty Merici College, ACT (1980-1984)
Mr B Devlin Balwyn High School, Vic (1984)
Br K Friel Kogarah Marist High School, NSW (1982)
Dr C J Harman Mitchell CAE, Bathurst, NSW (1980-1981, 1983-1984)
Mr D J King Dickson College, ACT (1978-1984)
M P-O Legrand Professeur Certifié, French Polynesia (1983-1984)
Mr R McCreddin Education Department of WA (1983)
Dr J M Mack University of Sydney, NSW (1978-1980)
Mr J E Munro Canberra CAE, ACT (1978-1984)
Prof R Scoins University of Waterloo, Canada (1984)
Dr A Street University of Queensland (1983-1984)
Dr P J Taylor Canberra CAE, ACT (1978, 1980-1984)
Ms R Thornely ACT Schools Authority (1982)
Mr D Thorpe ACT Schools Authority (1980-1984)
Dr M Ward Australian National University, ACT (1979)
Mr J L Williams University of Sydney, NSW (1980-1984)
DEDICATION

This book is dedicated to

Josephine Dianne Edwards


who died on May 25, 1985.

Jo played an integral part in the growth and development


of the Australian Mathematics Competition and also instigated
work on this volume. From the inception of the AMC in 1978,
Jo was a member of the Board of Governors and the
Management Committee, and chaired the Editorial Committee.
CONTENTS Questions
page
Solutions
page

Arithmetic 1 99
Approximation 1 99
Decimals 1 99
Directed Numbers 4 102
Exponents 5 102
Fractions 6 104
Mean 10 110
Metric System 11 111
Money 11 112
Order of Operations 12 112
Percentages 12 113
Properties of Numbers 14 115
Properties of Prime Numbers 18 123
Proportion 20 126
Rate, Time and Distance 20 126
Square Root 24 132

Algebra 25 133
Absolute Value 25 133
Collection of Like Terms 26 137
Equations — Diophantine 27 138
Equations — Linear 28 143
Equations — Quadratic 33 149
Equations — Rearrangement of 34 151
Equations — Roots of 35 152
Equations — Simultaneous 36 153
Equations — Surdic 37 154
Equations — Trigonometric 37 156
CONTENTS Questions
page
Solutions
page
Exponents 38 157
Expressions 39 159
Extreme Value 40 160
Factorisation 41 161
Fractions 42 162
Functions 42 163
Graphs 43 163
Inequalities 44 164
Logarithms 46 170
Operations 47 171
Percentages 48 172
Polynomials 49 173
Progressions 50 175
Ratio 51 177
Recursion Relations 51 178
Substitution 52 180
Surds 53 180

Geometry 54 182
Angles 54 182
Area 58 186
Circles 64 195
Coordinate Geometry 70 204
Cubes 72 208
Polygons 75 214
Pythagoras’ Theorem 76 216
Quadrilaterals 77 218
Ratio 78 219
Solid Geometry 79 222
CONTENTS Questions
page
Solutions
page
Triangles 82 227
Trigonometry 85 230
Volume 85 231

Other Topics 87 233


Counting Techniques 87 233
Logic 90 245
Probability 91 246
Sets 92 247
Trigonometric Identities 92 248
Miscellaneous 93 249
QUESTIONS
QUESTIONS
ARITHMETIC

APPROXIMATION
1 1978 J.10 (69%), I.3 (87%)
Which of the following is the closest approximation to 1.96 × 3.142?
(A) 60 (B) 6 (C) 0.6 (D) 0.06 (E) 0.006

2 1979 J.7 (53%), I.5 (79%), S.3 (86%)


2.7 × 32
Which of the following numbers is closest in value to ?
14.7
(A) 60 (B) 6 (C) 90 (D) 3 (E) 0.6

3 1980 J.6 (65%), I.3 (74%), S.2 (88%)


Which of the following is closest in value to 49.5 ÷ 0.5?
(A) 10 (B) 25 (C) 50 (D) 100 (E) 250

4 1983 I.8 (25%)


Of the following, which is the best estimate for the positive square root
1983
of ?
10 000
(A) 0.0045 (B) 0.0141 (C) 0.0445 (D) 0.1408 (E) 0.4453

5 1984 J.7 (41%)


Which of the following is closest in value to 601 ÷ 0.305?
(A) 2 (B) 20 (C) 200 (D) 2 000 (E) 20 000

DECIMALS
1 1978 J.1 (93%)
51.7 − 42.8 equals
(A) 94.5 (B) 9.1 (C) 11.1 (D) 11.9 (E) 8.9
2 Arithmetic

2 1978 J.2 (68%)


0.40 × 6.38 equals
(A) 0.2552 (B) 2.452 (C) 2.552 (D) 24.52 (E) 25.52

3 1979 J.1 (96%), I.1 (97%)


36.3 − 17.5 equals
(A) 18.8 (B) 19.2 (C) 19.8 (D) 21.2 (E) 18.2

4 1979 J.5 (50%), I.3 (73%), S.1 (90%)


2 2
(0.4) − (0.1) equals
(A) 0.09 (B) 1.5 (C) 0.15 (D) 0.6 (E) 0.06

5 1979 J.12 (60%)


Grumpy is as much taller than Dopey as Dopey is taller than Happy.
Grumpy is 1.27 m tall and Dopey is 1.11 m tall. Happy’s height, in
metres, is
(A) 1.05 (B) 0.95 (C) 0.16 (D) 1.43 (E) 1.19

6 1979 S.4 (77%)


0.1 1 0.3
If p = ,q= and r = , then which one of the following is true?
0.3 0.3 1
(A) p > q and q > r (B) q > r and r > p (C) q > p and p > r
(D) r > p and p > q (E) p > r and r > q

7 1980 J.1 (95%)


27.3 − 16.4 equals
(A) 1.9 (B) 9.9 (C) 11.1 (D) 10.9 (E) 11.9

8 1980 J.5 (63%)


Which of the following is a correct way of writing seventeen hundredths?
(A) 1700 (B) 0.17 (C) 0.0017 (D) 17.00 (E) 0.017
Decimals 3

9 1981 J.2 (84%), I.1 (90%)


1.1 − 0.64 equals
(A) 1.74 (B) 1.54 (C) 1.46 (D) 0.46 (E) 0.56

10 1981 J.12 (39%)


(0.3)2 × 0.7 equals
(A) 0.063 (B) 0.0063 (C) 0.63 (D) 0.042 (E) 0.42

11 1982 J.1 (99%), I.1 (99%)


2.5 + 2.1 equals
(A) 4.6 (B) 5.6 (C) 3.6 (D) 4.7 (E) 4.5

12 1982 J.6 (72%), I.3 (84%)

The diagram shows part of a scale of a mea- 25 26

suring device. The arrow indicates a reading


of 

(A) 25.03 (B) 25.15 (C) 25.3


(D) 25.6 (E) 25.25

13 1982 I.10 (34%), S.3 (51%)


3 2
(0.3) − (0.2) equals
(A) 0.5 (B) 0.23 (C) −0.13 (D) −0.013 (E) 0.05

14 1983 J.1 (95%)


1.2 + 3.4 equals
(A) 2.3 (B) 4.5 (C) 3.6 (D) 3.5 (E) 4.6

15 1984 J.1 (93%)


2.3 + 4.8 equals
(A) 6.1 (B) 6.11 (C) 7.1 (D) 8.1 (E) 7.11
4 Arithmetic

16 1984 J.3 (45%)


The sum of the smallest and the largest of the numbers 0.5129, 0.9, 0.89
and 0.289 is
(A) 1.189 (B) 0.8019 (C) 1.428 (D) 1.179 (E) 1.4129

17 1984 J.4 (47%), S.1 (94%)


(0.2)2 equals
(A) 0.04 (B) 0.4 (C) 2.0 (D) 0.02 (E) 0.004

18 1984 I.1 (94%)


7.3 − 4.9 equals
(A) 3.4 (B) 4.4 (C) 2.4 (D) 3.6 (E) 2.6

19 1984 J.10 (49%), I.3 (73%), S.2 (91%)


When doing a series of additions on a calculator, a student noted that
she added 35 095 instead of 35.95. In order to obtain the correct total in
a single step she should now
(A) add 35.95 (B) subtract 35 059.05 (C) subtract 35 130.95
(D) add 35 130.95 (E) subtract 35 095

DIRECTED NUMBERS
1 1978 J.6 (49%)
(−10) − (−14) equals
(A) −24 (B) −4 (C) 4 (D) 24 (E) −140

2 1981 J.7 (52%)


−6 + 4 − (−3) equals
(A) 1 (B) −7 (C) −5 (D) −13 (E) 7
Exponents 5

EXPONENTS
1 1979 S.29 (4%)
If      
P = (1 + 4) 1 + 42 1 + 44 1 + 48 1 + 416 1 + 432
then P equals

2128 + 264 − 5 2127 + 263 + 5 2128 − 1


(A) (B) (C)
3 3 3
2126 − 1
(D) (E) none of these
3
2 1981 S.9 (72%)
8
How many digits are there in 5 ?
(A) 4 (B) 5 (C) 6 (D) 7 (E) 8

3 1982 I.6 (28%), S.1 (74%)


3 2
The value of 9 × 3 is
(A) 275 (B) 276 (C) 37 (D) 38 (E) 312

4 1983 J.22 (14%)


1983
If 7 is divided by 100, then the remainder is
(A) 1 (B) 7 (C) 43 (D) 49 (E) 57

5 1983 S.7 (64%)


15 14
1.236 × 10 − 5.23 × 10 equals
(A) 7.13 × 1014 (B) 7.13 (C) 71.3 (D) −3.994 (E) 7.13 × 1013

6 1983 S.13 (12%)


If the numbers x = 2100 , y = 375 and z = 550 are ordered from the
smallest to the largest, then they are written in the order
(A) x, y, z (B) x, z, y (C) y, x, z (D) y, z, x (E) z, y, x
6 Arithmetic

7 1984 I.16 (9%)


25
If the result of 100 − 25 is written in decimal notation, the sum of the
digits in that number is
(A) 219 (B) 444 (C) 432 (D) 453 (E) 462

FRACTIONS
1 1978 J.5 (85%)
4 35 + 2 12 equals
4 4 1 3
(A) 6 (B) 6 (C) 7 (D) 6 (E) none of these
5 7 10 10
2 1978 S.4 (62%)
A cake is divided by removing at each cut one-quarter of the cake
present before the cut. After three cuts have been made, the fraction
left of the original cake is
1 27 37 1 1
(A) (B) (C) (D) (E)
4 64 64 64 2
3 1979 J.3 (86%), I.2 (88%)
2 23 − 1 12 equals
1 1 1 5 3
(A) 1 (B) 1 (C) 2 (D) (E)
3 6 6 6 5
4 1979 J.17 (68%), I.7 (83%)
1 10 3
If p = 3, q= 3 and r = 10 then which one of the following is true?
(A) p > q and q > r (B) q > r and r > p (C) q > p and p > r
(D) r > p and p > q (E) p > r and r > q

5 1979 J.21 (51%)


1 9 1 16 17
If P = Q= 3, R= 7, 5, S= 31 and T = 10 then the largest and
smallest fractions are
(A) S and R (B) S and P (C) T and S (D) T and P (E) T and R
Fractions 7

6 1979 S.11 (44%)


Of the following, which fraction is an integer multiple of each of the
fractions 67 , 14
5
and 10
21 ?
7 7 15 30 80
(A) (B) (C) (D) (E)
30 15 7 7 21
7 1980 J.2 (80%), I.1 (87%), S.1 (93%)
1 23 + 5
6 equals
(A) 2 12 (B) 2 13 (C) 1 79 (D) 2 23 (E) 1 12

8 1981 J.1 (87%)


2
3 of 12 is
(A) 6 (B) 8 (C) 9 (D) 4 (E) 18

9 1981 J.3 (87%)


1 3
3 + 8 equals
4 3 1 17 11
(A) (B) (C) (D) (E)
11 11 8 24 24
10 1981 J.11 (45%), S.1 (93%)
3 7
+
8 8 equals
4
5
21 25 5 25
(A) 1 (B) (C) (D) (E)
16 32 16 16
11 1981 J.14 (71%), I.7 (83%)
A block of chocolate is divided among three children, such that the first
is given 25 of the block, and the second is given 13 of the block. The
amount remaining for the third child is
11 3 4 5
(A) (B) (C) (D) (E) None
15 8 15 8
8 Arithmetic

12 1981 S.22 (45%)


37
The fraction can be written in the form
13
1
2+
1
x+
1
y+
z
where (x, y, z) equals
(A) (11, 2, 5) (B) (1, 5, 2) (C) (5, 2, 11)
(D) (1, 2, 5) (E) (13, 11, 2)

13 1982 J.2 (89%)


2 1
3 + 4 equals
1 3 11 21 8
(A) (B) (C) (D) (E)
6 7 12 34 3
14 1982 J.7 (41%), I.7 (62%)
1 1
The number halfway between 3 and 5 is
1 8 2 4 1
(A) (B) (C) (D) (E)
4 15 15 15 2
15 1982 I.4 (90%)
 
3 2 1
+ × equals
4 3 4
1 11 17 5 5
(A) (B) (C) (D) (E)
8 12 48 16 6
16 1982 S.15 (47%)
The recurring decimal 1.451 (i.e. 1.451515151. . .) is equal to
459 463 469 479 487
(A) (B) (C) (D) (E)
290 310 320 330 340
17 1983 J.3 (90%)
3 13 − 1 14 equals
11 1 11 7 7
(A) 1 (B) 2 (C) 2 (D) (E) 2
12 12 12 12 12
Fractions 9

18 1983 I.3 (53%)


3 7
+
7 3 equals
29
42
29 21 42
(A) 2 (B) 4 (C) (D) (E)
21 29 29
19 1983 I.6 (23%), S.2 (62%)
1
x− y
1 equals
y− x
x y −x
(A) (B) (C) 1 (D) −1 (E)
y x y
20 1984 J.2 (75%)
1
2÷ 3 equals
4 2 3
(A) (B) (C) (D) 6 (E) 3
3 3 3
21 1984 J.12 (26%)
A fraction which is less than 1 has both a positive numerator and
denominator. If 3 is added to both the numerator and denominator, the
new value of the fraction is
(A) increased by 1 (B) increased by 3 (C) decreased
(D) closer to 1 (E) unchanged

22 1984 J.21 (20%)


The value of
1 97 × 95
+ − 97
96 96
is
1 1
(A) (B) 0 (C) − (D) 1 (E) −1
96 96
10 Arithmetic

23 1984 I.5 (55%)


  1
  
The product 1 − 1 − 13 1 − 14 1 − 15 is
2
119 5 43 1 1
(A) (B) (C) 2 (D) (E)
120 7 60 5 120

MEAN
1 1981 J.19 (33%), I.15 (48%)
A meter records voltages between 0 volts and 20 volts. If the average
value for three readings on the meter was 16 volts, then the smallest
possible reading, in volts, was
(A) 8 (B) 9 (C) 6 (D) 11 (E) 10

2 1982 J.14 (22%)


The average weight of 5 boys is 70 kg and the average weight of 4 girls is
61 kg. The average weight of the 9 children, in kilograms, is
(A) 65.0 (B) 66.0 (C) 67.0 (D) 65.5 (E) 66.5

3 1983 J.4 (55%), I.4(68%)


The sum of three consecutive odd numbers is 27. The smallest of the
three is
(A) 11 (B) 9 (C) 8 (D) 7 (E) 5

4 1983 J.8 (36%), I.5 (54%)


The average of six numbers is 4. A seventh number is added and the
new average is 5. The seventh number is
(A) 6 (B) 5 (C) 10 (D) 11 (E) 12

5 1983 I.24 (23%)


If 105 is expressed as the sum of n consecutive positive integers, then of
the following numbers the value of n cannot be
(A) 3 (B) 4 (C) 5 (D) 6 (E) 7
Metric System 11

6 1984 J.23 (16%), I.19 (24%)


In his latest game of tenpin bowling Ken scored 199 and this raised his
average over a number of games from 177 to 178. To raise his average to
179 with the next game he must score
(A) 179 (B) 180 (C) 199 (D) 200 (E) 201

METRIC SYSTEM
1 1978 J.12 (20%)
The distance between two towns A and B is 150 kilometres. This
distance is represented on a map by a length of 300 millimetres. The
scale of this map is
(A) 1:500 000 (B) 30:50 (C) 1:20 000 (D) 1:5000 (E) 1:200 000

2 1982 J.10 (84%)


To anaesthetise a child, a doctor uses 4 mg of Pentothal for each
kilogram of body weight. To anaesthetize a child weighing 35 kg, the
number of milligrams of Pentothal a doctor would use would be
(A) 140 (B) 1400 (C) 14 (D) 8.75 (E) 39

MONEY
1 1983 J.20 (55%)
A party of 18 people went to a restaurant. They each chose a $21 meal,
but four of them forgot to bring their money. In order to settle the total
bill, those who brought their money each had to pay an extra
(A) $5.25 (B) $4.50 (C) $1.50 (D) $6.00 (E) $3.00

2 1984 J.6 (80%)


If you buy seven pens at $1.32 each the change from a $20 note should
be
(A) $9.24 (B) 76 cents (C) $10.76 (D) $18.68 (E) $10.86
12 Arithmetic

3 1984 I.12 (62%)


My father gives me half as much pocket money as he gives my older
sister, but one and a half times as much as he gives my younger brother.
He pays out $6.60 in pocket money to the three of us each week. How
much do I get each week?
(A) $3.05 (B) $3.60 (C) $2.00 (D) $1.80 (E) $2.20

ORDER OF OPERATIONS
1 1980 J.3 (59%)
2 + 3(8 − 4) equals
(A) 20 (B) 22 (C) 36 (D) 9 (E) 14

2 1983 J.2 (98%)


2 × (8 − 3) equals
(A) 13 (B) 2 (C) 3 (D) 5 (E) 10

PERCENTAGES
1 1978 J.3 (70%)
5% of 1200 equals
(A) 60 (B) 600 (C) 6 (D) 240 (E) 24

2 1978 I.24 (6%), S.16 (12%)


The cost of living in one quarter increased by 2%. To what annual
percentage inflation rate does this correspond? (Nominate the nearest
correct answer.)
(A) 2.0 (B) 8.0 (C) 8.1 (D) 8.2 (E) 8.3

3 1979 J.2 (79%)


3
expressed as a percentage is
8
(A) 60% (B) 62.5% (C) 42.5% (D) 40% (E) 37.5%
Percentages 13

4 1980 J.7 (76%)


24% expressed as a fraction is
12 1 4 6 5
(A) (B) (C) (D) (E)
5 4 25 25 24
5 1980 J.8 (29%)
What is the percentage of whole numbers from 2 to 21 inclusive which
are exact multiples of 4?
(A) 25 (B) 21 (C) 20 (D) 26 (E) 24

6 1981 J.5 (85%)


70% of $2.00 is
(A) 35c (B) $1.35 (C) 70c (D) $1.40 (E) $140

7 1981 I.9 (22%)


A car salesman eager to sell his old stock decides to decrease the price of
each car by 10%. He then realises that he will make a loss at these new
prices and increases them by 5%. His nett discount is
(A) 4.5% (B) 5.5% (C) 5.0% (D) 6.0% (E) 4.0%

8 1981 S.7 (63%)


A man invests $1000 for 3 years at 10% per annum simple interest. How
much greater would his return have been if the money had been invested
at 10% per annum compound interest, compounded annually, for the
same period?
(A) None (B) $10 (C) $20 (D) $21 (E) $31

9 1982 J.4 (89%)


25% of 24 equals
(A) 3 (B) 4 (C) 6 (D) 8 (E) 12
14 Arithmetic

10 1982 J.9 (22%)


A rare blood group is present in 0.15 per cent of the Australian
population of 14 million. How many people in Australia are in this
group?
(A) 2100 (B) 21 000 (C) 210 (D) 210 000 (E) 2 100 000

11 1983 J.6 (79%)


35% expressed as a fraction is
1 20 7 5 7
(A) (B) (C) (D) (E)
3 7 2 7 20
12 1984 J.17 (38%)
A town has 2500 residents of whom 60% voted in an election to fill a
council vacancy. The result was that of those who voted, 38% voted for
P , 32% for Q and 30% for R. Under the voting system, P was elected.
The number of residents who voted for P was
(A) 450 (B) 570 (C) 1250 (D) 950 (E) 1500

PROPERTIES OF NUMBERS
1 1978 J.16 (30%), I.10 (49%), S.8 (75%)
If x = (n + 1)(n + 2)(n + 3) where n is a positive integer then x is not
always divisible by
(A) 1 (B) 2 (C) 3 (D) 5 (E) 6

2 1978 J.23 (53%)


If the numbers 1, 2, 3, 4, 5, 6, 7, 8, 9, 10, 11, 12 are arranged in 3
columns of 4 numbers each so that the sum of the numbers in each
column is the same, then the sum of each column is
(A) 21 (B) 26 (C) 32 (D) 18 (E) none of these

3 1979 J.6 (58%)


Which one of the following must be odd when n is an integer (a whole
number)?
(A) 3n (B) 2n + 1 (C) n2 (D) n3 (E) n + 2
Properties of Numbers 15

4 1979 J.10 (65%)


Given a two-digit number, a new three-digit number is made from it by
putting the digit 1 after it. The new number is then
(A) the old number plus one (B) ten times the old number, plus one
(C) one hundred plus the old number
(D) one hundred times the old number, plus one (E) the old number

5 1979 J.25 (26%), I.16 (34%)


 3
What is the last digit of the number 75 ?
(A) 1 (B) 3 (C) 5 (D) 7 (E) 9

6 1979 J.30 (8%), I.30 (10%), S.30 (10%)


A shopkeeper receives the following account:

22 model X cassette players: $ 29.3

with the first and last digits smudged beyond recognition. He knows
that each cassette player costs over $25. The price, in dollars, of each
cassette player is between
(A) 25 and 28 (B) 28 and 32 (C) 32 and 35
(D) 35 and 40 (E) 40 and 50

7 1981 J.26 (18%), I.23 (11%), S.17 (9%)


1981
When 3 + 2 is divided by 11, the remainder is
(A) 5 (B) 0 (C) 7 (D) 6 (E) 3

8 1982 J.3 (89%)


What are the next two whole numbers greater than 1009?
(A) 1100, 1101 (B) 1010, 1011 (C) 1007, 1008
(D) 1010, 1020 (E) 1008, 1010
16 Arithmetic

9 1982 J.16 (37%)


A street of houses numbered from 1 to 100 inclusive is to be numbered
with new brass numerals. How many of the digit ‘2’ would be needed to
complete the job?
(A) 20 (B) 19 (C) 18 (D) 10 (E) 11

10 1983 J.25 (16%)


Upon substituting the four numbers, 1, 9, 8 and 3, for the four letters in
the addition problem (different numbers for different letters)
BAD + M AD + DAM , the largest sum obtainable is
(A) 1916 (B) 2045 (C) 2056 (D) 2065 (E) 2049

12 1982 J.25 (37%), I.24 (25%), S.19 (26%)


When the three-digit numbers 6a3 and 2b5 are added together, the
answer is a number divisible by 9. The largest possible value of a + b is
(A) 12 (B) 9 (C) 2 (D) 20 (E) none of these

13 1982 I.27 (11%), S.26 (13%)


Sheep cost $40 each, cows $65 each and hens $2 each. If a farmer bought
a total of 100 of these animals for a total cost of $3279, then he must
have bought
(A) 35 sheep and more than 42 hens
(B) 42 hens but an indefinite number of sheep and cows
(C) An even number of sheep (D) 23 sheep and an odd number of cows
(E) 31 cows and 42 hens

14 1982 S.2 (90%)


The digits of the number 1982 are arranged in descending order and then
in ascending order. The difference between the resulting two numbers is
(A) 8668 (B) 909 (C) 8642 (D) 8532 (E) 1982

15 1982 S.22 (5%)


If 100! is expressed as an integer, how many zeros would there be at the
end of the number?
(A) 11 (B) 20 (C) 21 (D) 22 (E) 24
Properties of Numbers 17

16 1982 S.24 (24%)


The notation a ≡ b (mod m), where a, b and m are integers, means that
a − b is divisible by m. For which one of the following statements does
there exist an x which makes it true?
(A) 2x ≡ 3 (mod 12) (B) 3x ≡ 7 (mod 12) (C) 6x ≡ 11 (mod 12)
(D) 5x ≡ 9 (mod 12) (E) 10x ≡ 5 (mod 12)

17 1983 J.14 (37%)


Ann, Wendy and Christopher each take two Vitamin C tablets each day,
while Bill takes a single tablet each day. There are enough tablets in a
full bottle to last exactly 24 days. How many days will the tablets in a
full bottle last if Bill is also to take two tablets daily?
(A) 21 (B) 22 (C) 18 (D) 20 (E) 16

18 1983 S.24 (34%)


We can devise a shorthand notation for large numbers by letting dn
stand for the occurrence of n consecutive ds where n is a positive integer
and d is a fixed digit (0 ≤ d ≤ 9). Thus, for example, 14 95 82 36 denotes
the number 11119999988333333. Find the ordered triple (x, y, z) if

2x 3y 5z + 3z 5x 2y = 53 72 83 51 73 .

(A) (4,5,3) (B) (3,6,3) (C) (3,5,4) (D) (5,3,4) (E) (5,4,3)

19 1983 S.25 (9%)


x denotes the largest integer less than or equal to x. For example,
3 = 3, 5.7 = 5. If
√3

3
√3 √
 1 +  2 +  3 + · · · +  3 n = 2n,

then the value of n is


(A) 29 (B) 33 (C) 41 (D) 49 (E) 53
18 Arithmetic

20 1984 J.24 (2%), S.10 (10%)


The value of 50! is the product of all the whole numbers from 1 to 50
inclusive, i.e. 50! = 1 × 2 × 3 × · · · × 49 × 50. The maximum number of
times that 2 will divide into 50! exactly is
(A) 25 (B) 50 (C) 47 (D) 42 (E) 46

21 1984 J.25 (8%)


Each of the faces of a cube is numbered with a different integer. Each
vertex is assigned a ‘vertex number’ which is the sum of the numbers on
the faces which intersect in that vertex and then the sum of the vertex
numbers is calculated. The highest number which must divide this sum,
for every possible numbering of the faces, is
(A) 3 (B) 4 (C) 6 (D) 8 (E) 12

22 1984 S.27 (6%)


Three-digit numbers are formed using only digits which are odd. The
sum of all such three-digit numbers is
(A) 69 375 (B) 19 375 (C) 6253 (D) 34 975 (E) 33 300

PROPERTIES OF PRIME NUMBERS


1 1978 J.30 (8%), I.29 (8%)
If p and q are different primes (so each ≥ 2) and if n = pq, then the
number of integers of the set 2, 3, 4, . . . , n which have no common factors
(other than 1) in common with n is
(A) n − pq (B) pq − (p + q) (C) pq − (p + q + 1)
(D) pq − 3 (E) pq − 4

2 1978 S.13 (45%)


Two adults have ages in years whose product is 770. The sum of their
ages is
(A) 25 (B) 57 (C) 69 (D) 87 (E) 117
Properties of Prime Numbers 19

3 1978 S.28 (8%)


The number of pairs (p, q) of distinct prime numbers (≥ 2) such that p
divides (q 2 − q) and q divides (p2 + p) is
(A) 0 (B) 1 (C) 2 (D) 3 (E) infinite

4 1981 I.5 (41%)


In how many ways can the number 24 be expressed as the sum of two
prime numbers? (Note that the number 1 is not a prime number.)
(A) None (B) 1 (C) 2 (D) 3 (E) 4

5 1982 J.28 (14%), I.17 (20%), S.14 (27%)


The product of my children’s ages in whole numbers of years is 1664.
The youngest is at least half the age of the eldest. I am fifty. How many
children do I have?
(A) 2 (B) 3 (C) 4 (D) 5 (E) 6

6 1983 J.13 (19%)


A child glues together 42 cubes with 1 cm edges to form a solid
rectangular-faced brick. If the perimeter of the base is 18 cm, the height
of the brick, in centimetres, is
7
(A) 3 (B) 6 (C) 2 (D) 7 (E)
3
7 1983 I.26 (11%)
During the 1914–1918 War, in the Po Valley in Italy, a skeleton, a
battered uniform and a halberd (a weapon no longer than 10 feet) were
found. Archaeologists found that they belonged to a French captain.
The length, in feet, of the halberd (a whole number) times the number
of days in the month in which the French captain was killed times half
the number of years between the death of the captain and the discovery
of his skeleton times half the age of the captain when he died makes
451 066.
The captain was killed in the Battle of
(A) Torino, Feb. 1522 (B) Cremona, March 1712 (C) Pavia, Feb. 1512
(D) Marengo, June 1800 (E) Castiglione, Aug. 1796
20 Arithmetic

8 1984 J.18 (9%), I.8 (18%)


The least positive integer by which 504 should be multiplied so that the
product is a perfect square is
(A) 2 (B) 6 (C) 7 (D) 14 (E) 56

9 1984 I.17 (13%)


The integer N is the square of a square and has 18 as a factor. The least
N
value of is
18
(A) 36 (B) 48 (C) 72 (D) 2 (E) 18

PROPORTION
1 1978 I.17 (60%)
On the Réamur temperature scale, water freezes at 0 and boils at 80◦ ,

instead of 0◦ and 100◦ , respectively, on the Celsius scale. If the


temperature is 28◦ Réamur, then the temperature in degrees Celsius is
(A) 28 (B) 35 (C) 48 (D) 8 (E) 42

2 1980 J.22 (12%)


A common mixture for making concrete is 1 part cement, 2 parts sand
and 4 parts gravel, by volume. In one cubic metre of the dry mixture
the volume of sand, in cubic centimetres, is closest to
(A) 286 (B) 2860 (C) 286 000 (D) 28 600 (E) 0.286

RATE, TIME AND DISTANCE


1 1978 S.25 (27%)
At 12 noon, the hour hand of a clock begins moving at three times its
normal rate, while the minute hand begins moving at half its normal
rate. The true time when the hands next meet is
(A) 12.30 pm (B) 2.00 pm (C) 3.30 pm (D) 4.00 pm (E) midnight
Rate, Time and Distance 21

2 1979 J.16 (54%)


A car averages 40 km/h for 20 km and 60 km/h for another 20 km. The
time taken in minutes for the 40 km is
(A) 24 (B) 48 (C) 50 (D) 150 (E) 300

3 1979 I.11 (49%)


Jack climbed at a uniform rate up the beanstalk. At 2 o’clock he was
one-sixth of the way up and at 4 o’clock he was three-quarters of the
way up. At 3 o’clock the fraction he had climbed was
11 7 11 1 2
(A) (B) (C) (D) (E)
12 12 24 8 5
4 1979 I.23 (6%)
A car averages 50 km/h for 20 km and 60 km/h for another 20 km. The
average speed in kilometres per hour for the 40 km is
6 5 6
(A) 55 (B) 54 (C) 54 11 (D) 55 11 (E) 55 11

5 1981 J.22 (6%)


A driver covers the first 10 km of a 20 km journey at an average speed of
25 km/h. What speed, in kilometres per hour, must be averaged over
the remaining 10 km to make the overall average 40 km/h?
(A) 100 (B) 200 (C) 25 (D) 40 (E) 55

6 1981 I.25 (11%), S.26 (9%)


A jogger ran for 1 12
hours. First she ran along a level track and then up
a hill to the top, where she turned around and ran back to her starting
point along the same route. If she runs at 8 km/h on the level track,
6 km/h uphill and 12 km/h downhill, then the total distance, in
kilometres, she ran was
(A) 8 (B) 14 34 (C) 16 12
(D) 12 (E) not determined by the given information
22 Arithmetic

7 1982 J.18 (18%), I.12 (40%), S.7 (64%)


A certain substance doubles its volume every minute. At 9.00 am a
small amount is placed in a container and at 10.00 am the container just
fills. The time at which the container was one-quarter full was
(A) 9.15 am (B) 9.30 am (C) 9.45 am (D) 9.50 am (E) 9.58 am

8 1982 J.23 (25%), S.12 (61%)


An aeroplane flies at 800 km/h for one-third of its flight time and
averages 700 km/h for the entire trip. What is the average speed, in
kilometres per hour, over the remaining part of the journey?
(A) 600 (B) 750 (C) 650 (D) 500 (E) 625

9 1983 J.10 (11%)


The five tyres of a car (four road tyres and a spare) were each used
equally on a car that travelled 20 000 km. The number of kilometres of
use of each tyre was
(A) 4 000 (B) 5 000 (C) 16 000 (D) 20 000 (E) 100 000

10 1983 J.12 (28%)


In addition to spanning a river which is 50 metres wide, a straight
bridge also has considerable overlap of both banks of the river.
One-third of the bridge overlaps one bank and one-half overlaps the
other bank. The total length, in metres, of the bridge is
(A) 100 (B) 150 (C) 200 (D) 300 (E) 400

11 1983 J.18 (25%)


Two missiles are initially 5000 km apart. They travel along a straight
line directly towards one another, one travelling at 2000 km/h and the
other at 1000 km/h. How many kilometres are they apart 1 minute
before impact?
(A) 3000 (B) 1000 (C) 500 (D) 100 (E) 50
Rate, Time and Distance 23

12 1983 J.27 (6%), I.17 (9%)


The wheels of a truck travelling at 60 km/h make 4 revolutions per
second. The diameter of each wheel, in metres, is
25 6π 25π 100 25
(A) (B) (C) (D) (E)
12π 25 6 6π 6π
13 1983 I.13 (23%)
A painter can paint a room in twelve hours. An apprentice, who can
paint the room in twenty-four hours, is added to the workforce. If they
work together at the rates indicated, the number of hours required to
paint the room will be
(A) 6 (B) 8 (C) 9 (D) 12 (E) 18

14 1983 I.20 (19%), S.15 (37%)


In a race of 2000 m, Raelene finishes 200 m ahead of Marjorie, and 290 m
ahead of Betty. If Marjorie and Betty continue to run at their previous
average speeds, by how many metres will Marjorie finish ahead of Betty?
(A) 90 (B) 100 (C) 120 (D) 180 (E) 200

15 1984 J.8 (53%)


The double feature at the cinema consisted of two films which lasted 1
hour 37 minutes and 1 hour 51 minutes respectively, with a 15 minute
interval. If the show began at 7.30 pm, at what time did it finish?
(A) 11.13 pm (B) 10.33 pm (C) 10.13 pm (D) 10.23 pm (E) 11.03 pm

16 1984 J.15 (76%), I.10 (88%), S.5 (96%)


Fuel consumption of cars is usually quoted as the number of litres of fuel
required to travel 100 km. My car travels 12.5 km on one litre of petrol.
How many litres of petrol does my car use in travelling 100 km?
(A) 8 (B) 7 (C) 5 (D) 12.5 (E) 10
24 Arithmetic

17 1984 J.26 (3%), I.23 (6%), S.14 (16%)


Kirsten runs twice as fast as she walks. When going to school one day
she walks for twice the time she runs and takes 20 minutes. The next
day she runs for twice the time she walks. How many minutes does it
take her to get to school on the second day?
(A) 16 (B) 15 (C) 13 13 (D) 18
(E) not determined by the given information

SQUARE ROOT
1 1978 J.8 (52%)
The square root of 15 × 20 × 12 equals
(A) 6000 (B) 60 (C) 6 (D) 80 (E) none of these
ALGEBRA

ABSOLUTE VALUE
1 1978 J.27 (8%), I.15 (15%)
If |x − 1| = 2x then x must equal
(A) −1 only (B) 1 only (C) 3 only
1 1
(D) −1 or (E) only
3 3
2 1979 I.12 (38%)
If |x| = |y| and x < 0 and y > 0 then which of the following statements
is false?
(A) x2 y > 0 (B) x + y = 0 (C) xy < 0
1 1 x
(D) − =0 (E) +1=0
x y y
3 1979 I.22 (48%)
If |x − 1| − |x − 2| = 0 then x equals
2 1 3
(A) 0 (B) (C) 1 (D) (E)
3 2 2
4 1980 I.22 (18%), S.18 (54%)
For what values of x is |x| + |x − 1| = 1?
(A) 0 and 1 only (B) 0 and −1 only (C) all x
(D) −1 < x ≤ 1 (E) 0 ≤ x ≤ 1

5 1981 S.19 (49%)


The statement |x + 1| + 2|x − 2| < 6 is equivalent to
(A) −1 < x < 2 (B) 0 < x < 1 (C) −1 < x < 3
(D) x < 2 (E) x < −1 or x > 2
26 Algebra

6 1983 S.22 (5%)


What is the area of the region in the Cartesian plane whose points (x, y)
satisfy
|x| + |y| + |x + y| ≤ 2?

(A) 2.5 (B) 3 (C) 2 (D) 4 (E) 3.5

COLLECTION OF LIKE TERMS


1 1979 I.4 (86%)
15x − 10y − 8x + 13y equals
(A) 5x + 5y (B) 7x + 3y (C) 7x − 3y
(D) 28x − 18y (E) 7x − 23y

2 1978 J.7 (66%)


8x + 3y + 4x − 5y equals
(A) 12x + 8y (B) 12x − 8y (C) 24xy
(D) 3x + 7y (E) 12x − 2y

3 1979 J.9 (54%)


If y = 3x and z = 4x then x + y + z equals
(A) 7x (B) 8x (C) 9x (D) 12x2 (E) 8x3

4 1982 I.5 (95%)


3(2x − 4y) + 5x equals
(A) 11x − 12y (B) 10x − 12y (C) 11x − 4y
(D) 10x − 4y (E) 10x − 7y

5 1981 I.4 (78%)


2x + 1 − 3(2 − x) simplifies to
(A) 5x − 5 (B) −x − 5 (C) x − 5 (D) 5 − x (E) 8x − 5
Equations: Diophantine 27

EQUATIONS: DIOPHANTINE
1 1984 J.22 (22%), I.20 (28%), S.12 (45%)
Kathryn has 20 coins in her purse. They are 10c, 20c and 50c coins and
the total value of the coins is $5. If she has more 50c than 10c coins,
how many 10c coins has she?
(A) 4 (B) 9 (C) 2 (D) 7 (E) 5

2 1984 I.29 (12%), S20 (19%)


Articles X, Y and Z are for sale. Article X can be bought at the rate of
eight for $1. Article Y costs $1 each and Article Z costs $10 each. You
buy a selection of all three types and find that you have purchased
exactly 100 articles at a cost of $100. The number of articles of type Y
was
(A) 14 (B) 18 (C) 20 (D) 21 (E) 24

3 1982 J.26 (31%), I.21 (54%)


If x and y are positive integers and x + y + xy = 34 then x + y is
(A) 10 (B) 12 (C) 20 (D) 34 (E) not determined

4 1979 I.27 (23%), S.21 (22%)


The least positive integer which has remainders 1, 1 and 5 when divided
by 3, 5 and 7 respectively, is
(A) 166 (B) 151 (C) 145 (D) 131 (E) none of these

5 1979 S.27 (9%)


2 2
If x and y are integers such that (x − y) + 2y = 27, then the only
numbers x can be are
(A) 3, 5 (B) −6, 4 (C) 0, 4, 6
(D) 0, −4, 4, −6, 6 (E) 0, −2, 2, −4, 4, −6, 6
28 Algebra

6 1983 I.29 (8%), S.27 (9%)


One of the solutions of the equation 19x + 83y = 1983 in positive
integers x and y is obviously given by (x, y) = (100, 1). It turns out that
there is exactly one more pair of positive integers (x, y), which satisfies
the above equation. The value of x + y for that pair is
(A) 27 (B) 37 (C) 47 (D) 57 (E) 67

7 1980 S.30 (3%)


The sum of five positive integers x, y, z, u, v is equal to their product. If
x ≤ y ≤ z ≤ u ≤ v, then the number of distinct solutions (x, y, z, u, v) is
(A) 0 (B) 2 (C) 3 (D) 4 (E) 5

8 1978 I.30 (8%)


The number of solutions in positive integers of the equation
3x + 5y = 1008 is
(A) 1 (B) 134 (C) 68 (D) 67 (E) infinite

EQUATIONS: LINEAR
1 1982 J.12 (21%), I.8 (70%)
The solution of the equation 3(x − 4) = 7x − 10 is
1 1 1 1 1
(A) (B) 5 (C) 2 (D) 1 (E) −
2 2 5 2 2
2 1978 J.13 (31%)
The solution of the equation 2x + 5 = 9 − 3x is
5 7 4
(A) 1 (B) 4 (C) (D) (E)
4 8 5
3 1979 J.15 (28%), I.6 (74%)
The solution of the equation 2x + 5 = 5x − 11 is
1 1 2 1
(A) 2 (B) 5 (C) 2 (D) −5 (E) −2
3 3 7 3
Equations: Linear 29

4 1980 I.11 (43%), S.7 (77%)


1 1 1
Given the equation = + where x = 2 and y = 3 then the value of
x y z
z is
1 5 6
(A) −6 (B) (C) (D) (E) 6
6 6 5
5 1983 I.14 (76%)
If a + b = 7, b + c = 9, a + c = 8, then abc is
(A) 60 (B) 63 (C) 64 (D) 27 (E) 120

6 1980 J.15 (44%), I.10 (58%)


Tom is 3 years older than Suzanne. The sum of their ages is 15. Given
that Tom’s age is x years, which of the following equations will find x?
(A) x = 15 − 3 (B) x + (x − 3) = 15 (C) x + 3x = 15
(D) x + (x + 3) = 15 (E) x = 15 + (x − 3)

7 1980 J.14 (83%)


Twins Toni and Toby are given the same amount of pocket money. Toni
buys 2 apples and has 70c left. Toby buys 4 apples and has 20c left.
What amount of money did each receive?
(A) 30c (B) $1.20 (C) 25c (D) $1.00 (E) $2.20

8 1978 J.11 (35%)


The angles of a triangle are in the ratio 2 : 3 : 4. The size of the largest
angle, in degrees, is
(A) 40 (B) 80 (C) 45 (D) 90 (E) 72

9 1982 J.20 (59%), I.15 (74%)


A bag contains 20 marbles coloured either red, white, blue or green.
There is one more red than white, 4 more white than blue and one more
blue than green. The number of red marbles is
(A) 8 (B) 2 (C) 7 (D) 3 (E) 10
30 Algebra

10 1983 I.10 (57%)


A precious stone weighing 25 carats was broken into two pieces. The
two pieces were put on a balance, one on either side. An additional
weight of 9 carats on one side was required to maintain balance. The
weight, in carats, of the larger piece was
(A) 14 (B) 17 (C) 19 (D) 21 12 (E) 8

11 1980 J.20 (20%), I.13 (28%)


At a party there is an equal number of boys and girls. One-quarter of
the boys have part-time jobs. Twice as many of the girls have part-time
jobs. Altogether 30 of those present are without part-time jobs. The
number of people at the party is in the range
(A) 100–124 (B) 60–72 (C) 34–38 (D) 39–43 (E) 44–48

12 1982 J.22 (46%)


Jane and her younger brother Ben take their pocket money, $2 and $1
respectively, to the shops to buy fruit. Together they spend $1.40. Jane
has three times as much money left as Ben. Jane spent
(A) 80c (B) 60c (C) 35c (D) $1.05 (E) 30c

13 1980 S.12 (24%)


A person’s age on his birthday in 1980 is equal to the sum of the digits
of the year 19xy in which he was born. Therefore x and y satisfy the
relation
(A) 70 − 2x − 2y = 0 (B) 69 − 11x − 2y = 0 (C) 70 − 11x − 2y = 0
(D) 69 − 2x − 2y = 0 (E) 70 − 11x + 2y = 0

14 1983 J.21 (25%)


Adam bought a number of ping-pong balls in a store in Texas, where a
5% sales tax is added to every purchase. If he did not have to pay tax,
he could have bought 3 more balls for the same amount of money. How
many balls did he buy?
(A) 30 (B) 45 (C) 57 (D) 60 (E) 63
Equations: Linear 31

15 1979 S.23 (10%)


Two candles of equal length start burning at the same time. One of the
candles will burn down in 4 hours, the other in 5 hours. How many
hours will they have to burn before one candle is 3 times the length of
the other?
40 45 63 47
(A) (B) 3 (C) (D) (E)
11 12 20 14
16 1979 I.17 (13%), S.10 (31%)
A rectangular pool is 50% longer than it is wide. It is surrounded by a
path 1 metre wide. If the area of the path is 44 square metres, then the
area of the pool, in square metres, is in the range
(A) less than 80 (B) 80 to 90 (C) 91 to 100
(D) 101 to 120 (E) 121 to 170

17 1983 I.15 (52%)


A furnace is at three-quarters of its normal operating temperature, in

C, when the power goes off. Two hours later the temperature of the
furnace has dropped by 100◦ C and the furnace is now at five-eighths the
normal operating temperature. What is the normal operating
temperature of the furnace?
(A) 100◦ C (B) 400◦ (C) 600◦ C (D) 800◦ C (E) 1000◦ C

18 1982 I.23 (21%)


.........................
............. .......
....... ........
In the diagram there is a linear relationship be- ........ .. ..........
..... .....
....... ... .
.... ....
. ... . ...
tween the two integers occupying opposite sectors. ...
. .... ... ...
... ... ... ...
.. .. ...
The value of the missing integer is ....
. 50 ...
... ....
... ...
..
2 ...
...
... ..... ...
... ..
. .
... ........
. ..
... .. .. ...
(A) 23 (B) 27 (C) 35 ...
... 11 .
..
..
.
. ...
.... 15 ...
.
...
... ... ...
..
... ... ...
...
(D) 41 (E) 61 ... ... ...
....
.....
..... .....
...... ..
6 ...
... .....
.......
.
..
.
....... .......
...........
..............................
32 Algebra

19 1979 J.24 (24%), I.14 (37%)


To make plum wine, sugar is added to plum juice until the volume
increases by 10%. The plum juice is in a cylindrical container with a
base of radius 12 cm and a height of 16.5 cm. What height, in
centimetres, of plum juice is needed so that when the sugar is added the
container is just filled?
(A) 12 (B) 13 (C) 14 (D) 15 (E) 16

20 1978 S.12 (43%)


After 200 throws, a coin has shown heads exactly 110 times. How many
consecutive heads must now be thrown for heads to have shown for
exactly 70% of the total number of throws?
(A) 30 (B) 300 (C) 220 (D) 100 (E) none of these

21 1980 S.13 (50%)


The arithmetic mean (average) of a set of 50 numbers is 38. Two
numbers of the set, namely 45 and 55, are discarded. The arithmetic
mean of the remaining set of numbers is
(A) 35.5 (B) 36 (C) 36.5 (D) 37 (E) 37.5

22 1983 J.29 (5%), I.25 (5%), S.19 (8%)


In his will a father left all his money to his children in the following
1
manner: $1000 to the first born and of what then remains, then
10
1
$2000 to the second born and of what then remains, then $3000 to
10
1
the third born and of what then remains, and so on.
10
When this was done each child had the same amount. How many
children were there?
(A) 6 (B) 7 (C) 8 (D) 9 (E) 10
Equations: Quadratic 33

EQUATIONS: QUADRATIC
1 1978 I.7 (40%)
2
The solutions of the roots of the equation x − 9x − 36 = 0 are
(A) −3, 12 (B) 3, −12 (C) −3, −12 (D) 0, −4 (E) −9, 4

2 1982 S.6 (42%)


The average of the roots of the equation 2x2 + 14x + 17 = 0 is

17 15 7 7
(A) (B) −7 (C) (D) (E) −
4 9 2 2
3 1983 S.6 (41%)
If (x − 3)(2x + 1) = 0 then the possible values of 2x + 1 are
1
(A) 0 only (B) 0 and 3 (C) − and 3
2
1 7
(D) 0 and 7 (E) − and −
2 2
4 1981 I.20 (8%)
1 1
If x + = 3, then x2 + 2 equals
x x
(A) 9 (B) 10 (C) 27 (D) 11 (E) 7

5 1983 I.21 (13%)


After mowing a 4 m wide strip around the sides of a square lawn,
three-quarters of the lawn has been mown. The number of square
metres remaining to be mown is
7
(A) 1 (B) 4 (C) 16 (D) 64 (E) 192
9
6 1979 S.14 (14%)
2 2
If 9 (log x) + 4 (log y) = 12 (log x) (log y), then
(A) x3 = y 2 (B) x2 = y 3 (C) x = y
(D) x + y = 1 (E) 3x = 2y
34 Algebra

EQUATIONS: REARRANGEMENT OF
1 1978 I.12 (35%)
a + 3b a
If = 3 then equals
a−b b
(A) 1 (B) 2 (C) 3 (D) 4 (E) 5

2 1983 I.16 (12%), S.9 (60%)


1 1 1
If = + then z equals
x y z
xy x−y xy y−x
(A) (B) (C) x − y (D) (E)
x−y xy y−x xy
3 1983 S.10 (13%)
If a + b = 1 and a2 + b2 = 2 then a4 + b4 equals
1
(A) 4 (B) 8 (C) 1 (D) 3 (E) 3
2
4 1978 I.14 (23%)
The equation relating the period T of a pendulum, its length , and the
gravitational acceleration g is


T = 2π .
g

From this equation, g equals



 2π 4π 2  4π 2  2π2
(A) 2π (B) (C) (D) (E)
T T T T2 T2
5 1980 I.17 (17%)
A circle has perimeter p cm. Its area, in square centimetres, is
p2 p2 2π
(A) (B) (C) πp2 (D) 2πp (E)
4π 4 p
Equations: Roots of 35

6 1979 I.20 (27%)


The equation giving the surface area A of a closed cylinder is
A = 2πr(r + h), where r is the radius of the base and h is the height.
From this equation h equals
A A − 2πr2 A A
(A) (B) (C) (D) A − 2πr2 (E)
2πr 2πr 2πr2 2πr(r + h)
7 1981 S.16 (58%)
  
1 1
If v = 2GM − then r equals
r R
2GM R 2GM R 2GM
(A) (B) (C)
v 2 R − 2GM v 2 R + 2GM v 2 R − 2GM
2GM 2GM R
(D) (E)
v 2 − 2GM R v 2 R + 2GM R
8 1980 I.21 (15%)
x+2
If y = , then x is
x−3
3y − 2 3y + 2 2 − 3y y+2 y−2
(A) (B) (C) (D) (E)
y+1 y−1 1+y y−3 y+3

EQUATIONS: ROOTS OF
1 1980 I.5 (55%)
3
If 8r = 1, what is the value of r?
1 1 1 1
(A) 2 (B) (C) (D) − (E)
24 8 2 2
2 1978 I.23 (5%)
√ 2
If 3 − 1 is a root of the equation ax − 5x + 1 = 0 then a equals
√ √ √
(A) 10 3 − 6 (B) 3 − 9 (C) 9 − 3
√ 3
(D) 2 3 + (E) none of these
2
36 Algebra

EQUATIONS: SIMULTANEOUS
1 1980 J.10 (61%)
2
A rectangle has perimeter 20 cm and area 21 cm . What are its
dimensions, in centimetres?
1
(A) 1 and 20 (B) 4 and 6 (C) 9 and 2 (D) 3 and 7 (E) 6 and 3
2
2 1978 I.8 (16%)
The graph of 2x − y + 1 = 0 intersects the graph of y = x2 in the points
A and B. The x-coordinates of the points A and B are the solutions of
the equation
(A) x2 + 2x + 1 = 0 (B) x2 − 2x − 1 = 0 (C) 2x + 1 = 0
2
(D) x = 0 (E) none of these

3 1981 J.21 (47%), I.12 (70%)


If ab = 12, bc = 20, ac = 15 and a is positive, then abc equals
(A) 360 (B) 3600 (C) 60 (D) 36 (E) 600

4 1983 S.18 (66%)


If

−a + b + c + d = x
a−b+c+d = y
a+b−c+d = z
a+b+c−d = w,

and b = 0 then x − y + z + w = kb for a particular value of k. This value


of k is
(A) 1 (B) 2 (C) 3 (D) 4 (E) 5
Equations: Surdic 37

5 1982 I.30 (6%), S.30 (6%)


Tina and Louise both celebrate their birthdays today. In three years,
Tina will be four times as old as Louise was when Tina was two years
older than Louise is today. If Louise is a teenager, Tina’s age is
(A) 17 (B) 29 (C) 25 (D) 21
(E) not determined by the given information

EQUATIONS: SURDIC
1 1982 S.17 (17%)
Solve for x: √ √
7x − 3x = 4
√ √ √ √
(A) 10 + 2 21 (B) 10 − 2 21 (C) 7+ 3
√ √
(D) 7 − 3 (E) 4

2 1979 S.20 (29%)


The solution of the equation
 
(x + 1) + (x − 1)
  =3
(x + 1) − (x − 1)

is
3 4 5 5
(A) 3 (B) (C) (D) (E)
5 5 4 3

EQUATIONS: TRIGONOMETRIC
1 1981 S.13 (4%)
◦ x
The number of solutions of the equation sin x = is
360
(A) 0 (B) 1 (C) 3 (D) 5 (E) infinite
38 Algebra

2 1983 S.20 (6%)


The number of distinct solutions of
 √ 
3
(2 sin θ − 1) sin 2θ + =0
2

for 0◦ ≤ θ ≤ 360◦ is
(A) 2 (B) 3 (C) 4 (D) 5 (E) 6

3 1980 S.23 (23%)


For all real values of x for which the terms are defined,
1 sin kx
cot x − cot x =   .
4 sin 14 x (sin x)

The value of k is
3 5 3 1 1
(A) (B) (C) (D) 1 (E) 1
8 8 4 4 2

EXPONENTS
1 1980 J.11 (50%), I.6 (78%)

Find a2 + b2 + c2given that a = 1, b = 2, c = 2.
√ √
(A) 5 (B) 10 (C) 3 (D) 5 (E) 9

2 1981 I.11 (27%), S.5 (79%)


 −a

−b −1
If a = 2 and b = 3 then 2 +2 equals
2 8 1 3
(A) (B) (C) (D) (E) 12
3 3 12 8
3 1980 S.9 (93%)
1 1
If x = 4 and y = 9 then x 2 y − 2 is
1 4 2
(A) −9 (B) (C) (D) 6 (E)
36 9 3
Expressions 39

4 1984 S.6 (60%)


m m
9 −3
For all values of m, equals
3m
3m − 1
(A) 9m − 1 (B) 8 (C) 2 (D) (E) 3m − 1
3
5 1983 I.19 (47%)
b a
For integers a and b we define a ∗ b = a + b . If 2 ∗ x = 100, then the
value of x is
(A) 2 (B) 3 (C) 4 (D) 5 (E) 6

6 1980 I.26 (16%), S.19 (72%)


 3n  
2 + 2−3n 23n − 2−3n equals
(A) 26n − 2−6n (B) 26n + 2 − 2−6n (C) 29n − 2−9n
2 2
(D) 29n − 2−9n (E) 46n − 4−6n

7 1979 I.25 (9%), S.13 (64%)


If a2b = 5, then 2a6b − 4 equals

(A) 26 (B) 246 (C) 242 (D) 12 5 − 4 (E) 8

8 1980 S.24 (29%)


1 7 1
Solve for x the equation 8 + x =
6 √ . 3
3− 2

(A) 2 2 (B) 4 (C) 8 (D) 27 (E) 64

EXPRESSIONS
1 1984 I.2 (67%)
2x + 1 − (x − 3) equals
(A) x − 2 (B) 3x − 2 (C) 3x + 4 (D) x − 4 (E) x + 4

2 1983 I.1 (68%), S.1 (87%)


5x − 2(4 − x) equals
(A) 7x − 8 (B) 3x − 8 (C) 7x − 6 (D) 3x − 6 (E) 4x − 8
40 Algebra

3 1980 J.13 (36%), I.4 (76%), S.3 (92%)


6(3 − x) − 2(1 − x) simplifies to
(A) 16 (B) 16 + 4x (C) 16 − 4x (D) 16 − 8x (E) 12 − 2x

4 1982 S.5 (43%)


The square of an integer is called a perfect square. If n is a perfect
square then the next largest perfect square greater than n is
(A) n + 1 (B) n2 + 1 (C) n2 + 2n + 1

(D) n2 + n (E) n + 2 n + 1

5 1980 J.12 (64%)


The number which is three less than twice a given number n is
(A) 2n + 3 (B) 3 − 2n (C) 3n − 2 (D) 2(n − 3) (E) 2n − 3

6 1979 J.13 (62%)


A truckload of nails contains x cartons. Each carton contains y boxes
and each box contains z nails. The number of nails in the truckload is
xy
(A) x + y + z (B) xy + xz + yz (C)
z
(D) x(y + z) (E) xyz

EXTREME VALUE
1 1979 I.15 (23%)
2
The minimum value of x + 4x + 1 is
(A) 3 (B) 1 (C) 0 (D) −2 (E) −3

2 1978 I.27 (28%), S.22 (37%)


2
If (a − b) + 6ab = 48 then the maximum value for ab is
(A) 0 (B) 24 (C) 6 (D) 8 (E) infinite
Factorisation 41

3 1984 I.25 (33%)


Let a, b and c be distinct integers from one to nine inclusive. The
a+b+c
largest possible value of is
abc
3 1 4
(A) 2 (B) (C) (D) 1 (E)
4 21 3
4 1982 S.21 (41%)
If p and q are positive integers, p > q, and (p + q) − (p − q)2 > 29 then
2

the smallest possible value of p is


(A) 7 (B) 3 (C) 6 (D) 4 (E) 5

FACTORISATION
1 1981 S.2 (89%)
n3 − n
For all values of n not equal to zero, has the same value as
n
(A) n3 − 1 (B) n3 (C) n2 − 1 (D) n2 − n (E) n

2 1979 S.2 (74%)


Which of the following does not have x + y as a factor?
(A) x2 + xy (B) x2 − y 2 (C) y 2 + xy (D) x2 + y 2 (E) 2x + 2y

3 1979 J.27 (11%)


A right-angled triangle has a hypotenuse of length p cm and one other
side of length q cm. If p and q differ by 1, then the length of the
remaining side, in centimetres, is
 
(A) p − q (B) (p + q) (C) (p − q)

(D) (p2 + q 2 ) (E) p2 − q 2

4 1978 S.26 (24%)


The number of distinct real numbers satisfying the equation
8
x3 + x − 8 = 2 is
x
(A) 0 (B) 1 (C) 2 (D) 3 (E) 5
42 Algebra

FRACTIONS
1 1983 I.6 (23%), S.2 (63%)
1
x−
y
equals
1
y−
x
x y −x
(A) (B) (C) 1 (D) −1 (E)
y x y
2 1984 S.3 (64%)
1 1 1
If = − then G equals
F H G
F −H FH 1 1 F − FH
(A) (B) (C) F − H (D) − (E)
FH F −H F H H
3 1978 I.26 (14%), S.19 (38%)
If
 n is apositive
 integer
  then
 the
 product

1 1 1 1
1− 1− 1− ... 1 − equals
2 3 4 n
1 n−1 2 2
(A) (B) (C) n (D) (E)
n n n(n − 1) n

FUNCTIONS
1 1979 S.5 (49%)
x
If f (x) = 4 , then f (x + 1) − f (x) equals
(A) 4 (B) f (x) (C) 2f (x) (D) 3f (x) (E) 4f (x)

2 1978 J.22 (17%)


 
2 1
If f (x) = then f (a) ÷ f equals
x a
1 1
(A) (B) a2 (C) 1 (D) 4a2 (E)
a2 4a2
Graphs 43

3 1978 I.22 (15%), S.15 (36%)


If f (n) = (n − 1)f (n − 1) for n > 1 and f (1) = 1 then f (4) equals
1 1
(A) 1 (B) (C) (D) 24 (E) 6
6 24

GRAPHS
1 1981 S.3 (92%)

The graph relating the distance a car DISTANCE  
travels to the time taken is a straight (km)  
line as shown. The graph indicates  
TIME (hours)
that the car is
(A) speeding up (B) slowing down (C) travelling uphill
(D) travelling at a constant speed (E) stationary

2 1978 I.13 (12%), S.7 (70%)


y
If the graph in the diagram represents 
a cubic, then the equation of the graph ............
... .......
.
..
... ... ..
..
is ....
..
...
..
..
.
.
..
... .. ..
..
.. ..
(A) y = (x + 1)2 (x − 2) ..
...
...
...
...
...
..
.
.
...
... .. ...
... .......... ...
(B) y = (x + 1)2 (2 − x) ....
2 ... ....
.
...
...
... ...
.. ..
(C) y = (1 − x)2 (2 − x) ...
..
..
..
..
..
.. .
.
..
..
.
.
..
..
... ...
.. ..
(D) y = −(x − 1)2 (x + 2) ..
... ...

...
... ..... .....
.. ..........
. ..
(E) y = (x − 1)2 (x + 2) −2 1 x
44 Algebra

3 1984 S.11 (19%)


Which sketch best represents the graph of the equation y = x(x2 − 1)? 2

(A)  ...
.. (B)  (C)  .
... ..
... ..
... ...
.
... .
..
...
...........
..... ....
............
...
.......
...
 ........... ............
..... ...... ....
.........................
 ....
.
...
...... .......... ......
..........
...

... .
... ..
... ..
... ...
..
...
... ...
... ..
.

(D) ........
...
 (E) ........
...
 ..
...
..

... ... ...


.
... ... ...
... ... ...
... ... ...
...
....
..
............
..... ....  ...
....
... .......
...

... ........... ... ...
...
. ...
.
...
...
.
.. .
.. ...
. .
... ... ...
...
... ... ...
... ... ...
.... ... ..

INEQUALITIES
1 1981 J.9 (70%)
If the integers n + 1, n − 1, n − 6, n − 5 and n + 4 are placed in
increasing order of magnitude, then the middle number is
(A) n + 1 (B) n − 1 (C) n − 6 (D) n − 5 (E) n + 4

2 1983 J.11 (27%)


The sides of a triangle have lengths, in centimetres, of 7 12 , 11 and x,
where x is a whole number. What is the smallest possible value of x?
(A) 3 (B) 4 (C) 6 (D) 2 (E) 5

3 1978 S.3 (55%)


The real number (|x| − 1) (1 + x) is positive if
(A) x > 1 (B) |x| > 1 (C) x < −1 or x > 1
(D) x > 0 (E) −1 < x < 1
Inequalities 45

4 1983 J.15 (14%), I.11 (27%), S.3 (75%)


If p is a number between 0 and 1, one of the following is true. Which is
it?
√ 1 √ 1
(A) p > p (B) > p (C) p >
p p
(D) p3 > p2 (E) p3 > p

5 1978 J.18 (16%)


If a > 0 and b < 0 which of the following must be true?
(A) a > −b (B) −a > b (C) a − b > 0 (D) −a > −b (E) ab > 0

6 1980 I.19 (19%)


If a, b and c are any numbers, and a > b then which of the following
must be true?
1 1
(A) > (B) ac > bc (C) a2 > b2
a b
1 1
(D) a + c > b + c (E) <
a b
7 1980 J.19 (35%), I.7 (52%)
If x > 5, which of the following is smallest?
5 5 5 x x+1
(A) (B) (C) (D) (E)
x x+1 x−1 5 5
8 1980 J.21 (27%)
Which of the following statements is true for all values (positive,
negative or zero) of the number c?
(A) 8c > 4c (B) 4c > 8c (C) 8c2 > 4c2
(D) 8 + c > 4 + c (E) 8 − 4c > 4 − 8c

9 1979 S.8 (54%)


If x − y > x and x + y < y then it follows that
(A) y < x (B) x < y (C) x < y < 0
(D) x < 0 and y < 0 (E) x < 0 and y > 0
46 Algebra

10 1979 S.17 (53%)


(x + 1)(x − 3)(x − 5) > 0 if
(A) −1 < x < 5 (B) x < −1 (C) 3 < x < 5
(D) 1 < x < 5 (E) −1 < x < 2

11 1978 I.21 (14%), S.14 (39%)


2
2x − 3x + 4
The solution of the inequality > 1 is
x2 + 2
(A) x < 1 or x > 2 (B) x < −2 or x > −1 (C) 1 < x < 2
(D) −2 < x < −1 (E) none of these

12 1980 I.27 (10%), S.21 (47%)


1
For what values of x is it true that < 4?
x−3
(A) all values of x (B) all values of x except 3 ≤ x ≤ 3 14
(C) only those values of x greater than 3 14
(D) only those values of x less than 3 (E) all values of x less than 3 14

LOGARITHMS
1 1978 S.1 (40%)
If  
(log 125)
log2 (log16 2) 5 = −a,

then the value of a is


1
(A) 0 (B) 1 (C) −3 (D) 6 (E)
4
2 1978 S.27 (41%)
If 0 ≤ x ≤ 1, then the maximum value of
  2
99 999x + 1
log10
1000
is
(A) 4 (B) 9 (C) 25 (D) 100 (E) 900
Operations 47

3 1979 S.14 (14%)


2 2
If 9 (log x) + 4 (log y) = 12 (log x) (log y), then
(A) x3 = y 2 (B) x2 = y 3 (C) x = y (D) x + y = 1 (E) 3x = 2y

4 1983 S.16 (17%)


An equivalent way of writing ax = y is x = loga y. If p = 14 , the value of
−p log2 p is
1 1 1 1 1
(A) (B) − (C) (D) − (E)
8 2 4 4 2

OPERATIONS
1 1978 J.20 (7%), I.9 (16%), S.6 (62%)
1
If the operation ∗ is defined by a ∗ b = then a ∗ (b ∗ c) equals
ab
1 a bc ab
(A) (B) (C) (D) (E) none of these
abc bc a c
2 1979 J.26 (11%), I.18 (42%)
In an algebra class, the students voted to have a new operation on
numbers called ‘super-multiplication’. They defined it by
1 1 1 1
a ∗ b = + + ab. The ‘super product’ of and 6, i.e. ∗ 6, equals
a b 3 3
1 1 1
(A) 2 (B) 18 (C) 5 (D) 2 (E) 8
6 2 3
3 1980 J.27 (18%), I.18 (41%)
For all numbers a, b the operation a ∗ b is defined by a ∗ b = ab − a + b.
The solution of the equation 5 ∗ x = 17 is
2 2
(A) 3 (B) 2 (C) −2 (D) 3 (E) 3
5 3
48 Algebra

PERCENTAGES
1 1978 J.14 (9%), I.5 (21%)
If each edge of a square is increased by 50% then the percentage increase
in area is
(A) 100 (B) 150 (C) 225 (D) 125 (E) none of these

2 1981 J.16 (17%)


The length of one pair of opposite sides of a square is reduced by 10%
and that of the other pair of sides increased by 10%. The area of the
new rectangle when compared with the original square is
(A) the same (B) 1% greater (C) 1% less
(D) 5% greater (E) 5% less

3 1978 S.9 (35%)


If each edge of a cube is increased by 60% then the percentage increase
in surface area is
(A) 28 (B) 60 (C) 156 (D) 1180 (E) 3996

4 1978 J.19 (19%)


A motor manufacturer makes m cars per week. The production is
increased by n%. The number of cars made per week is now
n mn
(A) m + n (B) m + (C)
 100 100
n  mn
(D) m 1 + (E) 1 +
100 100
5 1980 I.15 (10%)
A person’s wages increased by a% to $250 per week. The wage just prior
to this increase, in dollars per week, was
 a  a 100 + a
(A) 250 − × 250 (B) × 250 (C) × 250
100 100 100
100 250
(D) × 250 (E) × 100
100 + a a
Polynomials 49

6 1979 S.15 (51%)


An eager salesman offers you three successive discounts of 20%, 10% and
5% in any order you wish on the one purchase. The best percentage
discount you can get using all three discounts is
(A) 31.6 (B) 35 (C) 29.2 (D) 27.5 (E) 38.6

POLYNOMIALS
1 1978 S.23 (18%)
If
(1 + x)5 = a0 + a1 x + a2 x2 + a3 x3 + a4 x4 + a5 x5
then the value of
1
(a0 + a1 )(a1 + a2 )(a2 + a3 )(a3 + a4 )(a4 + a5 )
a0 a1 a2 a3 a4 a5
is
56 65
(A) 5! (B) (C) (D) 25 (E) 1
6! 5!
2 1978 S.30 (6%)
100 2
The remainder on dividing x by x + 3x + 2 is a polynomial R(x) of
degree less than 2. R(x) has constant term
(A) −299 (B) 2 − 2100 (C) 2101 + 1 (D) −2 (E) 2

3 1984 S.16 (9%)


99
The coefficient of x in the polynomial expansion
(x − 1)(x − 2) . . . (x − 100) is
(A) −5050 (B) −4950 (C) −99 (D) −100 (E) −4851
50 Algebra

PROGRESSIONS
1 1979 S.16 (26%)
A nest of boxes is made so that each box contains all those smaller than
it. The outermost weighs 1 kg, the innermost 0.1 kg. If their weights
decrease by equal amounts, and if the total weight is 11 kg, how many
boxes are there?
(A) 10 (B) 20 (C) 11 (D) 21 (E) 19

2 1980 S.25 (26%)


The sum of the series
     
1 1 2 1 2 3 1 2 3 4
+ + + + + + + + + +
2 3 3 4 4 4 5 5 5 5
 
1 99
··· + + ··· +
100 100

is
(A) 2725 (B) 2475 (C) 2985 (D) 3975 (E) 4950

3 1981 J.4 (71%)


What is the sum of the first 10 odd numbers?
(A) 100 (B) 25 (C) 110 (D) 55 (E) 19

4 1983 S.11 (39%)


The sum of 101 consecutive odd numbers is 12 827. The smallest of
them is
(A) 25 (B) 27 (C) 29 (D) 31 (E) 33

5 1984 J.20 (46%), I.14 (57%), S.8 (78%)


The positive integers are written consecutively in groups of five so that
the first row contains 1,2,3,4,5, the second row contains 6,7,8,9,10, etc.
The row which has a sum nearest 150 is the
(A) 5th (B) 6th (C) 7th (D) 8th (E) 26th
Ratio 51

RATIO
1 1978 J.17 (8%)
If car A has an average speed half as much again as the average speed of
car B for a journey of the same distance, then the ratio of time taken by
car A : time taken by car B equals
(A) 3 : 2 (B) 2 : 3 (C) 1 : 2 (D) 2 : 1 (E) 1 : 3

2 1984 J.11 (47%), I.6 (78%)


If $60 is divided among Anton, Barbara and Clare in the ratio 1 : 3 : 6,
what is Barbara’s share?
(A) $6 (B) $3 (C) $20 (D) $36 (E) $18

RECURSION RELATIONS
1 1980 S.15 (34%)
If f (1) = 5 and f (x + 1) = 2f (x) then what is the value of f (7)?
(A) 640 (B) 160 (C) 32 (D) 128 (E) 320

2 1979 S.19 (16%)


If x2n n
− xn−1 xn+1 = (−2) for n ≥ 1, and x0 = x1 = 1, then x3 is
(A) 1 (B) −3 (C) 3 (D) 5 (E) 13

3 1981 S.21 (17%)


If n is a positive integer and f (n) = f (n − 1) + 2n − 1 for all n > 1, and
f (1) = 1, then f (2n) is
(A) 2n2 (B) 4n2 (C) n2 (D) n2 + 1 (E) 2n
52 Algebra

4 1980 S.28 (50%)


Let f be a function such that for all integers m, n

(i) f (m) is an integer

(ii) f (2) = 2
(iii) f (mn) = f (m)f (n)

(iv) f (m) > f (n) when m > n.

The value of f (3) is


(A) 5 (B) 3 (C) 6 (D) 4 (E) 2

SUBSTITUTION
1 1978 I.1 (77%)
a − 2b
If a = 2 and b = −3 then equals
a+b
1 1
(A) 8 (B) −6 (C) −8 (D) − (E)
8 6
2 1981 J.10 (78%)
If y = x2 + 2x + 3, the value of y when x = 3 is
(A) 5 (B) 15 (C) 21 (D) 18 (E) 0

3 1981 I.3 (78%)


1
If a = 10 and t = 3 then the value of at2 is
2
1
(A) 45 (B) 90 (C) 30 (D) 22 (E) 450
2
4 1982 J.11 (47%)
xy
If x = 0.4, y = 1.1 and z = 0.2 then the value of is
z
(A) 0.022 (B) 0.22 (C) 2.2 (D) 22 (E) 20
Surds 53

SURDS
1 1982 S.4 (92%)
 √  √ 
3 + 5 6 − 2 5 equals
√ √ √
(A) 9 − 5 (B) 8 − 5 (C) 5−9 (D) 8 (E) −4

2 1978 J.26 (6%)



3

If the numbers 9, 5, 1, 2, 3 are arranged in order of magnitude, then
the middle number is
√ √
(A) 3 9 (B) 5 (C) 1 (D) 2 (E) 3
GEOMETRY

ANGLES
1 1981 J.6 (75%)

The size of the remaining interior angle of .....


... ...
this triangle is ... .....
.... .
.
...
. ◦.......
...
...
55 ...
...
(A) 45◦ (B) 55◦ (C) 80◦ .
...
...
.
. ...
...
...
.
.. ...
.. ...
(D) 90◦ (E) 100◦ .
..
.
..
.
..
. ...
...
...
.. ...
..
. ...
. ◦
.
...
..
. 45 ...
...

2 1984 J.5 (45%)


..
.......
In the diagram the value of x is ... ..
... ◦.....
..
..
...
x ...
...
...
... ...
(A) 10 (B) 60 (C) 50 .
.
..
.. ...
...
.. ...
. ...
..
(D) 30 (E) 70 .
...
.
.
. ...
...
...
. ...
... ...
◦ .....
120 .
.
...
... 130◦

3 1982 J.8 (73%)

In the right-angled triangle P QR the P


size of the angle P QR is  ◦
  75
(A) 30◦ (B) 10◦ (C) 20◦  
 .......
(D) 25◦ (E) 15◦ ..
Q R
4 1982 J.5 (82%), I.2 (95%)

P Q is a straight line. The size of angle P T S S...


...
is ...
...
...
...
...
(A) 95◦ (B) 85◦ (C) 90◦ ...
...

◦ ◦
...
...
... ......
R
(D) 100 (E) 80 ...
...
......
◦ ..............
70 ...
... ....... .
..
..

30
... .......
....
P T Q
Angles 55

5 1981 J.8 (57%), I.2 (84%)

The value of x in the figure is


(A) 20 (B) 70 (C) 110 x◦ 140◦
......
..... ......
...... ◦ ..........
......
.
......
. x ......
......
(D) 140 (E) 220 ..
.
.
......
.....
.
. ......
......
......
.
.... ......
...
. ......
...... ..

6 1983 J.9 (35%)


...
...
In the figure the value of x is ◦............
...
.... ....x
..... ...
..... ...
(A) 142 (B) 72 (C) 107 ...... ...
...... ...
...
....... ...
...
..... ...
(D) 108 (E) 145 ...
..... ...
.... ◦ ...
.
....
.
..
. .
....
..
35 ...
.. 107◦

7 1979 J.4 (44%)

In the diagram ADC is a straight line. The B...


...
size of  BDC, in degrees, is ...
...
...
...
...
(A) 20 (B) 50 (C) 80 ...
...
...
...
..
(D) 100 (E) 120 ◦ ......
4x 5x◦ ...

A D C

8 1978 J.4 (56%)

In the figure, P Q and RS are intersect- P ............................ ......... S


........
.........
ing straight lines, then x + y equals .........
......... ◦ .......................
......... 150 ...
◦ ............................................ ◦
.
x .
.........
......... y
(A) 15 (B) 30 (C) 60 .........
........
.........
.........
.........
.........
.........
.
...
.......... .........
(D) 180 (E) 300 R .
...... ....
Q

9 1980, J.4 (84%), I.2 (92%)

In the diagram, P OR is a straight line, R


..
...
and the size of  QOR is 38◦ . The size ...
...
....
of  P OR, in degrees, is ..
...
...
....
(A) 76 (B) 142 (C) 52 38◦ .
..
P O Q
(D) 128 (E) 322
56 Geometry

10 1983 I.2 (92%)


...
.....
.....
The lines l and m are parallel as shown. ..
......
.
.... ◦
The value of x in the diagram is .....
.....
.....
x l
..
......
.
....
.....
(A) 140 (B) 50 (C) 320 .
..
.....
.....
..... m
◦........
.
40 .....
(D) 180 (E) 40 .....
.....
.....

11 1984 J.14 (44%), I.9 (53%), S.4 (71%)


.
◦......
The sides of a quadrilateral are ex- 75 ....
............................... ....
...
...
...
...
...
...
...
...
...
...
.
tended to make the angles whose sizes ................................ ...
...

x ...
...
are shown. What is the value of x? ...
...
...
...
...
(A) 100 (B) 90 (C) 80 ...
...
...
... 115◦
(D) 75 (E) 70 90 ◦

12 1982 J.17 (24%)

In the diagram, the size of the angle S..


.. ◦
QP S is P .............5x
...........
...
.. ..........
...........
... ...........
...
(A) 90◦ (B) 96◦ (C) 60◦ ....
...........
...........
...........
.. ◦...........................
(D) 105◦ (E) 108◦
...
... 72◦ x ...........

R Q
13 1981 I.16 (20%), S.11 (33%)
The smaller angle between the hands of a clock at 12:35 is
1◦ 1◦
(A) 167 (B) 150◦ (C) 165◦ (D) 180◦ (E) 162
2 2
14 1980 J.23 (17%)
A bicycle chain travels over a front sprocket with 48 teeth and passes, as
usual, to the sprocket on the rear wheel axle which has 18 teeth. For
each complete revolution of the back sprocket, what is the angle through
which the pedals turn, in degrees?
1
(A) 135 (B) 360 (C) 960 (D) 120 (E) 67
2
Angles 57

15 1980 I.20 (9%)

In the diagram, P OS is a line through ..........


.........................
... R
...... ...............
..... ......... ... ........
the centre O of a circle of radius r cm. . ..... .................. ....
r
...
Q .
.................
................
. .... ...
...
The line P QR is drawn so that P Q is r .......... .... ..................
..
... ◦ ...
.....
.........
.
...
...
..... 60
......... ..
....
.. ...
..
of length r cm. If  ROS is 60◦ , then P ... ... S
...
... O ...
...
 OP Q, in degrees, is ... .
... ...
..... ...
...... ....
....... ......
.....................................
(A) 10 (B) 15 (C) 30
(D) 25 (E) 20
16 1982 S.10 (50%)

A wheel of radius 10 cm rests against a step 5 cm


.....
.....
high as shown. The wheel is rotated about Q by ...
....
...
pushing it until its centre is directly above Q. The ...
...
O
..... ....
spoke OQ has now been rotated through an angle ... 10 ............
... .....
... ............
.......
equal to Q...
...
..
...
5 ....................... .....
.....
.. ..................................
◦ ◦ ◦
(A) 30 (B) 75 (C) 45
◦ ◦
(D) 60 (E) 90
17 1980 J.28 (17%)
.
............
....... ........
Angles a◦ , b◦ , c◦ and x◦ are shown. What is ..
...........
....... c .....
.....
.....
.......
the value of x? .
.............. ...
.........
.
.....
.....
..... ......
..... ..
.. .....
.....
...
.....x .....
....... .....
.....
.......
(A) 360 − (a + b + c) (B) a + c − b ...
..... ... .......
............... ........
.....
.....
.......
.......
.......
b ...
...
..
(C) a + b + c (D) 360 + b − a − c .......
.......
....... .......
....... ...
...
(E) 360 + a + c − b
.......
a
....... .....
..........
..

18 1978 J.28 (19%), I.20 (38%)

In the diagram ABX and ACY are straight B ..............


........... X
.......................

lines. The bisector of  XBC meets the bi- .............. . .
........
..................... • ... .....
.
............... ...
...
......................
sector of  BCY at Z. If  BZC is 80◦ then
...
A ..........
.......... ...
...
...
...
 BAC, in degrees, is
..........
..........
.......... .
.
...
◦ ...
........ ...........
. Z
.......... .. .........
.................

..........
..........
C ...........
........
Y
(A) 10 (B) 20 (C) 80
(D) 100 (E) none of these
58 Geometry

19 1983 I.28 (5%), S.21 (16%)

If the exterior angles (x , y  , z  ) are in the ra- ......


......
...
tio 4 : 5 : 6, then the interior angles (x, y, z) z  ........ z.......................
......
... ......
are in the ratio ... ......
......
... ......
.... ......
......
.. ......
... ......
(A) 7 : 5 : 3 (B) 3 : 2 : 1 (C) 4 : 2 : 1 ...
.... x y
......
......
...... y
.....
..
...
(D) 8 : 5 : 2 (E) 6 : 5 : 4 ... x

AREA
1 1984 J.9 (32%), I.4 (64%)

In the diagram, lengths are marked in cen- 4



.. ....
....... .....
timetres. The area, in square centimetres, 
3 

of the given figure is 5 5

(A) 45 (B) 35 (C) 41 ........ ........


.
(D) 32 (E) 55 10

2 1978 J.9 (12%)


A rectangle is 150 cm by 50 cm. The area of the rectangle, in square
metres, is
(A) 7500 (B) 75 (C) 7.5 (D) 0.75 (E) 750

3 1979 J.8 (25%)


The side of a square has length 250 cm. The area of the square, in
square metres, is
(A) 6.25 (B) 625 (C) 62 500 (D) 62.5 (E) 0.625

4 1980 S.6 (61%)

In the diagram lengths are shown in cen- ....


....................................................................................
................ ........................................
timetres. What is the area of the shaded ................. ......................................
.................. ...... . . . . . . . . . . . . .
....... . . . . . . . . . . .
.................... ....... .....................
part of this rectangle in square centime- ....................
..................... .
...... . . . . . . . . . .
....... . . . . . . . . 5
...... . . . . . . . .
. ....................
tres? ......................
.......................
.........................
.
.
....... . . . . .
...... . . . . .
..............
...... . .
.......................... ...... . .
. . . . . . . . . . . . .. . .......
......
(A) 37.5 (B) 12.5 (C) 25 5 5
(D) 50 (E) 35
Area 59

5 1978 S.9 (35%)


If each edge of a cube is increased by 60%, then the percentage increase
in surface area is
(A) 28 (B) 60 (C) 156 (D) 1180 (E) 3996

6 1981 I.6 (22%)

The rectangle P QRS is divided into a P Q


square and three rectangles as shown. The x2 5x
area of the square is x2 cm2 , and of the rect- ..........................................................
.......................................
angles 5x cm2 and 3x cm2 as shown. The .......................................
.......................................
.......................................
area, in square centimetres, of the shaded 3x .......................................
.......................................
.......................................
rectangle is .......................................
.......................................
.......................................

(A) 15 (B) 15x2 (C) 8x2 S R


(D) 15x (E) 8x
7 1983 I.9 (54%), S.4 (78%)

The rectangle P QRS measures P T 3 Q


.................
................ . . ...
12 cm by 8 cm. Points T , U , 2 . ... ... .
... ......
....
......................... ...............
............................................................
.

............... . . . . . . . . . . . . . . . . . . ...
............................................................................................
V and W are on the sides with W ... . . . . . . . . . . . . . . . . . . . . . . . . ..
........................................................
.... . . . . . . . . . . . . . . . . . . . . . . . ....
measurements, in centimetres, as ......................................................
.... . . . . . . . . . . . . . . . . . . . . . . . ...
.......................................................
....................................................... 8
shown. Find the area, in square ... . . . . . . . . . . . . . . . . . . . . . . . . ...
.......................................................
.... . . . . . . . . . . . . . . . . . . . . . . . ....
.......................................................
centimetres, of the shaded por- ........................................................
... . . . . . . . . . . . . . . . . . . . . . ..............
... . . . . . . . . . . . . . . . . . ................... U
................................................. .
tion. .... . . . . . . . . .......................
.................................... . 2
.... . .................
............. 
S 3 V
(A) 36 (B) 48 (C) 42 12 R
(D) 24 (E) 60
8 1982 J.13 (30%), I.9 (56%)

P QRS is a rectangle. T is a point on P Q P . T Q


.........
... ..........
such that P T is of length 2 units. QR is ...
...
.
......
......
......
... ......
of length 3 units. SR is of length 7 units. .... ......
..
. ......
...
. ......
......
The area of triangle QRT , in square units, ..
.
.... ......
......
......
...
. ......
is .
.
.
.
.... ......
......
..

1 1 S R
(A) 10 (B) 14 (C) 6 (D) 15 (E) 7
2 2
60 Geometry

9 1982 I.14 (21%), S.9 (60%)


...
...
...
The given right-angled triangle has one side twice the ..
.
....
...
length of another. The area of the triangle, in square ...
...
...
units, is 10 ..
...
.
. 2y
...
...
.
....
..
50 100 ...
...
(A) 40 (B) 50 (C) (D) (E) 20 .
...
... .......
9 3 .
.. .
y

10 1984 J.16 (17%), I.13 (33%)


• • • • ... ........ ........ ........ ...
......
...... ...
...... ...
In the diagram the grid points are placed .
.
....
.
..
........
• • • • • ...
...
...
...
.
...... ...
at 1 cm intervals. The area, in square cen- • • • • • • • •
..
...
...
...
. ..
timetres, of the enclosed figure is ....
• • • • • • • • •
.
.
....
...
.
. ..
(A) 50.0 (B) 50.5 (C) 51.0 ...
..
.
• • • • • • • • • ....
.. .
•... • • • • • • • •............•
(D) 51.5 (E) 52.0 . ....
•.... • • • • • • ....•..................
.. .....
.
•........•........•........•........•........•........•.

11 1982 J.21 (30%), I.11 (51%)

The rectangular block shown has 3cm


surface area, in square centime- ... 
 ..... 
tres, of ...
.
 
..
(A) 388 (B) 373 (C) 365 ..
7cm
10cm ...
.
(D) 358 (E) 395 ...
...... ....... ....... ....... ....... ....... ....... ....... ....... ....... ..

..
.
.....

5cm
.....
10cm
Area 61

12 1981 J.15 (35%)

P QRS is a rectangle in which P Q = P T


.. .. . . . . . . . .
U Q
.......................... ...................
.................... ... . . . . . . . .
12 cm and QR = 8 cm. V is the mid- ....................
.....................
... ..........................
...................... ......................
point of RS. T and U are points on ...................... ......................
.... . . . . . . . . .
........................ ..................................
........................ ........................
P Q such that P T = U Q = 2 cm. ........................
.......................... ..........................
..........................
..........................
The shaded area, in square centime- ..........................
...........................
............................
.
............................
............................ ............................
tres, is .............................. .............................
..............................
............................. .. . . . . . . . . . . . . .
............................... ..............................................
............................... ................................
(A) 32 (B) 56 (C) 96 .................................................................
..

(D) 64 (E) 80 S V R

13 1981 I.14 (31%)

P QRS is a trapezium with P Q  SR. P  T Q


  
If P Q = 20 cm, SR = 12 cm, and 
the area of RST is 60 cm2 , then the    
   
area of trapezium P QRS, in square 
  
centimetres, is S R

(A) 180 (B) 320 (C) 300 (D) 160 (E) 150

14 1979 I.19 (22%), S.12 (40%)

ABCD and P QRS are two squares of side A B


10 cm. As shown in the diagram P is
placed at the centre of square ABCD and Q
P ...........................X .....................
................................ ...
BX = 4 cm. The area of quadrilateral ...
...
...
. ...
...
...
... ...
P XCY , in square centimetres, is ...
...
...
...
... ...
... ...
... ...
(A) 21 (B) 25 (C) 30 D Y ...
... C ...
...
... ...
... ...
(D) 24 (E) 28 ...
........ ...
..
...
................................................................
. .
R
S
62 Geometry

15 1980 I.24 (8%), S.17 (39%)

P QRS is a quadrilateral in which SP = SR, P 8 .. Q


.......
 P SR = 60◦ and  P QR = 90◦ . The length
of P Q is 8 cm, and the length of QR is 6 cm.
What, in square centimetres, is the area of 6
......
P QRS?

√ 25 3 ....
...........
...........
(A) 25 3 + 24 (B) + 24 ...........
...............
.
R
2 √ 60 ..
..
..
◦ ...................... ..
..
..........
...........
√ 25 3
(C) 25 2 + 24 (D) 48 + S
√ 2
(E) 48 + 25 3
16 1980 J.17 (18%)

 3x 
The diagram shows a circle enclosed in a ............................................
square. The diameter of the circle is 2x cm.
............................................
............. ................................. ..................
.................... ........ . . . . . . .

....... ........ ....................
..... . . . . . .
The square has sides of length 3x cm. What is ...........
.........
.... . . . . .
..............
... . . . . .
........ .............
the area of the shaded region, in square cen- ........
........ 
... . . . .
............
..... 
....... 2x ... . . . .
timetres? ........
......... .
............
.
.................
3x
......... ... . . . .
.
............. ......
.... . . . . .
..... ............
(A) 5x2 (B) (9 − π)x2 (C) 9 − π .................
........................ ....... . . . . . . .
........ . . . . . . .
...........................................................................
............................................
2
(D) (9 − 2π)x (E) (9 − 4π)x2 ............................................
............................................
............................................


17 1982 I.20 (21%), S.13 (29%)

OP Q is a quadrant of a circle and semicircles P ..


...........................
...................
....... .........
are drawn on OP and OQ. Areas a and b are .. . ...... . ........
...... . . ......
..................
a .... . . .......
... ..............
shaded. Find . .... . . . ......
...................
b ....................
... . . . . . ... b
...................
.................................................................
.
1 1 π .
..
..
..
....
.
.............. ... ........ . ..
....... . ...
(A) √ (B) (C) ..... . . . . . ..
...... . . . . . .
................ ...
............
..... ..
12 2 4 .... ................
.. ................
. a
.... ..
......
......
.....
π ............................... .....
... ........ ......... ....
(D) 1 (E)
3
.......................
..............  ...

O Q
Area 63

18 1980 J.25 (4%), I.25 (6%), S.14 (22%)


A beetle crawls round the outside of a square of side 1 metre, at all times
keeping precisely 1 metre from the boundary of the square. What is the
area enclosed, in square metres, in one complete circuit by the beetle?
(A) π + 4 (B) 5 (C) 2π + 4 (D) π + 5 (E) 9

19 1983 I.22 (5%), S.14 (17%)

An archway is constructed on a straight base R


.......
...... ...........
.....
P Q with two circular arcs P R and QR. Arc ..
.
.....
.....
.....
.....
.....
..... .....
.... ....
P R has centre Q and arc RQ has centre P . ...
.
.
. ...
...
...
... ...
If the length of P Q is 2 m, then the area, in .
.
..
.
.. ...
...
.. ...
.
square metres, under the archway is ....
...
...
...
...
... ...
4π 4π √ 4π √ ...
...
...
...
...
(A) (B) − 3 (C) + 3 ...
. ..
3 √ 3 3 P Q
3π 4π √
(D) (E) −2 3
4 3
20 1982 S.16 (5%)
A rhombus has half the area of the square with the same side length.
The ratio of the long to the short diagonal is
√ √ 1 √
(A) 2 + 3 (B) 2 − 3 (C) (D) 2 (E) 3
2
21 1978 S.17 (6%)
......
A regular octagon is formed by cutting off the cor- 

........... .........
ners of a square, as shown in the diagram. If the
.
 .
 
square has sides of length n cm, the total area re-
............ ............
moved, in square centimetres, is
√ √ 4n
(B) 4 2n2 (C) n2 − .......... 
(A) 4(n − 2)
. 
...........
.
√ 2  
........
(D) n2 (3 − 2 2) (E) none of these
22 1982 I.25 (6%)
The area of a circle circumscribed about a regular hexagon is 2π. The
area of the hexagon is

√ 3 √ √
(A) 6 (B) 3 3 (C) (D) 3 (E) 6 3
2
64 Geometry

23 1982 S.25 (9%)

In the figure RP Q is a right angle and ST is Q ...


............
parallel to P R. The lengths of P Q, P R, ST  ............
...........
............
............

.
are shown. The area, in square centimetres, T ......
............
.
...
...
..
...
...
....
4 cm  S
of triangle RQT is ..
...
...
 6 cm
(A) 12 (B) 24 (C) 6 (D) 16 ....
.. 
(E) not determined by the given information
...
...
....

..
... 
...
...
......
...
R 8 cm P

24 1983 J.19 (20%), I.12 (28%)

P and Q are midpoints of the sides of the Q


 ....
.. .
square as shown. What is the ratio of the .........
.

area of the shaded triangle to the area of  .................


................
 ...................
.
. . . . . . . . ..
.
the square?  .
.......................
.......................................
.
..............................

...................................................
.
.................
1 3 1 P  ..................................
.
(A) (B) (C) .............................
4 8 2  ............................
...................
........................
5 3  ...................
 ..............
.
.
(D) (E)  ............
6 4 ....
.........
 ..

...

CIRCLES
1 1979 J.11 (24%)
..................................................................................
. ...... ..... . . .....
... .....
..... .
... .
Two circles of equal size are just contained in a .
..
... ......
.....
....
...
.... ... ..
rectangle as shown. If the radius of each circle is ...
...
... .
.....
...
.......
..
....
..
.... ..
. . .... .
...
1 cm then the shaded area, in square centimetres, .
.. ..... ...
.
.. . . .. . ..
.. .
. . ............................................................................
.
.

is
(A) π − 4 (B) 8 − 2π (C) 8 − π (D) 4 − 2π (E) 4
Circles 65

2 1981 S.8 (53%)

The circle is inscribed inside the triangle P


P QR. SR = 7 cm, QS = 4 cm, T P = 
4 cm. The perimeter of the triangle P QR,  
in centimetres, is 
.
....
.
.. .........
....................................
.......
T ......... ..........
(A) 30 (B) 50 (C) 15π .....
..
. 
...
...

 ......... ...
... 
...

(D) 11π (E) 60  ...


...
.....
...
... 
.....
 
......
........ ......
.............................
Q S R

3 1983 S.8 (52%)


..............................................
......... .......
....... ......
Chords are drawn in a circle as shown. The .......... .....
. .....
..... .....
....
. ...
value of x is ....
.
.
..........
...
....
. ◦ ◦.............. .......
...
.. ........
. y
......
......
x ... ...... ...
...
(A) 44 (B) 48 (C) 52 ..
.... ......
......
...... .....
......
..
...
...
...
..... ◦ ..
...
......
z
......
...... ........ ......... .
...
(D) 84 (E) 40 ...
... .........
.
. .96 ◦ .
.
..
..
... .... ......... ...
...... ......
... ...... ...... ...
... ...... ...... ..
...
... .......... ...... ....
... ..... ◦ ......
...... ......
..... ..........
.......
.....
44 ..........
.....
......
....... .........
......... ...........
..........................................

4 1981 I.10 (24%), S.4 (53%)


.........................................
......... .. ........................ . ...............
P Q is the diameter of a circle with centre ....... ....
.....
.
... ......
. R .
.
..
............. ..
. .........................
............. Q
.....
.... ..
O. R is a point on the circumference so
..
.
.
...
.. .
...
..
.
 ...
...
...

. ...
that P O = OQ = QR = 1. The length of ....
..
.
....
..
. ....
. ...
...

... ... ...
P R is ...
... .
.
....
...
...
... .. •
 ...
√ 3 ...
... .
..
..
O ..
...
(A) 5 (B) 1 (C) ...
... .
.
...
..
 .... ...
..
.
.


...
2 ...
... ...
...
. ..
...
√ π ... ... ...

..... . ...
...... ....
(D) 3 (E) .....
...... ......
....
.....
2 P .......
.........
........................................ ....
.........
.
66 Geometry

5 1978 J.15 (7%)


A chord of length 10 cm is drawn in a circle of diameter 26 cm. The
distance, in centimetres, of the chord from the centre of the circle is
(A) 13 (B) 12 (C) 10 (D) 24 (E) 5

6 1978 S.11 (26%)


A circle is inscribed in an equilateral triangle. If the circumference has
length 3 cm, then the perimeter of the triangle, in centimetres, is
√ √
18 9π 3 9 3 6
(A) (B) (C) (D) 6 (E) √
π 2 π π 3
7 1978 S.2 (21%)
A plane figure consists of a circle and a pair of parallel lines tangent to
the circle. The number of points in the plane equidistant from the circle
and the two lines is
(A) 1 (B) 2 (C) 3 (D) infinite (E) none of these

8 1984 I.15 (14%), S.9 (39%)


....
...........................................
......... . . . . . . . . ........
P QRS is a diameter of a circle whose ra- ....................................
....... . . . . . . . . . . . . . .......
..
................................................
..............................................
dius is r. The lengths P Q, QR, RS are .... . . . . . . . . . . . .......................................
................................... ........ ...
... . . . . . . . . . . ....... ...... ...
.......................................... ..... ..
equal. Semicircles are drawn on P Q and .
............................
......
......
............................. .....
........................... ...
QS to create the shaded figure shown in ......................... ...
P ........................
.... . . . . . . . . .... • ..
.. S
the diagram. The perimeter of the shaded ..........................
........................... Q R .....
.
... ........................ . .
...
figure is ... ............ . ............
...
...
......... ...
...
... ....
.... ..
4πr 5πr .....
..... .....
.....
(A) 2πr (B) (C) ...... ......
......
........ ...
........
3 3 ........... ...
.....................................
.

3πr 31πr
(D) (E)
2 18
Circles 67

9 1978 J.24 (17%), I.18 (16%), S.20 (32%)


........................
......... ......
.....
Three pipes of diameter 1 m are held together ....
..... .....
...
...
..
.
. ....
....
by a taut metal band as shown. The length (in .
..
.......
......
.......
.
... .. ... ..
... ..... .. .....
metres) of the metal band is .... ... ..
.
. ...
...
.... ...
.. ....
...
... .................................. ..... .......... .
π ............... .......... ......
........... ....................
... ......................... .......
(A) 3 + π (B) 3 (C) 3 + ....
.
. .... ...
. .
. ....
...
...
2 ....
...
... ..
......
....
...
...
.... .
3+π ...
..
...... ..
...
(D) (E) 6 + π ...
... .
.
.
.
.. .... .
.
....

2 ...
.....
......
......... .....
....
. .
... .......
......
......... .....
....
...
.

........................ ........................

10 1984 I.18 (5%)

The tangent at P to the smaller of two con- R


............................................
.......... . . .. . . . ..........
........ . . . . . . . . . .........
centric circles intersects the outer circle at Q ....
....... .......................... .......
..................................................
.
..............................................
and R. The length of QR is 14 cm. The .... . . . . . . .... . . . . . . . . . . . .....
................................................
... . . . . . . . .. . . . . . . . . . . . . . ..
... ..................................................................................... .....
shaded area, in square centimetres, between ... .......................
... ...................
....... . . . . . . . ..
.................. ...
... . . . . . . . ...
... ................. ................. ..
the circles is ..........................
.....................
P ....................
.
.. . . . . . . ...
....................... .. ................
49π ... ..................
... ................... ................. ....
.
.. .................
(A) 196π (B) 49π (C) (D) 49 ... . . . . ..... . . . .......
... ...... ......... .............................................. ....
...
................... .
4 ................................................
... . .... . . . . . . . . . . . . . . . . . . ...
................................................
(E) not determined by the given information ...... . . . . . . . . . . . . . . . . . .....
...........................................
...... . . . . . . . . . . . . . . ......
Q ....... . . . . . . . . . . . . . .......
....................................
........ . . . . . . . . .........
............... . . . ..............
................

11 1981 I.22 (9%), S.15 (20%)


A ball 13 cm in diameter is floating so that the top of the ball is 4 cm
above the smooth surface of a pond. What is the circumference, in
centimetres, of the circle formed by the contact of the water surface with
the ball?
 √ 
(A) 12π (B) 18π (C) 6π (D) 4 22 π (E) 24π

12 1980 J.26 (12%), I.23 (15%), S.10 (29%)


...
In the diagram, R and P are points ......
......... ....
P
...
........... ...
on a circle with centre O. P Q has .
....
.........
.........
.. ...
...
...
.......... ...
length 50 cm and QR has length ..
...
.. .
.......
...
.. .
....... ...
...
...
.
.. .
..... ...
..
10 cm. P Q is perpendicular to OR. ..
...
.. .
.......
...
.. .
....... ...
...
.
.. .
..... ...
.
What, in centimetres, is the radius .
...
... .
........
. ......
.
...
..
O ... .
..
.. R
of the circle? .. Q

(A) 240 (B) 120 (C) 250 (D) 130 (E) 260
68 Geometry

13 1978 S.21 (57%)


A satellite travels in a circular orbit around the earth. The orbit has a
length (circumference) of 60 000 km. By how many kilometres would this
be lengthened if the radius of the orbit were increased by 1 kilometre?
(A) 2π (B) 1 000 (C) 60 000 (D) 2π × 60 000 (E) no change

14 1979 S.18 (34%)


The length of the common chord of two intersecting circles is 16 m. If
the radii are 10 m and 17 m, a possible value for the distance, in metres,
between the centres of the circles is

(A) 27 (B) 21 (C) 389 (D) 15 (E) 11

15 1978 I.28 (11%)

ABC is a triangle with sides 3 cm, 4 cm A


and 5 cm respectively. A circle is inscribed 

in the ABC as shown. The area of the 
circle, in square centimetres, is
........
...... 
.........................
......

.....
..... ...
... ...

.
.... ...
...
....
(A) 1 (B) 2 (C) π ...
..
...
. 

...
... ...
... .
3π 
... ..
.
..... ...
(D) (E) 2π ....

......
......... ......
.......................
2
B C

16 1983 I.23 (19%), S.17 (35%)

The triangle P QR, right-angled at Q, S ................................................


V
......... .......
has semicircles drawn with its sides as ....... ......
..
.......
. .....
.....
P ..... .....
....
diameters. The sides of the rectangle ...
. .... . . ...

.
......... ...
.... ...
...... ...

.
.
...
ST U V are tangents to the semicircles ...
.. ...
...

. ...
.... ...
and parallel to P Q or QR, as drawn. ....
...
 ...
.
...
...
6  ...
...
If P Q = 6 cm and QR = 8 cm then ...
...  ...
...


... ..
..... ..
the area of ST U V , in square centime- ..... ...
R
....... . .. ... .
.
............ .. .
...... . ...
tres, is Q
...
...
... 8 ...
...
... ...
... ...
(A) 121 (B) 132 (C) 144 ... ...
... ...
.....
..... .
.......
...... ...
....... ......
(D) 156 (E) 192 ..........
................................
.......

T U
Circles 69

17 1979 S.28 (8%)


15 billiard balls are lying on a table in such a way that they are just
squeezed inside an equilateral triangular frame whose inside perimeter is
876 mm. The radius of a billiard ball, in millimetres, is
73 146 146
(A) (B) √ (C) √
2 4+ 3 2+ 3
146
(D) √ (E) none of these
3+ 3
18 1981 I.27 (22%), S.25 (26%)
.................................
..................... ................
............... ..... ........
A circle of radius 2 cm with centre O, con- ....... .....
.
......... ....... ..... ......
... ......
...
.... .... ... .....
.... ... .....
tains three smaller circles as shown in the .
.
.
...
..
......
...
.
.....
...
.... .
...
.. ...
.
...
...
diagram; two of them touch the outer cir- ..
.
... ...
... ... .. ...
...
..
. . . .. .............
........ .....
... .....
cle, and touch each other at O, and the ...
.... ...
....
..... ...........
. . . . .. ........ ...
......
.. ...... ...... .....
........ ....
third touches each of the other circles. The ...
.....
........
.....
........................................ ....
......... ...
..
.
... .. ... .. ..
.....
O .
........ .....
radius of this last circle, in centimetres, is ...
...
... ..........
. .. .
.....
....... .
... .
.
..
...
. .
... ............ ..... ..
...
... ... ... .................... ...
2 1 1 ...
... ...
... ...
... ...
..
(A) (B) (C) ...
...
.
...
... ..
.. ...
...
.

3 2 3 ....
.....
.....
...
... ...
.
.....
....
..
..... . ... .....
5 ...... ......
....... ..... ..
...
.... ...........
.....
(D) 1 (E) ........ ......
..................
..
...... .........
....................................................
6

19 1983 S.23 (15%)


◦ 
The latitude of Canberra is 35 19 S. At its highest point in the sky
when viewed from Canberra the lowest star in the Southern Cross is
62◦ 20 above the southern horizon. It can be assumed that rays of light
from this star to any point on the earth are parallel. The northernmost
latitude at which the complete Southern Cross can be seen is
(A) 0◦ (B) 27◦ 01 S (C) 27◦ 01 N (D) 7◦ 39 N (E) 7◦ 39 S
70 Geometry

20 1984 S.28 (6%)


.......
.......
Two sections of straight road, each run-  ..
..
.....
..
ning east-west, and located as shown, are 900m .
..
to be joined by new roadway consisting ...
..
.....
of the arcs of two circles of equal radius. .. ...
..... ....

The existing roads are to be tangents at 1200m

the joins, and the arcs themselves are to have a common tangent where
they meet. The radii of these two circular arcs, in metres, are
(A) 600 (B) 625 (C) 450 (D) 750 (E) 500

COORDINATE GEOMETRY
1 1978 I.4 (48%)
The line with equation y = 3x − 6 passes through the point (a, 2). The
value of a is
8 3 2
(A) 2 (B) 0 (C) (D) (E) −2
3 8 3
2 1978 I.11 (34%)
If (4, 2) is the midpoint of the line joining (x, 4) and (3, y) then x + y
equals
(A) 5 (B) 6 (C) 7 (D) −7 (E) 0

3 1979 I.8 (33%)


y
The equation of the line AB is 
...
...A
...
3 ...
...
...
(A) y = −x − 3 (B) 3x + 2y − 6 = 0 (C) y = −x + 3 ...
...
...
...
...
(D) 2y − 3x + 6 = 0 (E) 2x − 3y − 9 = 0 ...
...
...
...
...
...
...
...

...
...B
...
O 2 .... x

4 1980 S.5 (71%)


A circle has its centre at (−3, 4). The line y = −2 is a tangent to it. The
radius of the circle is
(A) 1 (B) 2 (C) 5 (D) 6 (E) 7
Coordinate Geometry 71

5 1983 S.5 (82%)


The gradient of a straight line is − 32 and it cuts the x-axis at the point
(4, 0). The equation of the line is
(A) 2y − 3x = 6 (B) 2y = −3x + 12 (C) 2y = 3x + 8
(D) 2y = 3x − 12 (E) 2y = −3x + 8

6 1978 S.18 (29%)


If the point (x, y) is first reflected in the line x = 0 and the resulting
point is then reflected in the line y = 1, then the image point has
coordinates
(A) (x, 1 − y) (B) (0, 1) (C) (−x, 1 − y)
(D) (−x, 2 − y) (E) (y, −x)

7 1980 S.20 (15%)


A regular coordinate system has axes with scale 1 cm as the unit of
length. P QR has vertex P at (0, 3), Q at (4, 0) and vertex R at (k, 5),
where 0 < k < 4. If the area of the triangle is 8 cm2 , then the value of k
is
8 13 1
(A) 1 (B) (C) 2 (D) (E) 3
3 4 2
8 1979 I.29 (10%)
Two parallel lines intersect the x-axis cutting off a segment of length
3 cm and the y-axis cutting of a segment of length 4 cm. The
perpendicular distance, in centimetres, between these two lines is
12 5 4 3
(A) 5 (B) (C) (D) (E)
5 12 3 4
9 1984 S.21 (11%)
In a rectangular coordinate system with scale 1 cm = 1 unit on each axis,
we have A(0, 8), B(6, 0), C(x, 12). If 0 < x < 6 and area ABC is
20 cm2 , the value of x is
(A) 2 (B) 3 (C) 4 (D) 5 (E) 1
72 Geometry

10 1979 I.26 (9%)


The slope (gradient) of the straight line intersecting the graph of
y = 2x2 − 8x + 9 at (0, a) and (b, 1) is
(A) 9 (B) 2 (C) −2 (D) −4 (E) 4

11 1981 S.20 (26%)


Two straight lines intersect at 45◦ . If one line has a gradient of 2, what
two possible values can the gradient of the other line have?
1
(A) 3 or −2 (B) − 13 or 3 (C) 1
3 or −3
1
(D) 3 or 1 (E) ± 13

12 1984 S.30 (7%)


A cube of edge 6 cm is divided into 216 unit cubes by planes parallel to
the faces of the cube. A sphere of diameter 6 cm is inscribed in the large
cube so that faces of this cube are tangent to the sphere. The number of
complete unit cubes contained in the sphere is
(A) 48 (B) 56 (C) 60 (D) 64 (E) 40

CUBES
1 1980 J.9 (55%)
How many cubes with edges of length 2 cm can be made from a
rectangular block 6 cm long, 4 cm wide and 2 cm high?
(A) 24 (B) 12 (C) 6 (D) 8 (E) 48

2 1984 J.19 (5%), I.11 (6%), S.7 (16%)

P Q and QR are diagonals on two faces of a P .......


 ...... ....... .... 
.
cube, as shown. The angle P QR is
◦ ◦ ◦
 ...

.......
(A) 120 (B) 45 (C) 60 ..
..
.... Q
.
....
◦ ◦ ..
(D) 75 (E) 90 ..
.....
.....
..
.....
..
.....
..
.... 

..
.....
R
Cubes 73

3 1983 J.26 (4%), I.18 (7%)


How many planes of symmetry has a cube?
(A) 3 (B) 5 (C) 6 (D) 9 (E) 12

4 1982 J.15 (43%)


The least number of different colours needed to paint a cube so that no
two adjacent faces have the same colour is
(A) 2 (B) 6 (C) 4 (D) 3 (E) 5

5 1984 S.15 (10%)

The ratio of the surface area of the cube Q... R


..... 
.
P QRST U V W to the surface area of tetra-
P 
.
hedron U P RW is ...
..
S
√ ...
..
(A) 3 : 1 (B) 4 : 1 (C) 2 : 1 ...
√ ..
...
(D) 3 : 1 (E) 2 : 1 U ..
.
...
.. ....... ....... ....... ....... ....... .......
V
..
..... 

.
.....

T W

6 1982 I.16 (11%)

The diagram represents


√ a cube with diagonal P .....
..... ..... 
...
P Q equal to 12 units. The volume of the
cube, in cubic units, is  ... .....
..

...
... ..
.. ...
(A) 8 (B) 12 (C) 24 ... ..
√ √ ..
...
...
..
(D) 3 12 (E) 12 12 .. ...
.
..
........ ....... ....... ......... ....... .......
..

...
.
...
. .
.
. ...

.. ..
.....

7 1979 J.20 (29%), I.10 (46%), S7 (63%)


A cube has a side length of 4 cm. The cube is painted red all over and
then cut into 64 cubes of side 1 cm. How many of these cubes have
exactly one face painted red?
(A) 16 (B) 64 (C) 36 (D) 6 (E) 24
74 Geometry

8 1981 J.20 (19%), I.17 (22%), S.10 (27%)


A cube of side 4 cm is made up of individual 1 cm cubes. The number of
these 1 cm cubes which are face to face with exactly four other 1 cm
cubes is
(A) 0 (B) 8 (C) 24 (D) 28 (E) 16

9 1984 J.25 (8%)


Each of the faces of a cube is numbered with a different integer. Each
vertex is assigned a ‘vertex number’ which is the sum of the numbers on
the faces which intersect in that vertex and then the sum of the vertex
numbers is calculated. The highest number which must divide this sum,
for every possible numbering of the faces, is
(A) 3 (B) 4 (C) 6 (D) 8 (E) 12

10 1982 J.24 (32%)


......................
..............
The midpoints of the edges of a wooden cube are 
...................
.......
..............
.......

.......
.....................
....

joined as shown. The resulting triangular pyramids  .......


................
... ...
....
....... ....
.. ................... .....
. . . . .
.
.
. .
..
................................... ..
. . . . . . 
...................................................... .....
. . . . .
..... .................................................................................
..... ......................................... ...
whose edges are these lines and the original edges ..
....... ................................... .
...
. .............................
..... ........................ ..
...... ................... ..
.
of the cube are sawn off. The resulting solid has ...
.....
.........
.. ..........
.........
....
.
...
...
..... ........ ..
..... . ..... .... ....
(A) 14 faces and 24 edges .....
..... .....
.... . .. ... ..
... ..

..... ..... ......
..... ..... ...
..... .....
(B) 14 faces and 36 edges .....
..... ......
.......
....

(C) 16 faces and 24 edges
(D) 12 faces and 36 edges
(E) 16 faces and 36 edges

11 1979 J.29 (23%)


All the corners are cut neatly off a cube of volume 1000 cubic
centimetres, creating a new triangular face where each vertex used to be,
and each of these triangles is an equilateral triangle of side 1 cm. The
number of edges on the new solid is
(A) 24 (B) 12 (C) 16 (D) 36 (E) 30
Polygons 75

12 1981 J.30 (12%), I.30 (14%), S.30 (13%)


27 dots are arranged in a cube so that there is a dot at each corner, a
dot at the midpoint of each edge, a dot in the centre of each face and a
dot in the centre of the cube. How many sets of three dots lying in a
straight line can be found?
(A) 84 (B) 72 (C) 49 (D) 42 (E) 27

13 1983 J.30 (14%), I.30 (12%), S.30 (10%)


Some unit cubes are assembled to form a larger cube and then some of
the faces of this larger cube are painted. After the paint dries, the larger
cube is disassembled into the unit cubes, and it is found that 45 of these
have no paint on any of their faces. How many faces of the larger cube
were painted?
(A) 1 (B) 2 (C) 3 (D) 4 (E) 5

POLYGONS
1 1984 J.28 (6%), I.26 (3%), S.22 (4%)
A convex polygon is a polygon in which each interior angle is less than
180◦ . Which of the following numbers could not be the number of
diagonals of a convex polygon?
(A) 9 (B) 27 (C) 45 (D) 54 (E) 5

2 1984 I.22 (5%)

F GHIJKLM is a cube of side 1 metre. P , I... .....


U H
...... .. ....
Q, R, S, T , U are the midpoints of F I, F J, P .... ..
........
...
....... . ...
..... 
F .... ..... 
. . ..
.....
JK, KL, LH, HI respectively. The area of G
..
.....
. ..
the hexagon P QRST U , in square metres, is ...
...
.. .
...
T
....
√ √ ...
.. ..
..
3 3 3 3 3 Q ..... ...
.. .
...
(A) (B) (C) .....
M ..
.................. ....... ....... ....... ....... .......... L
4 4 √ 2 .. ..  .
..... ....... ......
....... .
1 3 ..
.... ..... ....... .......
.. S
(D) √ (E) J R K
4 3 8
76 Geometry

PYTHAGORAS’ THEOREM
1 1978 I.2 (53%)
If a boat sets out from a point A and sails south-east for 10 kilometres
then north-east for 24 kilometres then the distance it is from A, in
kilometres, is
(A) 34 (B) 17 (C) 25 (D) 26 (E) 30

2 1980 I.9 (24%)


The internal measurements of a rectangular box are 3 cm×4 cm×12 cm.
What is the length, in centimetres,of the longest rod which will fit in the
box?
√ √
(A) 19 (B) 19 (C) 13 (D) 12 (E) 160

3 1981 I.18 (31%), S.6 (63%)

A square box of side 5 cm is leaning against a P


 

vertical wall as shown with R 4 cm from the
  
wall. The height of P , in centimetres, above S 
the floor is 
√  
(A) 50 (B) 7 (C) 8  
√   Q
(D) 3 + 5 (E) 6
 5


R 4

4 1980 I.16 (30%)


A 25 m pole is propped against a vertical wall with its foot 20 m from
the wall. If its foot slips a further 4 m from the wall then the distance,
in metres, that the top of the pole will slip down the wall is

(A) 4 (B) 8 (C) 5 (D) 90 (E) 7
Quadrilaterals 77

5 1982 I.22 (11%), S.18 (36%)


A cube of side length 1m rests on a horizontal floor. An ant walking
along the floor bumps into the cube and begins to climb around its face
always walking in straight lines at a constant angle above the horizontal.
If it arrives at the top face of the cube at the point vertically above its
starting point after just one circuit, then the length, in metres, of its
path on the cube is
√ √ √ √
(A) 37 (B) 4 (C) 10 (D) 2 5 (E) 17

6 1980 I.28 (31%)

RST U is a rectangle. RS is 8 m. RU is 6m. R 8 S


RP and T Q are each perpendicular to SU .  
 Q 
Then P Q, in metres, is ....
...
......
6  . 
...... 
.

..
(A) 6.4 (B) 6 (C) 3.6
 .
P
(D) 2.8 (E) 1  
 
U T

7 1979 S.22 (13%)

E is an arbitrary interior point of a rectangle A... B


... ......
... ......
ABCD as shown. If the lengths of AE, BE, ...
...
.....
......
.....
CE and DE are a, b, c and x units respec-
...
...
...a b ......
....
...
... ......
... ......
... ......
tively, then x is given by ...
... ..........
........
.
....

...
...... ................
x ...... c
(A) x = a − b + c (B) x = a + b − c .
...
.......
...
E .........
.........
.........
...... .........
...... .........
......
(C) x = b + c − a (D) x2 = a2 − b2 + c2 D
....
C
(E) x2 = a2 + b2 − c2

QUADRILATERALS
1 1981 J.24 (18%), S.12 (60%)

When this 16 by 9 rectangle is cut in the man- .. 16


.......
ner shown, the pieces, when rearranged, form a  3
square of perimeter 5
9 
(A) 32 (B) 36 (C) 40  10

(D) 48 (E) 50 
78 Geometry

2 1979 S.25 (7%)

In the figure, ABCD is a trapezium, and A a cm B


M N is parallel to AB and DC. If M N  
bisects the area ABCD, the length of  
M N , in centimetres, is  
M N

a 2 + b2 a 2 + b2   
(A) (B) (C) (ab)  
2 a+b D b cm C
2ab a+b
(D) (E)
a+b 2

RATIO
1 1978 S.24 (33%)

In the figure, AB = AC and KL = LM . A


....
... ...
KB ..
... .....
The ratio is .
...
.
. ...
...
...
LC K ..
.
...
...
.
...
...
.... .............. ...
.
. ........
. ........ .....
(A) 1.5 (B) 2.0 (C) 2.5 ..
.
.. .
.
.......... ....
..
..
...
. L
.........
..........
... ..........
(D) 3.0 (E) 5.0 ... ... .........
...
. ... ........
........
... ... ........
... ...
B ...
M
C

2 1980 S.26 (25%)

P QRS is a parallelogram as shown. QT Q...... ..


R
... ...........
meets P R at V , P S at U and RS at T . If  .................. ......... ........
...........
.......... 
QU = 3 and QT = 6, then QV is  .......................V
...............
..........
..........

.. S
....
.. .....
...
. ..
.......... .....
.....
.....
(A) 1 (B) 1.5 (C) 1.8 P U .............. 

.....
.....
.....
(D) 2 (E) 2.5 .....
....
.....

T
Solid Geometry 79

3 1983 S.28 (9%)


S, T and U are points other than the vertices on the sides of the triangle
P QR dividing the sides QR, RP and P Q, respectively, in such a way
QS RT P U areaST U
that the four ratios , , and are all equal. This
QR RP P Q areaP QR
common ratio is
1 1 1 2 3
(A) (B) (C) (D) (E)
3 2 4 3 4
4 1984 S.26 (4%)

P QR is a triangle. Q is the midpoint of P


P S. The length of U R is 23 the length of 
P U . T is the point of intersection of the  
QT  U
lines QR and SU . The ratio equals   .....
.....
.....
.....
QR Q .
...
......
.
R
.....
.....
2 3 3  .
.....
.....
.....
.
T

.
(A) (B) (C) .....
.....
.
..
5 7 8 .....

4 5  .....
.
...
.
.....
.....
.

(D) (E)  .....


.....
.....
.....
9 11  ..
..
..
.....
S

SOLID GEOMETRY
1 1978 J.29 (22%)

The figure ABCDP QRS represents a cube. A ..... ..... B


..... ..
..... .... ........
The intersection of the two planes, one through .....
.....
.....
. ......
.....
..... .... .....
.....
the points A, B and R, the other through the .....
.....
D ....
.....
.....
..... . ... C
points B, C and P is the line ....
.

(A) CP (B) BS (C) P R .......... ....... ....... ....... ....... .......


.
Q
P .....
.....
.....
..
.....
..
.....
(D) P B (E) AQ .....
.....
.....
.....
..
.....
..... ..
....

S R
80 Geometry

2 1978 I.25 (30%)

The diagram represents a rectangular box in C


which the lengths of edges OA, OB and OC ..... 
are respectively 1, 2 and 3 units. In the same  ..
.. 
units the length of OD is O  ...
.
...
B
D
.
√ √ ....... ....... ....... ....... ....... ....

(A) 6 (B) 6 (C) 13 ..


..
.... 
√ .
.....
.....

(D) 14 (E) 14 ..
.... 
A

3 1978 S.29 (12%)

ABCD is a regular tetrahedron (i.e. a reg- A ...


.....
ular triangular pyramid) with all edges 2 cm .....
..... 
..... ...
...

M
.
...
....
in length. If L and M are the midpoints of .
... .
.....
.
......

..
..... ...
..... .
...... ...
BC and AD respectively, then the length of .....
D
.. .. ....
.............. ....... ....... ............. ....... ....... ....... ...................
LM , in centimetres, is B .........
......... . .... ...
.....
........... .......
....... ......
......... .......
√ √ L .......... .............
.......
(A) 2 (B) 3 (C) 2 C
1 √
(D) 1 (E) 5
2
4 1979 I.28 (13%)

The pyramid ABCDE has a square base E


..........
.... .. .....
ABCD and edges AE, BE, CE, DE of equal .
.
.... ... .....
.. . .... .........
. .. . .....
... ... .... .....
length. If AB is 20 cm long and the height (of ...
... ..
...
...
.....
.....
.....
.... .. ... .....
E above the base) is 10 cm, then the length of .
... . ...
. .......... ........... ....... ....... ................
.....

AE, in centimetres, is .
.
... .... ...
...
...
.... C
..
.. ...
.
.. ......
D ...
... ....
....
.....
√ √ √ ...
........
.
...
... ........
....
.

(A) 10 3 (B) 10 2 (C) 10 5 ..... ... .....


.... .........
....

(D) 15 (E) 16.5 A B


5 1980 J.18 (8%), I.8 (12%), S.4 (24%)
... ..
....... ......
...... .. ......
How many pairs of parallel edges are there in .....
...... .
..... .
.
. ....
......
.....
.
. .
the rectangular prism shown? ...
.
...
. ....... ....... ....... ....... ....... ....... ........
(A) 8 (B) 18 (C) 14 .
.....
......
......
....
......
. .....
...... ......
(D) 16 (E) 12
Solid Geometry 81

6 1980 J.29 (65%), I14 (75%), S.8 (83%)


Which one of the following figures can not be folded along the dotted
lines shown to form a cube?

(A) (B) (C)


. . . . . . . . . . . . . .
. . . . . .
. . . . . .
. . . . . .
. . . . . . . . . . . . . . . . . .
. .
. .
. .
. . . .

(D) (E)
. . . . . . . . . . . . . .
. . . . .
. . . . .
. . . . .
. . . . . . . . . . .

7 1980 S.22 (29%)

The diagram represents a circular cylinder of cir- ..........


.......................................................
cumference 6 cm and height 4 cm. Point P on the P .............. ......
..
.....
.
.........
...........................................................
top rim is diametrically opposite point Q on the
bottom rim. What is the shortest distance, in
centimetres, from P to Q along the surface of the
cylinder?
√ 6
(A) 52 (B) 4 + (C) 5 ....... ....
... ....... ....... ....
...

π .....
....
....
..
...
......
...........
........................................... ...
..
... ...... Q
36
(D) 7 (E) 16 + 2
π
82 Geometry

8 1983 J.7 (85%)

A piece of paper is cut out and labelled as shown


in the diagram. It is folded along the dotted Y
lines to make an open box. If the box is placed ...
..
....... ....... .......
..
..
...
on a table so that the top of the box is open, U ..
... V ...
. W
..
then the label on the bottom of the box is ....... ....... .......
...
.

(A) U (B) V (C) W X


(D) X (E) Y
9 1983 S.26 (8%)

Given the cube P QRST U V W as shown, the S R


plane which passes through P and the centres .... 
of the faces T U V W and U QRV intersects P  .... 
... Q
U V at X. .
..
..
UX ...
The ratio is .
XV W .
...
........ ....... ....... ....... ....... .......

V
.
....
3 5 5 ..
....
(A) 2 (B) (C) 3 (D) (E) ..
..... 
2 4 2 T U

10 1984 I.24 (7%), S.19 (12%)


Four spheres each of radius 10 cm lie on a horizontal table so that the
centres of the spheres form a square of side 20 cm. A fifth sphere of
radius 10 cm is placed on them so that it touches each of the spheres
without disturbing them. How many centimetres above the table is the
centre of the fifth sphere?
√ √ √ √
(A) 10 6 (B) 10(1 + 2) (C) 10(1 + 3) (D) 10(4 − 2) (E) 24

TRIANGLES
1 1983 J.5 (86%)

In the diagram, x equals x


 ◦


(A) 70 (B) 80 (C) 90  

(D) 40 (E) 50  ◦ 
 50 60◦ 
Triangles 83

2 1978 I.6 (22%)



The length of the shortest side of a 60 set square is 12 cm. The length
of the longest side, in centimetres, is
√ √ √
(A) 6 3 (B) 12 3 (C) 18 (D) 12 2 (E) 24

3 1979 J.14 (44%)

ABCD is a square of side 10 cm and QR is A P B


......
... ...
5 cm. The area of P QR, in square centime- ... ..
.. .....
.
... ....
tres, is ... ...
... ...
... ...
(A) 100 (B) 50 (C) 25 (D) 12 12 ..
.
..
. ...
...
...
... ...
..
. ...
(E) not determined by the given information .
..
.
..
...
...
... ...
..
. ...
..
. ...
... ...
...
... ...
... .
D Q R C

4 1982 J.19 (13%), I.13 (17%), S.8 (24%)


How many different triangles can be constructed with seven matchsticks,
in such a way that the perimeter of each triangle is the total length of
the seven matchsticks?
(A) 0 (B) 1 (C) 2 (D) 3 (E) 4

5 1979 J.19 (17%), I.9 (23%), S.6 (46%)

In the diagram AB, CD and EF are A... .


F
... ...
...
straight lines. The value of a + b − c, in ...
...
... .....
...
......
degrees, is .....
... ...
a
..... .....
...
. ...
. ...
...
(A) 120 (B) 150 (C) 180 .
...
...
...
...
...
.
.. ...
(D) 210 (E) none of these .
.
..
.
c ...
... b
D ..
.
..
.
.
.. ...
...
C
.
... ...
...
...
. ...
.
... ..

E B

6 1981 I.21 (25%), S.14 (29%)


Two vertical poles, 20 m and 80 m high, stand apart on a horizontal
plane. The height, in metres, of the point of intersection of the lines
joining the top of each pole to the foot of the other is

(A) 18 (B) 50 (C) 16 (D) 15 (E) 11 2
84 Geometry

7 1984 J.27 (5%), I.27 (6%), S.24 (7%)

Triangle P V Y is equilateral, with sides of P



length 3 cm. Q, R, S, U , W , X divide 
 
the sides of this original triangle into unit Q • • R
lengths, thus P Q, QS and SV each have  
length 1 cm. T is the common point of  
S • • •U

intersection of the lines QX, SU , RW . T
QX  P Y , RW  P V and SU  V Y . Of  
the 10 points P, . . . , Y , how many sets of • • • •
V W X Y
three are the vertices of equilateral trian-
gles?
(A) 10 (B) 13 (C) 12 (D) 9 (E) 15

8 1981 S.29 (6%)

Points S and T trisect the hypotenuse of the P


right-angled triangle P QR as shown. The 
length of RS is 7 cm, and of RT is 9 cm. The 
length of ST , in centimetres, is .... T
... 
...
√ √

.. ..
(A) 15 (B) 6 (C) 26 ...
...
√ .
...
..
S ..
.......
(D) 32 (E) 5 ...
... ....... 
.......
.. .
........

. .
..
... .....
.......
... .........
............
........ 
R Q

9 1982 S.28 (6%)


◦ Q
The angles P , Q and R are each 45 . The
segments QS and RS extended are per- 
pendicular to P Q and P R respectively. If ..
.........

P S = 20, the distance from Q to R is   S


 
√ 20  
(A) 20 (B) 20 2 (C) √  ........

2 P R

(D) 10 3 (E) 10
Trigonometry 85

TRIGONOMETRY
1 1978 S.5 (39%)
2rs
If tan A = 2 , where A is acute and r > s > 0, then cos A is equal
r − s2
to

r (r2 − s2 ) rs r r 2 − s2
(A) (B) (C) 2 (D) (E)
s 2r r + s2 r−s r 2 + s2
2 1983 S.12 (33%)
From a horizontal distance of 50 m, the angles of elevation of the top
and bottom of a vertical cliff face are 45◦ and 30◦ respectively. The
height of the cliff face, in metres, is
   
50 50 50 1 1
(A) √ (B) √ (C) √ (D) 50 1 − √ (E) 50 1 − √
3 2 2 3 2 3
3 1984 S.13 (10%)

In the diagram, angle P RQ is a right angle, P


and P S and SR are both of length 1 cm, QR 
is of length 2 cm. The value of tan θ is  1

1 1 1◦ 
(A) (B) √ (C) tan 22 .......
..
S
2 5 2  .
.....
.........
.
.......
.......

1 1 
θ .......
.......
.......
.......
1
(D) (E)  ..
.......
.
...
.........
.......
3 6  .......
.......
........
.
Q 2 R

VOLUME
1 1983 J.13 (19%)
A child glues together 42 cubes with 1 cm edges to form a solid
rectangular-faced brick. If the perimeter of the base is 18 cm, the height
of the brick, in centimetres, is
7
(A) 3 (B) 6 (C) 2 (D) 7 (E)
3
86 Geometry

2 1982 I.18 (10%), S.11 (41%)


Tennis balls are often supplied in a cylindrical container which contains
3 balls in a neat fit (the centres of the balls are collinear). What
proportion of the volume of the container is occupied by the balls?
(The volume of a sphere with radius r is 43 πr3 .)
2π 2 π 3
(A) (B) (C) (D) (E) none of these
3 3 4 4
3 1981 S.18 (27%)
2
The circular base of a cylindrical water tank has area 1 m . A solid cube
with edges of length 20 cm is dropped beneath the surface of the water.
The resulting increase in the height of the water level, in centimetres, is
π 0.8
(A) (B) (C) 0.8 (D) 8 (E) 80
0.8 π
OTHER TOPICS

COUNTING TECHNIQUES
1 1978 J.21 (23%)
In a football competition there are 9 teams. If each team plays each
other team twice then the total number of matches played is
(A) 18 (B) 144 (C) 36 (D) 72 (E) 81

2 1979 J.22 (47%), I.21 (67%)


In a tennis tournament, only the winner of each match plays another
match, and this process continues until the tournament winner is
decided. If there are 128 players, how many matches must be played in
order to determine the tournament winner?
(A) 129 (B) 256 (C) 128 (D) 64 (E) 127

3 1979 J.23 (6%), I.24 (6%)


The length, in centimetres, of each side of a scalene triangle (i.e. a
triangle with no two sides equal) is an integer. If the perimeter of the
triangle is 13 cm, then the number of such triangles (no two congruent) is
(A) 1 (B) 2 (C) 3 (D) 4 (E) more than 4

4 1979 S.26 (34%)


852 digits are used to number the pages of a book consecutively from
page 1. The number of pages in the book is
(A) 320 (B) 247 (C) 316 (D) 852 (E) 284
88 Other Topics

5 1980 J.30 (30%), I.29 (39%), S.27 (54%)

If only downward motion along lines is allowed, what is P


the total number of paths from point P to point Q in 
the figure shown? 

(A) 4 (B) 6 (C) 10 

(D) 20 (E) 40 
Q

6 1980 I.12 (21%)


Twenty people at a gathering all shook hands with each other exactly
once. The number of handshakes which took place was
(A) 190 (B) 200 (C) 19 (D) 400 (E) 380

7 1981 J.13 (27%), I.8 (37%)

A square 4 cm × 4 cm is divided into 16 squares of


side 1 cm as shown. The total number of squares
(of any size) in the diagram is
(A) 20 (B) 17 (C) 25
(D) 29 (E) 30

8 1981 J.17 (8%), I.13 (17%)


A shopkeeper sells house numbers, and has left a large supply of the
numerals 4, 7 and 8, but has sold out of all other numerals. How many
three-digit house numbers could be made up from these?
(A) 6 (B) 18 (C) 24 (D) 26 (E) 27

9 1981 J.18 (35%)

A domino set contains all number pairs from


double-zero to double-six with each number pair • • •
occurring exactly once. For example, the domino • • •
shown is two-four and also four-two. How many
dominoes in a set?
(A) 36 (B) 31 (C) 42 (D) 28 (E) 21
Counting Techniques 89

10 1981 J.23 (47%), I.19 (67%)


At a party 28 handshakes were exchanged. Each person shook hands
exactly once with each of the others. The number of people present was
(A) 7 (B) 8 (C) 27 (D) 14 (E) 28

11 1981 J.27 (7%), I.28 (9%), S.27 (11%)


In how many different ways can a careless office boy place four letters in
four envelopes so that no one gets the right letter?
(A) 4 (B) 9 (C) 12 (D) 6 (E) 24

12 1981 J.29 (5%), I.29 (6%), S.28 (7%)


Three judges for a talent quest have to vote publicly on three performers
A, B and C, listing their order of preference. In how many ways can the
judges vote so that any two of them agree in their order of preference,
while the third differs?
(A) 45 (B) 90 (C) 30 (D) 120 (E) 24

13 1983 J.16 (26%)


I have a 1c, a 2c, a 5c and a 10c coin. What is the total number of
non-zero amounts of money which can be formed from some or all of
these coins?
(A) 4 (B) 15 (C) 18 (D) 24 (E) 16

14 1983 J.24 (7%)


In the school band five children each own their own trumpet. In how
many different ways can exactly three of the five children take home the
wrong trumpet, while the other two take home the right trumpet?
(A) 5 (B) 6 (C) 10 (D) 20 (E) 30

15 1983 J.28 (5%)


Six problems (numbered 1 to 6) are set for a mathematics test. A
student can score 0, 1, 2 or 3 marks for each problem. The number of
ways a total of 15 marks for the 6 problems can be scored is
(A) 6 (B) 15 (C) 56 (D) 36 (E) 41
90 Other Topics

16 1983 S.29 (9%)


I have four pairs of socks to be hung out side by side on a straight
clothes line. The socks in each pair are identical but the pairs
themselves have different colours. How many different colour patterns
can be made if no sock is allowed to be next to its mate?
(A) 792 (B) 630 (C) 2520 (D) 864 (E) 720

17 1984 J.13 (13%), I.7 (18%)


Two couples, sitting together on a park bench, pose for a photograph. If
neither couple wishes to be separated, the number of different possible
seating arrangements is
(A) 1 (B) 2 (C) 3 (D) 4 (E) 8

18 1984 J.30 (8%), I.30 (8%), S.29 (5%)


Dots are arranged in a rectangular grid containing 4 rows and n
columns. Consider different ways of colouring the dots, colouring each
dot either yellow or green. Call a colouring of the diagram ‘good’ if no
four dots of the same colour form a rectangle (or square) with horizontal
and vertical sides. The maximum value of n for which the diagram
permits a ‘good’ colouring is
(A) 7 (B) 4 (C) 5 (D) 6 (E) 8

LOGIC
1 1978 J.25 (55%), I.19 (59%), S.10 (67%)
If, in the Republic of Utopia, the statement ‘all citizens have two legs’ is
false, which of the following statements must be true?
I All citizens have more than two legs.
II Not all citizens have two legs.
III No citizen has two legs.
IV There are some citizens with more than two legs.
(A) II only (B) IV only (C) I and III only
(D) II and IV only (E) I, II and IV only
Probability 91

2 1984 J.29 (9%), I.28 (13%), S.23 (20%)


Albert, Bernard, Charles, Daniel and Ellie play a game in which each is
a frog or kangaroo. Frogs’ statements are always false while kangaroos’
statements are always true.
Albert says that Bernard is a kangaroo.
Charles says that Daniel is a frog.
Ellie says that Albert is not a frog.
Bernard says that Charles is not a kangaroo.
Daniel says that Ellie and Albert are different kinds of animals.
How many frogs are there?
(A) 1 (B) 2 (C) 3 (D) 4 (E) 5

PROBABILITY
1 1984 S.25 (5%)
On a tiny, remote island where the death sentence still exists, a man can
be granted mercy after receiving a death sentence in the following way:
He is given 18 white balls and 6 black balls. He must divide them
among three boxes with at least one ball in each box. Then, blindfolded,
he must choose a box at random, and then a ball from within this box.
He receives mercy only if the chosen ball is white. The probability of the
man receiving mercy (provided he has distributed the balls in the most
favourable manner) is
11 3 10 8 1
(A) (B) (C) (D) (E)
12 4 11 11 4
2 1982 I.29 (3%), S.29 (3%)
A deck of 16 cards contains the four aces, four kings, four queens and
four jacks. The 16 cards are thoroughly shuffled and my opponent (who
always tells the truth) draws two cards simultaneously and at random
from the deck. He says ‘I hold at least one ace’. The chance that he
holds two aces in his hand is
1 3 1 2 1
(A) (B) (C) (D) (E)
5 16 6 15 9
92 Other Topics

SETS
1 1978 I.16 (7%)
In a survey of newspaper reading habits in a city with 3 daily
newspapers, A, B and C, it was found
34% read newspaper A
46% read newspaper B
52% read newspaper C
17% read newspapers A and B
19% read newspapers A and C
20% read newspapers B and C
15% read only newspaper A
The percentage who read newspaper C only is
(A) 52 (B) 30 (C) 32 (D) 20 (E) 10

TRIGONOMETRIC IDENTITIES
1 1979 S.9 (23%)
sin y ◦ + sin(x − y)◦ = sin x◦ for all y provided x is
(A) 60 (B) 90 (C) 180 (D) 270 (E) 360

2 1980 S.16 (23%)


2
(cot θ + tan θ) equals
(A) cosec2 θ sec2 θ (B) cot2 θ + tan2 θ − 2 (C) sec2 θ − cosec2 θ
(D) cot2 θ − tan2 θ + 2 (E) cot2 θ + tan2 θ

3 1982 S.23 (10%)


Find the degree measure of the least positive angle x which satisfies

sin 6x + cos 4x = 0

(A) 27 (B) 63 (C) 9 (D) 45 (E) 18

4 1984 S.17 (10%)


◦ ◦ ◦
Given cos 12 = sin 18 + sin θ the least positive value of θ is
(A) 42 (B) 35 (C) 32 (D) 6 (E) 30
Miscellaneous 93

MISCELLANEOUS
1 1983 J.17 (58%), I.7 (75%)

Fifteen tiles are arranged as shown. An P ............................. ............................. .............................


...................................... ...................................... ......................................
ant walks along the edges of the tiles, al- ......................................
......................................
......................................
......................................
......................................
......................................
...................................... ...................................... ............................
.......................................
ways keeping a black tile on its left. If the .............................
...................................... ......................................
...................................... ......................................
tiles are square with 10 cm sides, what is ......................................
............................
......................................
............................
...........................................................................................................
the shortest distance, in centimetres, that ......................................
......................................
......................................
......................................
......................................
......................................
...................................... ...................................... ......................................
the ant would walk in going from P to Q ...................................... ...................................... ......................................
Q
according to the given rules?
(A) 80 (B) 180 (C) 120 (D) 320 (E) 100

2 1980 S.11 (50%)


A palindromic number is a whole number that reads the same both ways
(for example, 28482). How many palindromic numbers are there
between 10 and 1000?
(A) 9 (B) 109 (C) 90 (D) 99 (E) 100

3 1982 J.27 (8%), I.19 (10%)


In 1970 I began collecting calendars and I have done so every year since.
I will cease collecting when every subsequent year can be served by one,
at least, of the calendars I have already collected. The last year in which
I must collect the calendars is
(A) 1983 (B) 1984 (C) 1997 (D) 1996 (E) 2000

4 1980 J.24 (54%)


The height of a tree is measured annually. It was x metres high when
planted, and has grown by exactly y metres between each annual
measurement. At the latest annual measurement it was 23 metres high.
Which of the following is possible?
(A) x = 8, y = 2 (B) x = 5, y = 7 (C) x = 6, y = 5
(D) x = 5, y = 3 (E) x = 2, y = 6
94 Other Topics

5 1979 J.28 (5%), I.13 (9%)


At certain times of the day the two hands of a clock point in the same
direction (e.g. midday). The number of times this happens between
3 am Tuesday and 3 am the next day is
(A) 24 (B) 12 (C) 2 (D) 23 (E) 22

6 1980 J.16 (67%)


A clock is set correctly at 1 pm. It loses 3 minutes every hour. What
will the clock read when it is 10 am the next day?
(A) 9:03 (B) 10:00 (C) 11:03 (D) 8:57 (E) 11:06

7 1983 J.23 (13%)


One electronic device makes a ‘bip’ every 60 seconds. Another electronic
device makes a ‘bip’ each 62 seconds. They both ‘bip’ at 10:00 am. The
time when they next make a ‘bip’ together is
(A) 10:30 am (B) 10:31 am (C) 10:59 am (D) 11:00 am (E) 11:02 am

8 1982 J.30 (9%), I.28 (9%), S.27 (10%)


I have two watches with a 12 hour cycle. One gains a minute each day
and the other loses 1 12 minutes per day. If I set them both on the correct
time, how long will it be before they next together tell the correct time?
(A) 288 days (B) 2880 days (C) 480 days
(D) 720 days (E) 1440 days
Miscellaneous 95

9 1980 I.30 (12%), S.29 (13%)


During a particular morning, a light signal goes on at precisely 9 am.
Thereafter it goes off and on at equal intervals, each lasting a whole
number of minutes. Later that morning it is observed the light is off at
9:09, on at 9:17 and on at 9:58. The light will be on that morning at
both
(A) 10:30 and 11:21 (B) 10:14 and 11:00 (C) 10:23 and 11:01
(D) 10:25 and 11:33 (E) 10:40 and 11:46

10 1979 S.24 (55%)

(i) Bill’s watch is 10 minutes fast but he thinks it is 5 minutes slow.

(ii) Joanna’s watch is 5 minutes slow but she thinks it is 5 minutes fast.
(iii) Harriet’s watch is 5 minutes fast but she thinks it is 10 minutes
slow.

(iv) John’s watch is 10 minutes slow but he thinks it is 10 minutes fast.

Using their watches, each of them leaves work in what each believes is
time to catch the 6 pm train. Who misses the train?
(A) Bill and John (B) Bill and Harriet (C) John and Joanna
(D) Harriet and Joanna (E) All of them

11 1982 J.29 (11%), I.26 (13%), S.20 (12%)


During a series of challenge races between Gregory and Michael over two
lengths of a swimming pool, it was observed that when Michael swam
faster than Gregory over the second lap Gregory had led at the end of
the first lap. Whenever Michael had led at the end of the first lap,
Gregory swam faster than Michael in the second lap. There were nine
races in which Michael swam faster than Gregory on at least one lap.
Gregory swam a faster time than Michael on 7 first laps and 6 second
laps. The minimum number of challenge races was
(A) 9 (B) 10 (C) 11 (D) 12 (E) 13
96 Other Topics

12 1981 J.28 (18%), I.26 (21%), S.24 (33%)


In a hockey tournament, each team played each other once. The final
league table was

Win Draws Losses Points


Falcons 1 2 0 4
Condors 1 1 1 3
Eagles 1 1 1 3
Hawks 1 0 2 2

If Hawks’ only win was against Condors, then

(A) Eagles defeated Condors, but lost to Hawks


(B) Falcons won against either Condors or Eagles
(C) In matches against Hawks, Condors were more successful than
Eagles
(D) In matches against Eagles, Falcons were more successful than
Condors
(E) Eagles were undefeated, except against Condors

13 1983 I.27 (6%)


American presidential elections are held on the Tuesday after the first
Monday in November in years whose number is divisible by four. The
Melbourne Cup is run every year on the first Tuesday in November. In
1983 the Melbourne Cup will be run on 1 November. How many times
this century (i.e. the years 1901 to 2000 inclusive) will the dates for the
two events coincide?
(A) 4 (B) 7 (C) 25 (D) 22 (E) 21

14 1981 J.25 (17%), I.24(18%), S.23 (19%)


The first of January 1981 fell on a Thursday. The first day of the
twentieth century (1 January 1901) fell on
(A) Tuesday (B) Wednesday (C) Thursday
(D) Friday (E) Saturday
SOLUTIONS
ARITHMETIC SOLUTIONS

APPROXIMATION
1 1978 J.10 (69%), I.3 (87%)
1.96 × 3.142 ≈ 2 × 3 = 6, hence (B).

2 1979 J.7 (53%), I.5 (79%), S.3 (86%)


2.7 × 32 3 × 30
≈ = 6, hence (B).
14.7 15
3 1980 J.6 (65%), I.3 (74%), S.2 (88%)
49.5 ÷ 0.5 ≈ 50 ÷ 0.5 = 100 ÷ 1 = 100, hence (D).

4 1983 I.8 (25%)


Sandwiching the given number between two perfect squares,
1600 1983 2500
< < .
10 000 10 000 10 000
Taking square roots,

40 1983 50
< < ,
100 10 000 100
i.e. 
1983
0.4 < < 0.5, hence (E).
10 000

5 1984 J.7 (41%)


600 6000
601 ÷ 0.305 ≈ = = 2000, hence (D).
0.3 3

DECIMALS
1 1978 J.1 (93%)
51.7 − 42.8 = 8.9,
hence (E).
100 Arithmetic Solutions

2 1978 J.2 (68%)


0.40 × 6.38 = 2.552,
hence (C).

3 1979 J.1 (96%), I.1 (97%)


36.3 − 17.5 = 18.8,
hence (A).

4 1979 J.5 (50%), I.3 (73%), S.1 (90%)


Alternative 1
(0.4)2 − (0.1)2 = 0.16 − 0.01 = 0.15,
hence (C).
Alternative 2
(0.4)2 − (0.1)2 = (0.4 − 0.1)(0.4 + 0.1) = (0.3)(0.5) = 0.15,
hence (C).

5 1979 J.12 (60%)


The difference between Grumpy’s and Dopey’s heights equals
1.27 − 1.11 = 0.16 metres. Therefore Happy’s height equals Dopey’s
height − 0.16 m, i.e. 1.11 − 0.16 = 0.95 metres,
hence (B).

6 1979 S.4 (77%)


0.1 1 1 10 0.3
p= = = 0.333 . . .. q = = = 3.333 . . .. r = = 0.3.
0.3 3 0.3 3 1
Therefore q > p and p > r,
hence (C).

7 1980 J.1 (95%)


27.3 − 16.4 = 10.9,
hence (D).

8 1980 J.5 (63%)


17 1.7 0.17
= = = 0.17,
100 10 1
hence (B).
Decimals 101

9 1981 J.2 (84%), I.1 (90%)


1.1 − 0.64 = 0.46,
hence (D).

10 1981 J.12 (39%)


2
(0.3) × 0.7 = 0.09 × 0.7 = 0.063,
hence (A).

11 1982 J.1 (99%), I.1 (99%)


2.5 + 2.1 = 4.6,
hence (A).

12 1982 J.6 (72%), I.3 (84%)


Between 25 and 26 the divisions on the scale are 0.1 apart. The reading
is 25.3,
hence (C).

13 1982 I.10 (34%), S.3 (51%)


3 2
(0.3) − (0.2) = 0.027 − 0.04 = −0.013,
hence (D).

14 1983 J.1 (95%)


1.2 + 3.4 = 4.6,
hence (E).

15 1984 J.1 (93%)


2.3 + 4.8 = 7.1,
hence (C).

16 1984 J.3 (45%)


The smallest number is 0.289 while the largest is 0.9. The sum is 1.189,
hence (A).

17 1984 J.4 (47%), S.1 (94%)


2
(0.2) = 0.04,
hence (A).
102 Arithmetic Solutions

18 1984 I.1 (94%)


7.3 − 4.9 = 2.4,
hence (C).

19 1984 J.10 (49%), I.3 (73%), S.2 (91%)


To correct the error the student must subtract 35 095 and add 35.95, i.e.
subtract (35 095 − 35.95) i.e. subtract 35 059.05,
hence (B).

DIRECTED NUMBERS
1 1978 J.6 (49%)
(−10) − (−14) = −10 + 14 = 4,
hence (C).

2 1981 J.7 (52%)


−6 + 4 − (−3) = −2 + 3 = 1,
hence (A).

EXPONENTS
1 1979 S.29 (4%)
Alternative 1
We are given that
     
P = (1 + 4) 1 + 42 1 + 44 1 + 48 1 + 416 1 + 432 .

Therefore
   
(1 − 4)P = (1 − 4)(1 + 4) 1 + 42 1 + 44 1 + 48
  
1 + 416 1 + 432
      
= 1 − 42 1 + 42 1 + 44 1 + 48 1 + 416 1 + 432
     
= 1 − 44 1 + 44 1 + 48 1 + 416 1 + 432
...
 
−3P = 1 − 464 .
Exponents 103

Therefore
464 − 1 2128 − 1
P = = ,
3 3

hence (C).
Alternative 2
If P is expanded in terms of its powers of 4, it can be seen that every
power of 4 up to 463 will appear precisely once, i.e.

P = 1 + 4 + 42 + 43 + · · · + 463 .

Thus P is a geometric progression with first term 1 and common ratio 4,


and with 64 terms. Therefore
 
1 1 − 464 464 − 1 2128 − 1
P = = = ,
1−4 3 3

hence (C).

2 1981 S.9 (72%)


Alternative 1
 4  2 2
58 = 52 = 254 = 252 = (625) = 390 625. This has 6 digits,
hence (C).
Alternative 2
Several different approximations are possible, such as

58 = (625)2 ≈ (600)2 ≈ 360 000

or
108 100 000 000 100 000 000
58 = 8
= ≈ = 400 000.
2 256 250
Within the limits of the approximation these show that 58 has 6 digits,
hence (C).

3 1982 I.6 (28%), S.1 (74%)


 3
93 × 32 = 32 × 32 = 36 × 32 = 38 ,
hence (D).
104 Arithmetic Solutions

4 1983 J.22 (14%)


Alternative 1
Note that 72 = 49, 73 = 343 and 74 = 2401. Then any power of 74 will
also end in . . . 01. Now
 495
71983 = 71980 × 73 = 74 × 343 = (. . . 01) × 343 = (. . . 43),
hence (C).
Alternative 2
 2
Since 74 = 72 = (49)2 = (50 − 1)2 = 2500 − 100 + 1 = 2401, and since
2401 ≡ 1 (mod 100), we have
 495
71983 = 71980 × 73 = 74 × 343 ≡ (1)(343) = 43 (mod 100),
hence (C).

5 1983 S.7 (64%)


15 14 14 14
1.236 × 10 − 5.23 × 10 = 12.36 × 10 − 5.23 × 10 =
(12.36 − 5.23) × 1014 = 7.13 × 1014 ,
hence (A).

6 1983 S.13 (12%)


100
 25
Note that x = 2 = 24 = (16)25 , similarly y = (27)25 and
z = (25) . Since 16 < 25 < 27, it follows that 1625 < 2525 < 2725 , i.e.
25

x < z < y,
hence (B).

7 1984 I.16 (9%)


10025 is 1 followed by 50 zeros. 10025 − 25 is forty-eight 9s followed by
75 (i.e.999 . . . 9975). The sum of the digits is
48 × 9 + 7 + 5 = 432 + 12 = 444,
hence (B).

FRACTIONS
1 1978 J.5 (85%)
3 1 6+5 1 1
4 +2 =6+ =6+1 =7 ,
5 2 10 10 10
hence (C).
Fractions 105

2 1978 S.4 (62%)


3
At each cut 4 of the cake remains. So after three cuts the proportion
remaining is
3 3 3 27
× × = ,
4 4 4 64

hence (B).

3 1979 J.3 (86%), I.2 (88%)


2 1 4 3 1
2 −1 =2 −1 =1 ,
3 2 6 6 6
hence (B).

4 1979 J.17 (68%), I.7 (83%)


1 10 10 100 3 9
p= = ;q= = ;r= = . Therefore q > p and p > r,
3 30 3 30 10 30
hence (C).

5 1979 J.21 (51%)


To find the smallest fraction note that P , R, S are each less than 1.
16 1 1 1 1 5
Also note that S = > while P < and R < . Now P = =
31 2 2 2 3 15
1 3
and R = = , so R is the smallest. To find the largest, note
5 15
17 7 1 9 2 1
T = =1 > 1 and Q = = 1 < 1 . Therefore T and R are the
10 10 2 7 7 2
required pair,
hence (E).

6 1979 S.11 (44%)


The lowest common denominator of the given fractions is 42 and
6 36 5 15 10 20
= , = , = .
7 42 14 42 21 42
The least common multiple of the numerators of these equivalent
fractions is 22 × 32 × 5 = 180 (since 36 = 22 × 32 , 15 = 3 × 5 and
20 = 22 × 5). Thus the least common (integral) multiple of the given
180 30
fractions is = ,
42 7
hence (D).
106 Arithmetic Solutions

7 1980 J.2 (80%), I.1 (87%), S.1 (93%)


2 5 4 5 9 1
1 + =1 + =1 =2 ,
3 6 6 6 6 2
hence (A).

8 1981 J.1 (87%)


2
× 12 = 8,
3
hence (B).

9 1981 J.3 (87%)


1 3 8 9 17
+ = + = ,
3 8 24 24 24
hence (D).

10 1981 J.11 (45%), S.1 (93%)


3 7
+
8 8 = 10 × 5 = 50 = 25 ,
4 8 4 32 16
5
hence (E).

11 1981 J.14 (71%), I.7 (83%)


2 1 15 6 5 4
Amount remaining = 1 − − = − − = ,
5 3 15 15 15 15
hence (C).

12 1981 S.22 (45%)

37 11
= 2+
13 13
1
= 2+
13
11
1
= 2+
2
1+
11
Fractions 107

1
= 2+
1
1+
11
2
1
= 2+ .
1
1+
1
5+
2
Comparing with the given fraction (x, y, z) = (1, 5, 2),
hence (B).
Note: An expression such as the above is called a ‘continued fraction’.
All rational numbers can be expressed as continued fractions. Irrational
numbers cannot be expressed as continued fractions but many can be
expressed very neatly as ‘infinite continued fractions’. For example,
√ 1
2=1+
1
2+
1
2+
2 + ···
An interesting description of this can be found in What is Mathematics,
pp 301–303, R Courant and H Robbins, O.U.P., 1941.

13 1982 J.2 (89%)


2 1 8 3 11
+ = + = ,
3 4 12 12 12
hence (C).

14 1982 J.7 (41%), I.7 (62%)


Alternative 1
1 5 1 3
= and = .
3 15 5 15
4
The number halfway between is ,
15
hence (D).
Alternative 2
The
 midpoint
 of 
two 
numbers is the same as their average. Here
1 1 1 1 8 4
+ = = ,
2 3 5 2 15 15
hence (D).
108 Arithmetic Solutions

15 1982 I.4 (90%)


 
3 2 1 3 2 9 2 11
+ × = + = + = ,
4 3 4 4 12 12 12 12
hence (B).

16 1982 S.15 (47%)


Alternative 1

If x = 1.451
then 10x = 14.5151 . . . (1)
and 1000x = 1451.5151 . . . (2)
(2) − (1): 990x = 1437

so
1437 479
1.451 = = ,
990 330
hence (D).
Alternative 2
The repeating part of the decimal may be expressed as an infinite
51 1
geometric progression with first term and common ratio , and
1000 100
its limiting sum found:
 
51 51
1.451 = 1.4 + 0.051 = 1.4 + + + ···
1000 100 000
51
14 14 51
= + 1000 = +
10 1 10 100 − 10
1−
100
14 51 14 17 14 × 33 + 17 479
= + = + = = ,
10 990 10 330 330 330
hence (D).

17 1983 J.3 (90%)


Alternative 1
1 1 10 5 40 − 15 25 1
3 −1 = − = = =2 ,
3 4 3 4 12 12 12
hence (B).
Fractions 109

Alternative 2  
1 1 1 1 1
3 −1 =2+ − =2 ,
3 4 3 4 12
hence (B).

18 1983 I.3 (53%)


3 7 9 + 49
+
7 3 = 21 = 58 × 42 = 4,
29 29 21 29
42 42
hence (B).

19 1983 I.6 (23%), S.2 (62%)


1 xy − 1
x−
y y x
= = ,
1 xy − 1 y
y−
x x
hence (A).

20 1984 J.2 (75%)


1
2 ÷ = 2 × 3 = 6,
3
hence (D).

21 1984 J.12 (26%)


Although the value of the fraction is increased, it is still less than 1
because the numerator is still less than the denominator,
hence (D).

22 1984 J.21 (20%)


1 97 × 95 1 97 × (95 − 96) 1 + (97)(−1) 96
+ − 97 = + = =− = −1,
96 96 96 96 96 96
hence (E).

23 1984 I.5 (55%)


    
1 1 1 1 1 2 3 4 1
1− 1− 1− 1− = × × × = ,
2 3 4 5 2 3 4 5 5
hence (D).
110 Arithmetic Solutions

MEAN
1 1981 J.19 (33%), I.15 (48%)
As the average reading is 16, the total of the three is 3 × 16 = 48. The
two maximum possible readings are 20, so the remaining and smallest
possible reading is 48 − 20 − 20 = 8 volts,
hence (A).

2 1982 J.14 (22%)


The average weight of the nine children is
70 × 5 + 61 × 4 350 + 244 594
= = = 66 kg,
9 9 9

hence (B).

3 1983 J.4 (55%), I.4(68%)


Alternative 1
By inspection the numbers are 7, 9 and 11,
hence (D).
Alternative 2
Let the odd numbers be n − 2, n and n + 2. Then
(n − 2) + n + (n + 2) = 27, i.e. 3n = 27, i.e. n = 9, and the smallest
number, n − 2, has the value 7,
hence (D).

4 1983 J.8 (36%), I.5 (54%)


The first six numbers total 6 × 4 = 24. The first seven numbers total
7 × 5 = 35. So the seventh number is 35 − 24 = 11,
hence (D).
Metric System 111

5 1983 I.24 (23%)


105
The average of the n integers must be .
n
n Average Solution
3 35 34, 35, 36 exists
4 26 14 This would need to be the average of
middle two integers, so not possible
5 21 19, 20, 21, 22, 23 exists
6 17 12 15, 16, 17, 18, 19, 20 exists
7 15 12, 13, 14, 15, 16, 17, 18 exists

hence (B).

6 1984 J.23 (16%), I.19 (24%)


Suppose Ken had played n games. Therefore
177n + 199
= 178.
n+1
Thus 177n + 199 = 178n + 178, or n = 21. If x is his next score then
178 × 22 + x
= 179.
23
So

x = 23 × 179 − 22 × 178
= 22 × 179 + 179 − 22 × 178
= 22 × 1 + 179
= 201,

hence (E).

METRIC SYSTEM
1 1978 J.12 (20%)
150 km = 150 000 000 mm. Therefore 300 mm represents 150 000 000 mm.
150 000 000
Therefore 1 mm represents mm = 500 000 mm,
300
hence (A).
112 Arithmetic Solutions

2 1982 J.10 (84%)


The required number of milligrams is 35 × 4 = 140,
hence (A).

MONEY
1 1983 J.20 (55%)
The amount owed by people without money is (4 × $21) = $84. Sharing
this among the remaining 14 people, each must pay $84/14 = $6.00,
hence (D).

2 1984 J.6 (80%)


The cost of 7 pens at $1.32 is $9.24. Therefore the change from a $20
note should be $10.76,
hence (C).

3 1984 I.12 (62%)


Suppose my weekly pocket money is $x. Then
2
x + x + 2x = 6.6,
3
i.e.
11 66
x= .
3 10
This gives
66 3
x= × = 1.8,
10 11

hence (D).

ORDER OF OPERATIONS
1 1980 J.3 (59%)
2 + 3(8 − 4) = 2 + (3 × 4) = 14,
hence (E).
Percentages 113

2 1983 J.2 (98%)


2 × (8 − 3) = 2 × 5 = 10,
hence (E).

PERCENTAGES
1 1978 J.3 (70%)
5
5% of 1200 equals × 1200 = 60,
100
hence (A).

2 1978 I.24 (6%), S.16 (12%)


2
Each quarter the cost of living increases by a factor of 1 + = 1.02.
100
Therefore the percentage increase in one year is
(1.024 − 1) × 100 = (1.08243216 − 1) × 100 ≈ 8.2,
hence (D).
Note: (1.02)4 can be either found by long multiplication or using a
binomial expansion:

(1 + 0.2)4 = 1 + 4(0.02) + 6(0.004) + 4(0.000008) + (0.02)4


= 1 + 0.08 + 0.0024 + · · ·
≈ 1.082

(since third decimal place accuracy, only, is required)

3 1979 J.2 (79%)


 
3 3
= × 100 % = 37.5%,
8 8
hence (E).

4 1980 J.7 (76%)


24 6
24% = = ,
100 25
hence (D).
114 Arithmetic Solutions

5 1980 J.8 (29%)


The number of whole numbers from 2 to 21 inclusive is (21 − 2) + 1 = 20.
(Notice that 19 is not correct!) There are 5 multiples of 4: 4, 8, 12, 16,
5 100
20. The required percentage is therefore × % = 25%,
20 1
hence (A).

6 1981 J.5 (85%)


70 × 200
70% of $2.00 = cents = $1.40,
100
hence (D).

7 1981 I.9 (22%)


Let the original price be 100x. The first discounted price is 90x. The
second discounted price is 1.05 × 90x = 94.5x. Therefore the nett
discount is 5.5%,
hence (B).

8 1981 S.7 (63%)


After
 3 years’ simple interest
 the investor has
10
$ 1000 + 1000 × × 3 = $1300.
100
After
 3 years’ compound interest
 the investor has
110 110 110
$ 1000 × × × = $(1000 × 1.331) = $1331. The return is
100 100 100
greater by $(1331 − 1300) = $31,
hence (E).

9 1982 J.4 (89%)


25 24
25% of 24 equals × = 6,
100 1
hence (C).

10 1982 J.9 (22%)


0.15 × 14 000 000
0.15% of 14 000 000 is = 0.15 × 140 000 = 21 000,
100
hence (B).
Properties of Numbers 115

11 1983 J.6 (79%)


35 7
35% = = ,
100 20
hence (E).

12 1984 J.17 (38%)


60
The number of residents who voted is × 2500. The number of
100
38 60
residents who voted for P is × × 2500 = 570,
100 100
hence (B).

PROPERTIES OF NUMBERS
1 1978 J.16 (30%), I.10 (49%), S.8 (75%)
Alternative 1
n + 1, n + 2, n + 3 are three consecutive integers. Since every second
integer is even, and every third one a multiple of 3, every product of
three consecutive integers contains a multiple of 2 and a multiple of 3,
and hence is divisible by 2 and 3. If x is divisible by both 2 and 3 it will
be divisible by 6. (Try it!) All integers are divisible by 1. Thus all
alternatives but (D) are excluded. You will find that x is sometimes, but
not always, divisible by 5,
hence (D).
Alternative 2
Let n = 1, then x = 2 × 3 × 4 = 24 and 5 is not a factor,
hence (D).

2 1978 J.23 (53%)


By direct addition, or using the sum of an arithmetic series,

1 + 2 + 3 + · · · + 12 = 78.
1
Therefore the sum of the numbers in each column equals 3 × 78 = 26,
hence (B).
116 Arithmetic Solutions

3 1979 J.6 (58%)


Alternative 1
For any integer n, 2n will be even and thus 2n + 1 will be odd,
hence (B).
Alternative 2
When n is even we note that each of 3n, n2 , n3 , n + 2 is even,
hence (B).

4 1979 J.10 (65%)


Alternative 1
Let the odd number be n, with digits X and Y . Thus n = 10X + Y .
Then the new number equals
100X + 10Y + 1 = 10(10X + Y ) + 1 = 10n + 1,
hence (B).
Alternative 2
Using a numerical example, say n = 32, then the new number is 321 or
10 × 32 + 1,
hence (B).

5 1979 J.25 (26%), I.16 (34%)


We need to note that the last digit of a product is equal to the products
of the last digits of the multiplicands. Thus if we focus only on last
digits we have 72 = (. . . 9), 73 = (. . . 3), 74 = (. . . 1); then,
 5 3  3
7 = 715 = 74 × 73 = (. . . 1)3 (. . . 3) = (. . . 3),
hence (B).

6 1979 J.30 (8%), I.30 (10%), S.30 (10%)


Let N cents be the unit cost, and let X, Y stand for the missing first
and last digits, respectively, of the account. Then the number X293Y
(i.e. the number with value X × 10 000 + 2000 + 900 + 30 + Y ) must be
equal to 22 times N . Thus X293Y is divisible exactly by both 11 and 2.
This means that Y must be even.
As the unit cost is more than $25, 22N > 22 × 2500 = 55 000, and hence
X = 6, 7, 8 or 9. Eleven is a factor of 22N and thus of X293Y . Using
the divisibility test for 11 (that the alternating sum of the digits is
Properties of Numbers 117

divisible by 11) we have:


(Notation: 11|X293Y means 11 divides exactly into X293Y )

11|X293Y ⇒ 11|X − 2 + 9 − 3 + Y
⇒ 11|X + Y + 4.
If X = 6 11|6 + Y + 4 ⇒ 11|10 + Y
⇒ Y = 1, but we require Y even.
If X = 7 11|7 + Y + 4 ⇒ 11|11 + Y
⇒ Y = 0, which is acceptable.
If X = 8 11|8 + Y + 4 ⇒ 11|12 + Y
⇒ Y is not a single digit.
If X = 9 11|9 + Y + 4 ⇒ 11|13 + Y
⇒ Y = 9, but we require Y even.

Thus 22N = 72930 and N = 3315. Therefore the unit cost is $33.15,
hence (C).

7 1981 J.26 (18%), I.23 (11%), S.17 (9%)


We note that 3 ≡ 3 (mod 11), 3 ≡ 9 (mod 11), 33 = 27 ≡ 5 (mod 11),
1 2

34 = 81 ≡ 4 (mod 11), 35 = 243 ≡ 1 (mod 11). Therefore


 
31981 + 2 = 3 31980 + 2
 396
= 3 35 +2
396
≡ 3 (1) + 2 (mod 11)
≡ 3 + 2 = 5 (mod 11)

The remainder on division of 31981 + 2 by 11 is 5,


hence (A).

8 1982 J.3 (89%)


The next two whole numbers are 1010 and 1011,
hence (B).
118 Arithmetic Solutions

9 1982 J.16 (37%)


Ten ‘2’s are needed for the numbers ending with 2 (2, 12, . . . , 92), and a
further ten are needed for the numbers beginning with 2 (20 to 29
inclusive). Thus twenty ‘2’s are required,
hence (A).

10 1983 J.25 (16%)


Alternative 1
Note that of the four letters, only A does not appear in the hundreds
position, hence it has the least effect on the sum and one can readily
assign to it the value 1. Of the others, all have equal weight in the
hundreds position, but D appears twice among the units, while M
appears there only once and B does not appear at all, hence D has the
greatest influence on the outcome—hence the assignment of 9 to it—to
be followed by assigning 8 to M and 3 to B. The sum of the resulting
numbers, 319, 819 and 918 is 2056,
hence (C).
Alternative 2
Since the value of the number presented by BAD is 100B + 10A + D,
with similar expansions for M AD and DAM , the value of the given sum
is 100B + 101M + 102D + 30A. This will be maximised if we allocate
the highest possible value to D, the next to M , the next to B and the
lowest to A. Thus D = 9, M = 8, B = 3, A = 1. The sum is then 2056,
hence (C).

12 1982 J.25 (37%), I.24 (25%), S.19 (26%)


Alternative 1
Adding the two numbers and isolating those parts which are multiples of
9 we have

6a3 + 2b5 = 600 + 10a + 3 + 200 + 10b + 5


= 808 + 10a + 10b
= 801 + 9a + 9b + a + b + 7
= 9(89 + a + b) + (a + b + 7)
Properties of Numbers 119

Now since 0 ≤ a ≤ 9 and 0 ≤ b ≤ 9, 0 ≤ a + b ≤ 18. If a + b + 7 is to be


divisible by 9 then a + b = 2 or 11. Thus the largest possible value is 11,
hence (E).
Alternative 2
Let N = 6a3 + 2b5. Either a + b < 10 or a + b ≥ 10. Then
N = 8(a + b)8, i.e. N = 9(a + b − 10)8. But N |9. Thus a + b = 2 or
a + b − 10 = 1, and a + b = 11 is the largest possible value of a + b,
hence (E).

13 1982 I.27 (11%), S.26 (13%)


However many sheep are bought, their cost, in dollars, will end in 0.
The cost of hens will be even. As the total cost ends in 9, the number of
cows must be odd, with cost ending in 5 and the number of hens must
be such that their cost ends in 4. Thus there are 2n + 1 cows and
5m + 2 hens (m and n integers). By subtraction there are
(97 − 2n − 5m) sheep. Costing these gives:

3279 = 65(2n + 1) + 2(5m + 2) + 40(97 − 2n − 5m)


= 3949 + 50n − 190m.
19m − 67
Therefore 190m − 50n = 670 and so n = . Now, if the
5
numerator is to be divisible by 5, 19m must end in 7 or 2, so m must
end in 3 or 8.
If m = 3, n < 0: not possible.
If m = 8, n = 17: 42 hens, 35 cows, 23 sheep.
If m = 13, n = 36: 65 hens, 73 cows, i.e. too many animals.
Thus the farmer bought 42 hens, 35 cows, 23 sheep,
hence (D).
Note: There are many alternative ways of answering this question.

14 1982 S.2 (90%)


9821 − 1289 = 8532,
hence (D).
120 Arithmetic Solutions

15 1982 S.22 (5%)


First note that
100! = 100 × 98 × · · · × 3 × 2 × 1. (1)
Zeros at the end correspond to factors of ten. These arise each time a
factor of five is multiplied by a factor of two. As there are only 15 even
numbers in the expansion (1) the power of 2 in 100! is at least 50.
The numbers in (1) divisible by 5 are 5, 10, 15, . . . , 95, 100.Of these
twenty, four (25, 50, 75, and 100) have 52 as a factor. Thus the power of
5 in 100! is 16 + 4 × 2) = 24. Therefore there are 24 zeros at the end of
the integer 100!,
hence (E).

16 1982 S.24 (24%)


Consider the alternatives:
2x − 3 2(x − 1) − 1
(A) 2x ≡ 3 (mod 12) ⇒ = is an integer. This is
12 12
false as the numerator is odd and the denominator is even.
3x − 7 3(x − 2) − 1
(B) 3x ≡ 7 (mod 12) ⇒ = is an integer. This is
12 12
false since 3 divides the denominator but not the numerator.
6x − 11 6(x − 2) + 1
(C) 6x ≡ 11 (mod 12) ⇒ = is an integer. This is
12 12
false since 6 divides the denominator but not the numerator.
10x − 5 2(5x − 2) − 1
(E) 10x ≡ 5 (mod 12) ⇒ = is an integer. This is
12 12
false since the numerator is odd and the denominator is even.
5x − 9
(D) 5x ≡ 9 (mod 12) ⇒ is an integer. This is true if x = 9,
12

hence (D).
Properties of Numbers 121

17 1983 J.14 (37%)


Alternative 1
Each day 7 tablets are taken. Since 7 × 24 = 168, there are 168 tablets
168
altogether. If 8 tablets are taken per day, they will last days, i.e. 21
8
days,
hence (A).
Alternative 2
7
Simply say × 24 days, i.e. 21 days,
8
hence (A).

18 1983 S.24 (34%)


We should start by noting that there are equal numbers of digits in both
summands. Since the initial 2s and 3s produce three 5s and since the
next digits of the sum are 7s, which can be produced only from 2s and
5s (i.e. 3 + 3 = 7), it follows that z = 3. Similarly, x = 5, since only two
7s are produced, which must result from the initial 2s of the first and
the middle 5s of the second summand. Noting that the total number of
digits is the same in all three numbers, one may further conclude that

x + y + z = 3 + 2 + 3 + 1 + 3 = 12,

hence
y = 12 − x − z = 12 − 5 − 3 = 4.
Thus (x, y, z) = (5, 4, 3),
hence (E).
Note: If the restriction on the second summand is weakened so that the
problem is written

2x 3y 5z + 3p 5q 2r = 53 72 83 51 73

then the same solution for (x, y, z) arises.


122 Arithmetic Solutions

19 1983 S.25 (9%)


Note that
√ 
if 1 ≤ n ≤ 7 then 3
n = 1,
√ 
if 8 ≤ n ≤ 26 then 3
n = 2,
√ 
and if 27 ≤ n ≤ 63 then 3
n = 3.

As each of the distractors is less than 64, one need go no further. Letting
√  √  √  √ 
3 3 3
f (n) = 1 + 2 + 3 + ··· + 3 n ,

it follows that

 n, 1≤n≤7
f (n) = f (7) + 2(n − 7) or 2n − 7, 8 ≤ n ≤ 26

f (26) + 3(n − 26) or 3n − 33. 27 ≤ n ≤ 63

Thus f (n) = 2n implies 3n − 33 = 2n or n = 33,


hence (B).
Note: The equation f (n) = kn has a unique solution for integral values
k > 2 as well. In fact, one may show that the equation is satisfied if and
only if
2
((k + 1)(k + 2))
n= − (k + 1),
4
which is integral for all values of k.

20 1984 J.24 (2%), S.10 (10%)


The number of terms divisible by 2 is 25, by 4 is 12, by 8 is 6, by 16 is 3
and by 32 is 1. The maximum number of divisions by 2 is
25 + 12 + 6 + 3 + 1 = 47,
hence (C).
Properties of Prime Numbers 123

21 1984 J.25 (8%)


Suppose the face numbers are a, b, c, d, e and f . There are 8 vertices,
each vertex number being a sum of these face numbers, i.e. 24 numbers
altogether. The 6 face numbers must occur equally often, in the sum of
the vertex numbers. The sum of the vertex numbers will be
4(a + b + c + d + e + f ), i.e. a number divisible by at least 4,
hence (B).

22 1984 S.27 (6%)


There are 125 such numbers, each of the 5 possible digits appearing in
each position 25 times.
Answer = 2 500 +250 +25 (from 1s)
+7 500 +750 +75 (from 3s)
+12 500 +1 250 +125 (from 5s)
+17 500 +1 750 +175 (from 7s)
+22 500 +2 250 +225 (from 9s)
= 62 500 +6 250 +625 = 69 375,
hence (A).

PROPERTIES OF PRIME NUMBERS


1 1978 J.30 (8%), I.29 (8%)
Since n = pq, and p and q are distinct primes, the numbers which do
have factors in common with n are the multiples of p and the multiples
of q in the range 2 to n:
p, 2p, 3p, . . . , (q − 1)p, qp = n,
q, 2q, 3q, . . . , (p − 1)q, pq = n,
giving in all p + q − 1 (since n is counted twice) distinct numbers. The
number of integers in the range 2 to n is n − 1. Hence the number of
integers with no factors in common is (n − 1) − (p + q − 1) = n − (p + q),
hence (B).

2 1978 S.13 (45%)


770 = 2 × 5 × 7 × 11. Clearly an adult cannot be 2, 5, 7 or 11. The other
combinations of ages are 10 and 77, 14 and 55, 22 and 35. The last pair
124 Arithmetic Solutions

is the only feasible pair and has sum 57,


hence (B).

3 1978 S.28 (8%)


If p and q are primes then p  q and q  p. Thus

p|q 2 − q = q(q − 1) ⇒ p|q − 1 (1)


and q|p2 + p = p(p + 1) ⇒ q|p + 1. (2)

From (1), q − 1 ≥ p, that is, p + 1 ≤ q. From (2), p + 1 ≥ q. Therefore


p + 1 = q. Since the only consecutive integers that are primes are 2 and
3, p = 2, q = 3. To complete the proof it is now necessary to check that
this pair actually satisfies the conditions (it is still possible that there is
no solution). It does,
hence (B).

4 1981 I.5 (41%)


The prime numbers less than 24 are 2, 3, 5, 7, 11, 13, 17, 19, 23.
24 = 5 + 19 = 7 + 17 = 11 + 13,
hence (D).

5 1982 J.28 (14%), I.17 (20%), S.14 (27%)


Noting that 1664 = 2 × 2 × 2 × 2 × 2 × 2 × 2 × 13 = 27 × 13, we can find
possible ages for the oldest and hence for the youngest child. From these
the products of the ages of the remaining children can be deduced:

Product of
Oldest Youngest remaining Valid?
13 8 16 NO
16 8 13 YES
16 13 8 NO
26 16 4 NO
32 26 2 NO

From this it follows that there are three children, aged 16, 13 and 8,
hence (B).
Properties of Prime Numbers 125

6 1983 J.13 (19%)


All side lengths of the brick must be factors of 42, and to help us find
these factors we notice that 42 = 1 × 2 × 3 × 7. Also the length and
width of the brick must add to 9 cm (half the base perimeter of 18 cm).
Hence the width is 2 cm and the length is 7 cm. Thus the height is 3 cm,
hence (A).

7 1983 I.26 (11%)

Let h = length of halberd (feet),


n = number of days in the month of the battle,
p = period between death and discovery,
and a = age of the captain at death.
p a
Therefore h × n × × = 451 066, or
2 2
3
h × n × p × a = 2 × 7 × 11 × 29 × 101. The only possibilities for n are
n = 28 or n = 29. Thus the month must be February. Suppose firstly
that n = 28, i.e. n = 22 × 7. We note that p can’t be 2, 29, 11, 22, 58,
nor 29 × 101. The only other possibilities are
(i) p = 2 × 101, in which case a × h = 29 × 11. But a = 11 (too young)
and h = 11 (too big).
(ii) p = 29 × 11, in which case a = 2 or 101, which are both impossible.
So n = 28. Thus n must be 29, and the year was a leap year,
hence (C).
Note: Whereas the above reasoning is sufficient to answer the question
the following reasoning leads to the year being 1512. Guessing that p
must be near 400, the only possibility is p = 22 × 101 = 404. Therefore
the captain has been killed between 1914 − 404 and 1918 − 404, i.e. 1510
and 1514. The only leap year is 1512. Note also that this gives
a × h = 2 × 7 × 11, yielding a = 22, h = 7.

8 1984 J.18 (9%), I.8 (18%)


504 = 2 × 2 × 2 × 3 × 3 × 7. To form a perfect square, 504 must be
multiplied by at least 2 × 7 = 14,
hence (D).
126 Arithmetic Solutions

9 1984 I.17 (13%)


2
If N has 18 (= 2 × 3 ) as a factor and is the square of a square, the least
N
value of N is 24 × 34 or 2 × 32 × 23 × 32 . Thus the smallest value of
18
is 23 × 32 = 72,
hence (C).

PROPORTION
1 1978 I.17 (60%)
The temperature, in degrees Celsius, is
100
28 × = 35,
80

hence (B).

2 1980 J.22 (12%)


2
The proportion of sand in the mixture is 7.
Hence in one cubic metre
(100 × 100 × 100 = 1 000 000 cm3 ) of mixture there are
2 × 1 000 000
= 286 000 cm3
7
of sand,
hence (C).

RATE, TIME AND DISTANCE


1 1978 S.25 (27%)
Let M denote the minute hand, H the hour hand.
Alternative 1
The time for one sweep of M is 2 hours. The time for one sweep of H is
4 hours.
Thus the hands next coincide after two sweeps of M (and one sweep of
H), after 4 hours, i.e. at 4.00 pm,
hence (D).
Rate, Time and Distance 127

Alternative 2
M normally moves at 12 times the angular speed of H. M now moves
only twice as fast as H.
Whenever M moves an integer number (k) times as fast as H, the
hands’ next coincidence in the 12 o’clock position is after k revolutions
of M (and one of H). During this time the hands have coincided at
k − 1 equally spaced time intervals.
Therefore the time interval is
time for H to make one revolution 4
= = 4,
k−1 2−1

hence (D).

2 1979 J.16 (54%)


The total time taken is
 
20 20 5
+ hours = hours = 50 min,
40 60 6

hence (C).

3 1979 I.11 (49%)


As Jack is climbing at a uniform rate he is, at 3 o’clock, halfway
between the two given positions.
Alternative 1
The fraction climbed at 3 o’clock equals
1
(fraction at 2 o’clock) + (distance between positions)
  2
1 1 3 1 1 1 7 11
= + − = + × = .
6 2 4 6 6 2 12 24
hence (C).
Alternative 2
A convenient way of finding the midpoint of the two numbers is to take
their average. This gives the fraction climbed at 3 o’clock as
 
3 1
+
4 6 1 22 11
= × = , hence (C).
2 2 24 24
128 Arithmetic Solutions

4 1979 I.23 (6%)


The total time taken is
 
20 20 11
+ hours = hours.
50 60 15
Thus the average speed is
40 600 6
km/h = km/h = 54 km/h,
11 11 11
15

hence (C).

5 1981 J.22 (6%)


The full journey is to take half an hour. The first 10 km takes
10 2
= hour.
25 5
The remaining 10 km then takes
1 2 1
− = hour.
2 5 10
This must be done at
10
1 = 100 km/h,
10

hence (A).

6 1981 I.25 (11%), S.26 (9%)


Suppose the distance to the base of the hill is x km and from the base to
the top y km. Then
x y y x
time taken = + + +
8 6 12 8
x+y
=
4
1
= 1 hours.
2
Rate, Time and Distance 129

Thus x + y = 6, and the total distance run is 2x + 2y = 12,


hence (D).

7 1982 J.18 (18%), I.12 (40%), S.7 (64%)


As the volume doubled every minute, it was half full at 9:59 am and
one-quarter full at 9:58 am, hence (E).

8 1982 J.23 (25%), S.12 (61%)


Suppose the first part of the journey takes t hours and the second part
takes 2t hours. The distance travelled in the first part is 800t km and for
the whole trip is 700 × 3t = 2100 km. Therefore the distance in the
second part is 2100t − 800t = 1300t km. Thus the average speed for the
second part is 1300t
2t = 650 km/h,
hence (C).
Note: This question can also be done by assuming that the whole
journey takes 3 hours.

9 1983 J.10 (11%)


Four tyres were used at any time. Thus the total number of kilometres
of wear is
20 000 × 4 = 80 000.
Since this wear was shared equally by each of the 5 tyres, each tyre
travelled 80 000/5 or 16 000 km, hence (C).

10 1983 J.12 (28%)


The total fraction of the bridge which spans banks is 2 + 3 = 56 .
1 1

Therefore 1 − 56 = 16 of the bridge spans the river. Thus the total length
of the bridge is 6 × 50 = 300 metres,
hence (D).

11 1983 J.18 (25%)


2000
One minute before impact, the faster missile must still travel km,
60
1000
while the slower missile must still travel km. Thus they are
60
2000 1000 3000
+ = = 50
60 60 60
kilometres apart with one minute to go, hence (E).
130 Arithmetic Solutions

12 1983 J.27 (6%), I.17 (9%)


...............................................
......... .......
Let the radius of the wheel be R, then ....... ......
...... .....
........
. .....
.....
....
the circumference is 2πR. The dis- ..
....
.
. ...
...
...
. ...
..
tance travelled per second is ....
. ...
...
...
... ...
...
....
...
.
....
.......... R ...
.
..
4 × circumference ...
... ..
.
...
... ..
.
... ...
= 8πR metres ...
...
...
...
...
...
8πR ...
.....
..... .....
.
...
.
= kilometres. .....
...... ......
.....
1000 .......
.........
...............................................
..........

Therefore the distance travelled per hour is


8πR × 3600
km = 60 km (given).
1000
So
60 × 1000 25
R= metres = metres.
8π × 3600 12π
25
So the diameter is metres, hence (E).

13 1983 I.13 (23%)
Let the time taken to paint the room be T hours. The fraction of the
room painted by the painter is T /12. The fraction of the room painted
by the apprentice is T /24. These must add up to 1. Thus

T T
+ = 1,
12 24
2T + T
i.e. = 1,
24
i.e. 3T = 24,

giving T = 8,
hence (B).

14 1983 I.20 (19%), S.15 (37%)


At the end of the race Marjorie has run 1800 metres while Betty has run
1710 metres. Thus while Marjorie runs the last 200 metres Betty runs
Rate, Time and Distance 131

an extra
1710
× 200 metres,
1800
that is, 190 metres. Thus when Marjorie finishes, Betty has run
1710 + 190 metres, that is, 1900 metres, so she is 100 metres behind
Marjorie,
hence (B).

15 1984 J.8 (53%)


The first film will finish at 9.07 pm. The interval will finish at 9.22 pm.
Adding 1 hour 51 minutes is then the same as adding 2 hours and
subtracting 9 minutes, giving 11.13 pm,
hence (A).

16 1984 J.15 (76%), I.10 (88%), S.5 (96%)


1 100
12.5 km/L = 12.5 L/ km = 12.5 L/100 km = 8 L/100 km,
hence (A).

17 1984 J.26 (3%), I.23 (6%), S.14 (16%)


Suppose Kirsten’s walking speed is v units and her running speed is 2v
units. On the first day she runs 13 of the time (i.e. 20
3 minutes) and
walks 23 of the time (i.e. 40
3 minutes). Thus distance d, from home to
school, equals
20 40 40v 40v 80v
2v × +v× = + = .
3 3 3 3 3
On the second day let her travelling time be 3t minutes, t spent walking
and 2t spent running. Then
80v
d = 2t × 2v + t × v = ,
3
from above. Thus
80v
5tv = ,
3
giving
80 3
3t = × = 16,
3 5

hence (A).
132 Arithmetic Solutions

SQUARE ROOT
1 1978 J.8 (52%)
√ √
15 × 20 × 12 = 5 × 3 × 5 × 4 × 3 × 4 = 5 × 4 × 3 = 60,
hence (B).
ALGEBRA SOLUTIONS

ABSOLUTE VALUE
1 1978 J.27 (8%), I.15 (15%)
Alternative 1
|x − 1| = 2x ⇒ 2x ≥ 0 ⇒ x ≥ 0. This excludes (A) and (D). (B) and (C)
can be excluded by substitution,
hence (E).
Alternative 2
|x − 1| = 2x ⇒ x − 1 = 2x or −(x − 1) = 2x, i.e. x = −1 or x = 13 . But
we require x ≥ 0 (as in Alternative 1), so x = 13 ,
hence (E).

2 1979 J.18 (17%), I.12 (38%)


|x| = |y| and x < 0 < y, hence x = −y. Checking the truth of the given
statements:
(A) x2 is positive, y is positive, so x2 y > 0 True
(B) x = −y ⇒ x + y = 0 True
(C) xy = −y 2 ⇒ xy < 0 True
1 1 1 1 1 1 1
(D) − =− − =− ⇒ − = 0
x y y y 2y x y
x −y x
(E) +1= +1⇒ +1=0 True
y y y

hence (D).

3 1979 I.22 (48%)


Alternative 1
|x − 1| − |x − 2| = 0. Therefore |x − 1| = |x − 2|. Considering these as
distances on the number line we have that the distance of x from 1
1
equals the distance of x from 2, i.e. x = 1 ,
2
hence (E).
134 Algebra Solutions

Alternative 2
|x − 1| = |x − 2|. Therefore

(x − 1)2 = (x − 2)2 , (∗)


3
i.e. x2 − 2x + 1 = x2 − 4x + 4, i.e. 2x = 3, i.e. x = , and this does
2
satisfy the equation,
hence (E).
Note: It is a necessary step to check this apparent solution in the
original equation, because in line (∗) an extra solution may have been
introduced.
Alternative 3
|x − 1| = |x − 2|.
Case (i): If x < 1 or x ≥ 2, x − 1 = x − 2, but no value of x in this range
is a valid solution.
3
Case (ii): If 1 ≤ x < 2, x − 1 = −(x − 2), i.e. 2x = 3, i.e. x = , which
2
is a valid solution,
hence (E).

4 1980 I.22 (18%), S.18 (54%)


Alternative 1
|x| + |x − 1| = 1
(distance of x from zero) + (distance of x from 1) = 1

Inspection of the number line

−2 −1 0.. 1.. 2.. 3..


 ... ..
.. ... .. .. ... .. ... .. ... .. ... .. 

shows that 0 ≤ x ≤ 1 is the solution,


hence (E).
Alternative 2 
x if x≥0
|x| =
−x if x<0
and 
x − 1 if x − 1 ≥ 0, i.e. x ≥ 1
|x − 1| =
−x + 1 if x − 1 < 0, i.e. x < 1
Absolute Value 135

Case (i): x ≥ 1. Here, |x| + |x − 1| = 1 gives x + x − 1 = 1, i.e. x = 1.


Case (ii): 0 ≤ x < 1. Here |x| + |x − 1| = 1, i.e. x + (−x + 1) = 1,
i.e. 1 = 1, hence all values of x such that 0 ≤ x < 1 satisfy the equation.
Case (iii): x < 0. Here, |x| + |x − 1| = 1 gives −x + (−x + 1) = 1,
i.e. x = 0, which is invalid as it does not lie in the restricted domain.
Valid solutions arise only in cases (i) and (ii), giving 0 ≤ x ≤ 1,
hence (E).

5 1981 S.19 (49%)


For questions such as this it is often simplest to draw a graph of the
function.
(i) For x < −1, |x + 1| + 2|x − 2| = −x − 1 − 2(x − 2) = −3x + 3 (line,
slope −3, intersecting y-axis at y = 3).

(ii) For −1 ≤ x < 2, |x + 1| + 2|x − 2| = x + 1 − 2(x − 2) = −x + 5 (line,


slope −1, intersecting y-axis at y = 5).

(iii) For ≥ 2, |x + 1| + 2|x − 2| = x + 1 − 2(x − 2) = 3x − 3 (line, slope


3, intersecting y-axis at y = −3).
..... ..
.... ....
.....
y ....
....

..... .
.... ..
.
.... ....
.... ....
....
.........
6
......... ....... ....... ....... ....... ....... ....... ....... ....... ....... ....... ....... ....... ....... ....... ....... ......
....
.
........... .
.
..
.... ......... ....
......... .... ..
......... .... ...
..
..
.........
......... .
.....
. ...
......... .
......... .... ..
.... ......... ....
......... .
..
..... ...
......... .... ..
.. ......... ..
.. ......... ....
.........
... .
.
.....
. ...
..
.........
.... ........ .......
..
3 ....... ....... ....... ....... ....... ....... ....... .......
..
...
..
.. .. ...
..
.... .... ...
..
.... ....
. . ...
.
.... ....
....
....
.
...
.
....
.
x

−1 1 2 3

The graph only intersects y = 6 at x = −1 and x = 3 and the solution is


hence −1 < x < 3,
hence (C).
136 Algebra Solutions

6 1983 S.22 (5%)

There are 8 cases to consider, depending x+y =0  x=0


on whether each of x, y and x + y are ≥ 0  2
or < 0. This is immediately reduced to 6  1
since 3  y =
0

 6
x ≥ 0, y ≥ 0 ⇒ x + y ≥ 0, 
and x < 0, y < 0 ⇒ x + y < 0. 4 5 


These are the 6 regions marked:


1 2  y=1
1. .............

..... x+y=1 ........................
..... ...........
......
.
............
............
 .
1

x ≥ 0, y ≥ 0, x < 0, y ≥ 0, x + y ≥ 0.
Here x + y + x + y ≤ 2. Here −x + y + x + y ≤ 2,
i.e. x + y ≤ 1. i.e. y ≤ 1.

3  4

..
....
. . ..
x=−1 ..........
.....
.................
. . . . . ..
............
.....................
.......
x+y=−1 ..


x < 0, y ≥ 0, x + y < 0. x < 0, y < 0


Here −x + y − x − y ≤ 2. Here −x − y − x − y ≤ 2.
i.e. x ≥ 1. i.e. x + y ≥ −1.
Collection of Like Terms 137

5 6

.............
 ........................
.
.....
......
..... ........... x = 1
........
..
...............
. . . . . .. .

y=−1

x ≥ 0, y < 0, x + y < 0. x ≥ 0, y < 0, x + y ≥ 0.
Here x − y − x − y ≤ 2. Here x − y + x + y ≤ 2,
i.e. y ≥ −1. i.e. x ≤ 1.

Each of these regions is a triangle with area 21 . Therefore the total area
equals 6 × 12 = 3,
hence (B).
Note: The total region looks like

1
........................

..........................
............................
..............................

................................
..................................

....................................
......................................
........................................
 1
..........................................
−1 
............................................
.............................................

...........................................
.........................................
.......................................
 .....................................
...................................

.................................
...............................
.............................

...........................
.........................
............
−1

COLLECTION OF LIKE TERMS


1 1979 I.4 (86%)
15x − 10y − 8x + 13y = 7x + 3y,
hence (B).
138 Algebra Solutions

2 1978 J.7 (66%)


8x + 3y + 4x − 5y = 12x − 2y,
hence (E).

3 1979 J.9 (54%)


x + y + z = x + 3x + 4x = 8x,
hence (B).

4 1982 I.5 (95%)


3(2x − 4y) + 5x = 6x − 12y + 5x = 11x − 12y,
hence (A).

5 1981 I.4 (78%)


2x + 1 − 3(2 − x) = 2x + 1 − 6 + 3x = 5x − 5,
hence (A).

EQUATIONS: DIOPHANTINE
1 1984 J.22 (22%), I.20 (28%), S.12 (45%)
Let the number of 50c coins be x and the number of 20c coins be y.
Then

50x + 20y + 10(20 − (x + y)) = 500,


i.e. 5x + 2y + 20 − x − y = 50,
i.e. 4x + y = 30.

Since there are 20 coins in all, the possibilities are

50c (x) 20c (y) 10c


4 14 2
5 10 5
6 6 8
7 2 11

The first row gives the only possibility with more 50c coins than 10c
coins,
hence (C).
Equations: Diophantine 139

2 1984 I.29 (12%), S20 (19%)


Let x, y, z be the numbers of articles of types X, Y , Z bought, so

1 x + y + z = 100 (number equation)


and $ 8 x + y + 10z = $100 (cost equation)

Therefore

x + y + z = 100 (1)
x
+ y + 10z = 100 (2)
8
7 72
Subtracting (2) from (1) gives x − 9z = 0, i.e. x = z. Substitution
8 7
back into (1) and (2) gives
72
z+y+z = 100
7
9
and z + y + 10z = 100,
7
each of which is equivalent to the equation
79
z + y = 100.
7
The only valid non-zero solution for z is then z = 7 which gives
y = 100 − 79 = 21 and x = 72,
hence (D).

3 1982 J.26 (31%), I.21 (54%)


Alternative 1
From
x + y + xy = 34 (1)
34 − y
we have x + xy = 34 − y, i.e. x = . Also, as the left-hand side of
1+y
(1) is symmetric in x and y we may restrict ourselves to finding a
solution where y ≤ x. Substituting:

y 1 2 3 4 5
33 32 31 30 29
x =6 <y
2 3 4 5 6
140 Algebra Solutions

Thus x + y = 6 + 4 = 10 is the only solution,


hence (A).
Alternative 2
From
x + y + xy = 34 (1)
we have x + 1 + y(x + 1) = 35, i.e. (x + 1)(y + 1) = 35. Therefore the
only possible factors are 5 and 7, i.e. (x + 1) + (y + 1) = 12, i.e.
x + y = 10,
hence (A).

4 1979 I.27 (23%), S.21 (22%)


Alternative 1
Let N be the required integer. From the given information we know
that there are integers x, y, z such that 3x + 1 = N , 5y + 1 = N ,
7z + 5 = N . From the first two it follows that 3x = 5y = N − 1, so there
is an integer k such that N − 1 = 15k. Therefore
15k − 4 k−4
N − 1 = 15k = 7z + 5 − 1 giving z = = 2k + . Now k = 4
7 7
gives the smallest integer solution for z, and N = 7 × 8 + 5 = 61,
hence (E).
Alternative 2
Having established that N − 1 is of the form 15k, that is, N = 15k + 1,
we have the following possibilities:
1, 16, 31, 46, 61, 76, . . . .
The minimum number in this list giving remainder 5 on division by 7 is
61,
hence (E).

5 1979 S.27 (9%)



If (x − y)2 = 27 − 2y 2 then x − y = ± 27 − 2y 2 is an integer since x
and y are integers. Therefore A = 27 − 2y 2 must be a square number. If
y = 0, A = 27 which is not a square. If y = ±1, A = 25 which is square,
implying x ± 1 = ±5, i.e. x = ±4, ±6. If y = ±2, A = 19 which is not
square. If y = ±3, A = 9 which is square. This implies x ± 3 = ±3, i.e.
x = 0, ±6. Further values of y make A negative and so not square. Thus
the only integer values for x are ±6, ±4 and 0,
hence (D).
Equations: Diophantine 141

6 1983 I.29 (8%), S.27 (9%)


Alternative 1
One may cleverly observe that
(19)(100) + (83)(1) = (19)(83 + 17) + (83)(20 − 19),
= (19)(83) + (19)(17) + (83)(20) − (83)(19),
= (19)(17) + (83)(20)
and cancellation yields the second pair (x, y) = (17, 20), and hence
x + y = 17 + 20 = 37,
hence (B).
Note: The key to the above is the introduction of the (19)(83) twice,
once with a positive sign, by writing 100 as 83 + 17 and once with a
negative sign, by writing 1 as 20 − 19.
Alternative 2
To solve the problem from scratch (i.e. without utilising the given
obvious solution (100, 1)), one proceeds as follows (via Euclid):
From 19x + 83y = 1983,
7 − 7y
x = 104 − 4y + = 104 − 4y + z,
19
where z is an integer and 7y + 19z = 7, a Diophantine equation (i.e. one
to be solved over the domain of integers) similar to the original one, but
with smaller coefficients. Now
2z
y = 1 − 3z + = 1 − 3z + v,
7
from which 2x − 7v = 0, i.e.
v
z = 3v + = 3v + w, where v = 2w.
2
Substituting, one finds that
z = 3v + w = 6w + w = 7w,
thus y = 1 − 3z + v = 1 − 21w + 2w = 1 − 19w,
thus x = 104 − 4y + z = 104 − 4(1 − 19w) + 7w,
= 100 + 83w,
i.e. (x, y) = (100 + 83w, 1 − 19w),
142 Algebra Solutions

where w is an arbitrary integer, gives all integer solutions of the original


equation.
In view of the restriction to positive values of x and y, one is obviously
limited to w = 0 and −1, yielding the two solutions found before,
hence (B).

7 1980 S.30 (3%)


Alternative 1
Since 1 ≤ x ≤ y ≤ z ≤ u ≤ v, 4 + v ≤ x + y + z + u + v ≤ 5v, i.e.
4 + v ≤ xyzuv ≤ 5v or xyzu ≤ 5. Also 4 + v ≤ xyzuv. So
4 ≤ (xyzu − 1)v, i.e. xyzu > 1 and we can further assert 2 ≤ xyzu ≤ 5.
Trialling possibilities:

x y z u Sum = Product Valid Solution?


1 1 1 2 5+v = 2v
i.e. v = 5 Yes
1 1 1 3 6+v = 3v
i.e. v = 3 Yes
1 1 2 2 6+v = 4v
i.e. v = 2 Yes
1 1 1 4 7+v = 4v
7
i.e. v = No
3
1 1 1 5 8+v = 5v
i.e. v = 2 No, since v < u

Thus there are three distinct solutions,


hence (C).
Alternative 2
Given x, y, z, u, v are integers satisfying 0 < x ≤ . . . ≤ v we have
x + y + z + u + v = xyzuv, i.e.
1 1
+ ··· + = 1.
yzuv xyzu
Hence xyzu < 5. Thus x = y = 1 as x = v, and xyzu < 5. If z = 2 then
u = 2 and 6 + v = 4v, giving v = 2. If z = 1 then u ≤ 4 and
3 + u + v = uv giving u = 2 (and v = 5), u = 3 (and v = 3) while u = 4
is impossible. So there are three solutions,
hence (C).
Equations: Linear 143

8 1978 I.30 (8%)


Alternative 1
3(336 − x)
3x + 5y = 1008 gives 5y = 1008 − 3x = 3(336 − x) or y = . If
5
x and y are both positive integers then the following x values only are
suitable, 1, 6, 11, . . . , 331. Now 331 = 1 + 66 × 5, so there are a total of
66 + 1 = 67 solutions,
hence (D).
Alternative 2
The following is a general method for solving ‘diophantine equations’,
i.e. for finding all solutions in integers of equations of the form
ax + by = c, where a, b, c are integers.
One solution of 3x + 5y = 1 is x = −3 and y = 2. The general solution is
xt = −3 + 5t, yt = 2 − 3t, where t is an integer, since

3xt + 5yt = 3(−3 + 5t) + 5(2 − 3t) = −9 + 15t + 10 − 15t = 1.

A solution to 3x − 5y = 1008 is

x = 1008(−3) = −3024 and y = 1008(2) = 2016,

with general solution

xt = −3024 + 5t and yt = 2016 − 3t,

where t is any integer. xt > 0 implies 5t > 3024, i.e. t ≥ 605. yt > 0
implies 3t < 2016, i.e. t ≤ 671. We then note that 671 − 605 = 66, so
there are 66 + 1 = 67 solutions in positive integers. Explicitly, the
solution set is

{(5t − 3024, 2016 − 3t)} , t is integer and 605 ≤ t ≤ 671,

hence (D).

EQUATIONS: LINEAR
1 1982 J.12 (21%), I.8 (70%)
3(x − 4) = 7x − 10, so 3x − 12 = 7x − 10, i.e. −2 = 4x or x = − 12 ,
hence (E).
144 Algebra Solutions

2 1978 J.13 (31%)


4
2x + 5 = 9 − 3x gives 5x = 4, i.e. x = ,
5
hence (E).

3 1979 J.15 (28%), I.6 (74%)


1
2x + 5 = 5x − 11, i.e. 16 = 3x, i.e. x = 5 ,
3
hence (B).

4 1980 I.11 (43%), S.7 (77%)


1 1 1 1 1 1
= + . So since x = 2 and y = 3 we have = + , i.e.
x y z 2 3 z
1 1 1 1
= − = , giving z = 6,
z 2 3 6
hence (E).

5 1983 I.14 (76%)


Adding all three equations gives 2(a + b + c) = 24, i.e. a + b + c = 12.
a + b = 7 then gives c = 5, b + c = 9 gives a = 3 and a + c = 8 gives
b = 4. Thus abc = 60,
hence (A).

6 1980 J.15 (44%), I.10 (58%)


Suzanne’s age is (x − 3) years. Since the sum of Tom’s and Suzanne’s
ages is 15 years, x + (x − 3) = 15,
hence (B).

7 1980 J.14 (83%)


Alternative 1
Toby bought two more apples than Toni, leaving him with 50c less.
Thus the apples each cost 25c. Hence Toby’s pocket money was
(20 + (4 × 25))c = $1.20,
hence (B).
Alternative 2
Let the apples each cost x cents. Since Toni and Tobi each have the
same pocket money, 2x + 70 = 4x + 20, i.e. 50 = 2x or x = 25. So Toni’s
Equations: Linear 145

pocket money is ((2 × 25) + 70)c = $1.20,


hence (B).

8 1978 J.11 (35%)


◦ ◦ ◦
Let the angles of the triangle be 2x , 3x and 4x . Then
2x + 3x + 4x = 180, i.e. 9x = 180 or x = 20. So the largest angle 4x◦
equals 80◦ ,
hence (B).

9 1982 J.20 (59%), I.15 (74%)


Suppose there are x green marbles. Then there are x + 1 blue marbles,
x + 5 white marbles and x + 6 red marbles. Thus

x + (x + 1) + (x + 5) + (x + 6) = 20,

from which we have 4x + 12 = 20 and hence x = 2. Thus there are


x + 6 = 8 red marbles,
hence (A).

10 1983 I.10 (57%)


Let the larger piece weigh x carats. Then x + (x − 9) = 25, i.e.
2x − 9 = 25 or x = 17,
hence (B).

11 1980 J.20 (20%), I.13 (28%)


Suppose 4x boys and 4x girls are present. Then x boys and 2x girls have
jobs and 8x − x − 2x = 5x of the group do not. Thus 5x = 30, x = 6 and
the number present (8x) is 48,
hence (E).

12 1982 J.22 (46%)


Having spent $1.40, Jane and Ben still have $1.60. If Ben has x cents
then Jane has 3x cents. Thus 160 = x + 3x = 4x, and x = 40. Hence
Jane has 120c of her original $2, and had spent 80c,
hence (A).
146 Algebra Solutions

13 1980 S.12 (24%)


The number written 19xy has value 1900 + 10x + y. Since

1980 = (year of birth) + (age of person)


1980 = (1900 + 10x + y) + (1 + 9 + x + y)
= 1910 + 11x + 2y.

Therefore 70 − 11x − 2y = 0,
hence (C).

14 1983 J.21 (25%)


Suppose Adam buys x balls, each costing $y. With tax he pays
$(xy × 1.05). Without tax he pays $[(x + 3)y]. Therefore
3 300
x × 1.05 = x + 3, i.e. 0.05x = 3, giving x = = = 60,
0.05 50
hence (D).

15 1979 S.23 (10%)


Suppose initially that each candle is of length l. Let candle A be the
faster burning candle, expiring in 4 hours, and let B be the slower
burning candle. After t hours the lengths of the candles are
   
t t
A:l 1− and B :l 1− .
4 5

Now we require t such that

length of candle B = 3 (length of candle A) ,


   
t t
i.e. l 1 − = 3l 1 − ,
5 4
i.e. 20 − 4t = 60 − 15t,
i.e. 11t = 40,
40
i.e. t = ,
11
hence (A).
Equations: Linear 147

16 1979 I.17 (13%), S.10 (31%)


Let the pool be 2x m wide and 3x m long. Then the area of the path is
[2 × (3x + 2) × 1] + [2 × (2x) × 1]. Therefore 44 = 6x + 4 + 4x, i.e.
40 = 10x or x = 4. Thus the area of the pool is 12 × 8 = 96 m2 ,
hence (C).

17 1983 I.15 (52%)


3 5
Let the normal operating temperature be T ◦ . Then T − 100 = T , i.e.
4 8
1
T = 100 or T = 800,
8
hence (D).

18 1982 I.23 (21%)


We note that 11 = 3 × 2 + 5 and 50 = 3 × 15 + 5. If N is the missing
integer we have either 3 × 6 + 5 = N giving N = 23 or 3 × N + 5 = 6
1
giving N = . As N is an integer, N = 23,
3
hence (A).
Note:
(i) There is an additional correct integer answer to this question,
although it was not offered as an alternative. This is because there
is also the linear relation with 2 = 12 × 11 − 130 and
50 = 12 × 15 − 130 giving a possible integer solution
12 × 6 − 130 = −58.
(ii) There are also two non-integer solutions possible.
(iii) In higher mathematics the term ‘linear’ would normally be
restricted to cases of the form y = ax (i.e. lines through the
origin). The term ‘affine’ is used to cover all relations of the form
y = ax + b.

19 1979 J.24 (24%), I.14 (37%)


Alternative 1
Let h cm be the required height of juice. Then the volume of juice plus
10% of this volume will equal the volume of the container. Therefore
2 1
π (12) h + π(12)2 h = π(12)2 (16.5),
10
148 Algebra Solutions

1
i.e. h+ h = 16.5,
10
i.e. 11h = 165,
i.e. h = 15,
hence (D)
Alternative 2
Noting that the volume of a cylinder is proportional to its height, we
1
immediately have the equation h + h = 16.5, which is solved as above,
10
hence (D).

20 1978 S.12 (43%)


Let x be the required number of additional consecutive heads. Then
110 + x 7
= .
200 + x 10
Therefore 1100 + 10x = 1400 + 7x, i.e. x = 100,
hence (D).

21 1980 S.13 (50%)


Let x denote the sum of the remaining numbers. Then
x + 45 + 55
= 38, i.e. x = 1800. Therefore the mean of the remaining
50
x 1800
numbers is , i.e. or 37.5,
48 48
hence (E).

22 1983 J.29 (5%), I.25 (5%), S.19 (8%)


Let the total amount of money to be distributed be $P . Equating the
amounts received by the first and second children only,
 
1 1 1
1000 + (P − 1000) = 2000 + [P − 2000 − 1000 + [P − 1000] ],
10 10 10
  
Amount received
by first child

with solution P = 81 000. The first child receives


1
1000 + (80 000) = 1000 + 8000 = 9000.
10
Equations: Quadratic 149

The second child receives


1
2000 + (70 000) = 2000 + 7000 = 9000,
10
and so on. As all children received equal amounts there were
81 000
= 9 children,
9 000
hence (D).
Note: There is the trivial solution (1 child), but this is considered not
consistent with the wording of the question.

EQUATIONS: QUADRATIC
1 1978 I.7 (40%)
Alternative 1
The roots have a sum of 9 and a product of −36,
hence (A).
Alternative 2
Since x2 − 9x − 36 = 0 we have (x − 12)(x + 3) = 0, i.e. x = 12 or
x = −3,
hence (A).

2 1982 S.6 (42%)


Alternative 1
We use the result that the sum of the roots, α + β of the equation
−b
ax2 + bx + c = 0 is given by α + β = . The average of these roots is
a
α+β −b −b −14 −7
thus = . In this case a = 2, b = 14, so = = ,
2 2a 2a 2×2 2
hence (E).
Alternative 2
The roots of the equation 2x2 + 14x + 17 = 0 are

−14 ± 196 − 8 × 17
x= .
4
The average is
 √ √ 
1 −14 + 196 − 8 × 17 −14 − 196 − 8 × 17
+ ,
2 4 4
150 Algebra Solutions

 
1 −14 −14 14 7
i.e. 4 + =− =− ,
2 4 4 2
hence (E).

3 1983 S.6 (41%)


1
Since (x − 3)(2x + 1) = 0 then possible values of x are 3 and − . Thus
2
possible values of 2x + 1 are 7 and 0,
hence (D).

4 1981 I.20 (8%)


 2
1 1 1 1
x + = 3. Thus x+ = 9, i.e. x2 + 2 + 2 = 9, i.e. x2 + 2 = 7,
x x x x
hence (E).

5 1983 I.21 (13%)


2
If the remaining area is x square metres, 
4
as shown, x2 = 14 (x + 8)2 , i.e. 4x2 =
x2 + 16x + 64, i.e. 3x2 − 16x − 64 = 0, ....... ....... ....... 

or (3x + 8)(x − 8) = 0. The only positive x
solution of this equation is x = 8. So the ........
....... ....... ....... 
area of the unmown section is 82 , ...
.
...
. 
.. ... 4
i.e. 64 square metres, .. .

 4  x  4  
... ...
hence (D). . .

6 1979 S.14 (14%)


The expression can be rearranged and factorised as a quadratic:

9(log x)2 − 12(log x)(log y) + 4(log y)2 = 0

leading to (3 log x − 2 log y)2 = 0, i.e. 3 log x = 2 log y, i.e.


log x3 = log y 2 , or x3 = y 2 ,
hence (A).
Equations: Rearrrangement of 151

EQUATIONS: REARRANGEMENT OF
1 1978 I.12 (35%)
a + 3b a
= 3 implies a + 3b = 3a − 3b, i.e. 6b = 2a or 3 = ,
a−b b
hence (C).

2 1983 I.16 (12%), S.9 (60%)


1 1 1 1 1 1 y−x xy
If = + then = − = , i.e. z = ,
x y z z x y xy y−x
hence (D).

3 1983 S.10 (13%)


1
If a2 + b2 = (a + b)2 − 2ab = 12 − 2ab = 2, then ab = − . Therefore
2
 2
4 4
 2 
2 2 2 2 2 1 1 1
a +b = a +b − 2a b = 2 − 2 − =4− =3 ,
2 2 2

hence (E).

4 1978 I.14 (23%)



1 4π 2 l 4π 2 l
T = 2π implies T 2 = , i.e. g = 2 ,
g g T
hence (D).

5 1980 I.17 (17%)


2
Suppose the circle has radius r cm and area A cm . Then
p  p 2 p2
p = 2πr ⇒ r = , and A = πr2 ⇒ A = π = ,
2π 2π 4π
hence (A).

6 1979 I.20 (27%)


A A A − 2πr2
If A = 2πr(r + h) then r + h = , i.e. h = −r = ,
2πr 2πr 2πr
hence (B).
152 Algebra Solutions

7 1981 S.16 (58%)


    
1 1 1 1
If v = 2GM − then v 2 = 2GM
− , i.e.
r R r R
v2 1 1 1 v2 1 v 2 R + 2GM
= − . Thus = + = .
2GM r R r 2GM R 2GM R
2GM R
Thus r = 2 ,
v R + 2GM
hence (B).

8 1980 I.21 (15%)


x+2
Since y = , xy − 3y = x + 2, i.e. xy − x = 3y + 2, i.e.
x−3
3y + 2
x(y − 1) = 3y + 2 or x = ,
y−1
hence (B).

EQUATIONS: ROOTS OF
1 1980 I.5 (55%)
1 1
If 8r3 = 1 then r3 = , i.e. r = ,
8 2
hence (E).

2 1978 I.23 (5%)



Substituting x = 3 − 1 into ax2 − 5x + 1 = 0,
√ 2 √   √  √
a 3 − 1 − 5 3 − 1 + 1 = 0, i.e. a 3 − 2 3 + 1 − 5 3 + 6 = 0.
This gives
√  √   √ 
5 3−6 5 3−6 2+ 3
a = √ =  √ × √ 
4−2 3 2 2− 3 2+ 3
√ √ √
10 3 + 15 − 12 − 6 3 4 3+3 √ 3
= = =2 3+ ,
2(4 − 3) 2 2

hence (D).
Equations: Simultaneous 153

EQUATIONS: SIMULTANEOUS
1 1980 J.10 (61%)
Alternative 1
The only integer lengths giving an area of 21 cm2 are 7 cm and 3 cm.
These do indeed give a perimeter of 2(7 + 3) = 20 cm,
hence (D).
Alternative 2
Let the width be x cm. Then the length is (10 − x) cm. Therefore
x(10 − x) = 21, i.e. 10x − x2 = 21, i.e. x2 − 10x + 21 = 0 or
(x − 3)(x − 7) = 0. Hence the width is 3 cm (taking the width to be less
than the length) and the length is 7 cm,
hence (D).

2 1978 I.8 (16%)


Substituting y = x2 into 2x − y + 1 = 0 gives 2x − x2 + 1 = 0. Therefore
x2 − 2x − 1 = 0,
hence (B).

3 1981 J.21 (47%), I.12 (70%)


Alternative 1
We have ab × bc × ac = 12 × 20 × 15 = (4 × 3) × (4 × 5) × (3 × 5).
Therefore a2 b2 c2 = 32 × 42 × 52 , i.e. abc = 3 × 4 × 5 = 60,
hence (C).
Alternative 2
ab a 12 3
= = = . Also ac = 15 and a is positive. Thus a = 3 and
bc c 20 5
c = 5. Since bc = 20, b = 4. Thus abc = 3 × 4 × 5,
hence (C).

4 1983 S.18 (66%)


Adding the first, third and fourth equations, and subtracting the second,
one obtains x − y + z + w = 4b,
hence (D).

5 1982 I.30 (6%), S.30 (6%)


Suppose Tina and Louise are now aged T and L years, respectively.
Suppose the time referred to in the question when ‘Tina was two years
154 Algebra Solutions

older . . .’ was x years ago (x > 0). The information about age in three
years time yields T + 3 = 4(L − x), i.e.

T = 4L − 4x − 3. (1)

If we look at x years ago we have T − 2 = L + x, i.e.

T = L + x + 2. (2)

Equating (1) and (2) gives 4L − 4x − 3 = L + x + 2, i.e. 3L = 5x + 5 or

5(x + 1)
L= . (3)
3
Since L is an integer, the numerator of (3) must be divisible by 3, so x
must be one of x = 2, 5, 8, . . .. Using equations (2) and (3):

if x=2 L=5 and T = 9;


if x=5 L = 10 and T = 17;
if x=8 L = 15 and T = 25;
if x = 11 L = 20 and T = 33; etc.

As Louise is known to be a teenager, Tina must be 25,


hence (C).

EQUATIONS: SURDIC
1 1982 S.17 (17%)
Alternative 1
√ √
Since
√ √ 7x −
√  3x √
= 4 =√7 −3√
we have
√  √ √ √
x 7− 3 = 7− 3 7 + 3 , i.e. x = 7 + 3 or
√ √
x = 7 + 3 + 2 21 = 10 + 2 21,

hence (A).
Alternative 2
√ √ √
Since 7x − 3x = 4 by squaring we have 7x + 3x − 2 21x2 = 16.
Division by 2 then gives

5x − 8 = 21x2 . (1)
Equations: Surdic 155

By squaring (1) we obtain 25x2 − 80x + 64 = 21x2 and by dividing this


by 4 we obtain x2 − 20x + 16 = 0. Therefore
√ √
20 ± 400 − 64 20 ± 4 21 √
x= = = 10 ± 2 21.
2 2
√ √
Now note that if x = 10 − 2 21 = 10 − 84, then 0 < x < 1 and
−8 < 5x − 8 < −3. Thus if we compare with equation (1), the left-hand
side is negative
√ while the right-hand side√is not. Consequently,
x = 10 − 2 21 is invalid and x = 10 + 2 21 is the required solution,
hence (A).

2 1979 S.20 (29%)


Alternative
√ 1 √
x+1+ x−1
Since √ √ = 3 we have
√ + 1 − x −√1
x√ √ √ √
x + 1 + x − 1 = 3 x + 1 − 3 x − 1. Therefore 4 x − 1 = 2 x + 1,
5
i.e. on dividing by 2 and squaring, 4(x − 1) = x + 1, i.e. 3x = 5 or x = ,
3
hence (E).
Alternative 2
a c a+b c+d
Using = implies = (Prove!) we have
b d a−b c−d
√ √
x+1+ x−1 3
√ √ = ,
x+1− x−1 1

i.e. √
2 x+1 3+1 4
√ = = .
2 x−1 3−1 2
√ √ 5
Therefore x + 1 = 2 x − 1, i.e. x + 1 = 4x − 4 or x = ,
3
hence (E).
Note: This question could have been made more difficult by giving ‘none
of these’ as a possible answer. In that case the answer x = 53 would have
to be verified to show that it was indeed a solution and not merely
introduced by the operation of squaring
√ both sides. Note also that any
solution must satisfy x ≥ 1, since x − 1 is not otherwise a real number.
156 Algebra Solutions

5
Verifying that x = 3 is indeed a solution:
  √ √
8/3 + 2/3
8+ 2
LHS =   =√ √
8/3 − 2/3 8− 2
√ √ √
2 2+ 2 3 2
= √ √ = √ = 3 = RHS.
2 2− 2 2

EQUATIONS: TRIGONOMETRIC
1 1981 S.13 (4%)

A graph clearly shows that the curve y = sin x and the straight line
x
y= have exactly 3 points of intersection.
360

 .......................... y = 360  ............
y x

.........
.....
... 
..... ...
. .... ....
.... .
...
... ....
 ....
....
...
 ........
... .. ...
... ... .. ..
.
... 
... . .
.. ...
.. 
...
..

.. ... ..
 ....
...
... ... ..
.. 
...
...
.
. .
..
..
.
..
..
.
x
....... ...
.
...
. .....
...
. ...
.
...
.
... . ... .
.... ... .... ...
....... ......... ...... .........
..........
.........
y = sin x◦

x
Therefore the equation sin x◦ = has 3 solutions,
360
hence (C).

2 1983 S.20 (6%)


1
For 2 sin θ − 1 = 0, i.e. sin θ = , there are the roots 30◦ and 150◦ . For
√ 2√
3 3
sin 2θ + = 0, i.e. sin 2θ = − , 2θ can be 240◦ , 300◦ , 600◦ , 660◦
2 2
(noting that 0◦ ≤ θ ≤ 360◦ , i.e. 0◦ ≤ 2θ ≤ 720◦ ) and thus θ can be 120◦ ,
150◦ , 300◦ or 330◦ . The root 150◦ is repeated. Thus the number of
distinct roots is 5,
hence (D).
Exponents 157

3 1980 S.23 (23%)


sin kx 1
Since   = cot x − cot x we have
1 4
sin x (sin x)
4
x x x
sin kx = cot sin sin x − cot x sin x sin ,
4 4 4
x x
= cos sin x − cos x sin ,
4 x
4
= sin x − ,
 4
3
= sin x .
4

3
Thus k = will suffice,
4
hence (C).

EXPONENTS
1 1980 J.11 (50%), I.6 (78%)
√ √ √
If a = 1, b = 2, c = 2, then a2 + b2 + c2 = 1 + 4 + 4 = 9 = 3,
hence (C).

2 1981 I.11 (27%), S.5 (79%)


If a = 2 and b = 3, then
 −1  −1
 −a −1  −2 −1 1 1 3 8
2 + 2−b = 2 + 2−3 = + = = ,
4 8 8 3
hence (B).

3 1980 S.9 (93%)


1
− 12
√ 1 2
If x = 4, y = 9 then x y2 = 4× √ = ,
9 3
hence (E).
158 Algebra Solutions

4 1984 S.6 (60%)


9m − 3m 3m (3m − 1)
= = 3m − 1,
3m 3m
hence (E).

5 1983 I.19 (47%)


x 2
Since 2 ∗ x = 2 + x = 100, it suffices to search for powers of 2 among
the numbers 100 − x2 , where x is an integer. From the table

x 0 1 2 3 4 5 6 7 8 9 10
100 − x2 100 99 96 91 84 75 64 51 36 19 0

It is clear that only x = 6 satisfies the requirements and we see


26 + 62 = 100,
hence (E).

6 1980 I.26 (16%), S.19 (72%)


 3n      2   2
2 + 2−3n 23n − 2−3n = 23n − 2−3n = 26n − 2−6n ,
hence (A).

7 1979 I.25 (9%), S.13 (64%)


6b
 
2b 3 3
2a −4=2 a − 4 = 2(5) − 4 = 246,
hence (B).

8 1980 S.24 (29%)


Rearranging and rationalising,
 √ 
1 7 1 7 3+ 2 √ √ √
x =
3 √ −8 =
6 − 2 = 3 + 2 − 2.
3− 2 7

Therefore x = (3)3 = 27,


hence (D).
Expressions 159

EXPRESSIONS
1 1984 I.2 (67%)
2x + 1 − (x − 3) = 2x + 1 − x + 3 = x + 4,
hence (E).

2 1983 I.1 (68%), S.1 (87%)


5x − 2(4 − x) = 5x − 8 + 2x = 7x − 8,
hence (A).

3 1980 J.13 (36%), I.4 (76%), S.3 (92%)


6(3 − x) − 2(1 − x) = 18 − 6x − 2 + 2x = 16 − 4x,
hence (C).

4 1982 S.5 (43%)



The perfect square n is the square of n. The next perfect square is
√ 2 √
( n + 1) = n + 2 n + 1,
hence (E).

5 1980 J.12 (64%)


Twice a given number n is 2n. Three less than this is 2n − 3,
hence (E).

6 1979 J.13 (62%)

Total number of nails


= (number of nails in a box) × (total number of boxes),
= (number of nails in a box) × (number of boxes in carton)
× (number of cartons on truck)
= (z)(y)(x) = xyz,

hence (E).
160 Algebra Solutions

EXTREME VALUE
1 1979 I.15 (23%)
2 2 2
x + 4x + 1 = (x + 4x + 4) − 3 = (x + 2) − 3 ≥ −3 with equality when
x = −2,
hence (E).

2 1978 I.27 (28%), S.22 (37%)


2 2
Since (a − b) ≥ 0, 6ab = 48 − (a − b) ≤ 48, so ab ≤ 8. When a = b,
(a − b)2 = 0, 6ab = 48, ab = 8,
hence (D).

3 1984 I.25 (33%)


The value 1 is achievable in the case a = 1, b = 2 and c = 3 (say). The
following argument shows that 1 cannot be exceeded. The maximum
value achievable by the numerator is 24 (e.g. the case a = 7, b = 8,
c = 9). So we need only to consider the cases where the denominator is
less than 24. These are listed as follows:

a b c a+b+c abc
1 2 3 6 6
4 7 8
5 8 10
6 9 12
7 10 14
8 11 16
9 12 18
1 3 4 8 12
5 9 15
6 10 18
7 11 21
1 4 5 10 20

In these cases a + b + c < abc and in all other cases abc ≥ 24 while
a + b + c ≤ 24,
hence (D).
Factorisation 161

4 1982 S.21 (41%)


Since (p + q) − (p − q) > 29, then p + 2pq + q − (p − 2pq + q 2 ) > 29,
2 2 2 2 2
1
i.e. 4pq > 29 or pq > 7 . As p and q are positive integers with p > q:
4
p largest value of pq
1 −
2 2×1=2
3 3×2=6
4 4 × 3 = 12

So 4 is the smallest possible value of p,


hence (D).

FACTORISATION
1 1981 S.2 (89%)
n3 − n n(n2 − 1)
= = n2 − 1 (n = 0),
n n
hence (C).

2 1979 S.2 (74%)


2 2
x + y cannot be factorised,
hence (D).
Note:
(A) x2 + xy = x(x + y) (B) x2 − y 2 = (x − y)(x + y)
(C) y 2 + xy = y(x + y) (E) 2x + 2y = 2(x + y)
162 Algebra Solutions

3 1979 J.27 (11%)


......
......
......
From the given information we know ei- ......
......
......
......
ther p − q = 1 or q − p = 1. As the hy- ......
......
......
...... p
x ......
potenuse (opposite the right angle) is the ......
......
......
......
longest side the former is true. Let the ......
......
......
......
......
remaining side have length x cm. Then, .......
..
......
....
q
by Pythagoras,

x2 = p2 − q 2 = (p − q)(p + q)
= p + q, since p − q = 1.

Therefore x = p + q,
hence (B).

4 1978 S.26 (24%)


8
Since x3 + x − 8 = 2 , we have x5 + x3 − 8x2 − 8 = 0, i.e.
x
x3 (x2 + 1) − 8(x2 + 1) = 0, i.e. (x3 − 8)(x2 + 1) = 0 and since x2 + 1 > 0
this gives x3 − 8 = 0 or x = 2 as the only real solution,
hence (B).

FRACTIONS
1 1983 I.6 (23%), S.2 (63%)
1 xy − 1
x−
y y x
= = ,
1 xy − 1 y
y−
x x
hence (A).

2 1984 S.3 (64%)


1 1 1 1 1 1 F −H
Since = − we have = − = . Therefore
F H G G H F FH
FH
G= ,
F −H
hence (B).
Functions 163

3 1978 I.26 (14%), S.19 (38%)


     
1 1 1 1 1 2 3 n−1 1
1− 1− 1− ... 1 − = × × ×···× = ,
2 3 4 n 2 3 4 n n
after cancellation,
hence (A).

FUNCTIONS
1 1979 S.5 (49%)
x+1 x x x x
f (x + 1) − f (x) = 4 − 4 = 4 × 4 − 4 = (4 − 1)4 = 3f (x),
hence (D).

2 1978 J.22 (17%)


 
1 2 2 2 2 1 1
f (a) ÷ f = ÷ = ÷ 2a = × = 2,
a a 1 a a 2a a
a
hence (A).

3 1978 I.22 (15%), S.15 (36%)


f (n) = (n − 1) × f (n − 1) and f (1) = 1. Therefore

f (4) = 3 × f (3) = 3 × 2 × f (2) = 3 × 2 × 1 × f (1) = 6,

hence (E).

GRAPHS
1 1981 S.3 (92%)
As the line has a constant (and non-zero) slope the car is travelling at a
constant (and non-zero) speed,
hence (D).

2 1978 I.13 (12%), S.7 (70%)


2
The double zero at x = 1 indicates (x − 1) is a factor. The zero at
x = −2 indicates (x + 2) is a factor. Then to distinguish between (D)
and (E) either
164 Algebra Solutions

(i) note that from the overall shape of the graph the coefficient of x3 is
positive, or
(ii) substitute x = 0, requiring that y = 2,
hence (E).

3 1984 S.11 (19%)


Alternative 1
The correct answer may be deduced by first sketching 
a graph of the
expression f (x) = x(x2 − 1) and then sketching y = ± f (x), where
f (x) is positive. Since f (x) = x(x − 1)(x + 1):

 ..
....
..... ..... ....... ..
.. ....
.. .
.
. ..
.. .................... ..... .. ...
... ......
..... ..... . ... ...
.... ..... ... .
......
..
...
−1 ......
.....
.......... ..
....
.
.... 
..
.. . ......... .. ... 1
.. .... ... ...... .
..
..... ..
.
. .. .
..................... ...
..
.. ... ..
... .... .... ...
.. . ..... ....... ..
..
y=f (x) . √
y=± f (x)

hence (A).
Alternative 2
The following is an indirect proof, eliminating the distractors. First,
y = 0 for x = 0, −1 and +1 so this eliminates (E). Secondly, the graph
must be symmetric about y = 0 because of the y 2 term. This eliminates
(C). Finally, there must be values of y in −1 ≤ x ≤ 0 because this gives
positive values of y 2 . There can be no values of y in 0 < x < 1 because
y 2 would then be negative. Of the remaining three possibilities, only (A)
can now be valid,
hence (A).

INEQUALITIES
1 1981 J.9 (70%)
Alternative 1
n − 6 < n − 5 < n − 1 < n + 1 < n + 4,
hence (B).
Inequalities 165

Alternative 2
Substituting a particular value for n, say n = 10 gives: n + 1 = 11,
n − 1 = 9, n − 6 = 4, n − 5 = 5, n + 4 = 14. The middle ranking number
is 9 or n − 1,
hence (B).

2 1983 J.11 (27%)


By the triangle inequality, the sum of any two sides must be greater
1 1
than the third. Hence 7 + x > 11, i.e. x > 3 . The smallest whole
2 2
number with this property is 4,
hence (B).

3 1978 S.3 (55%)


Alternative 1
y

... x2..−1
..
(i) If x ≥ 0, |x| = x and .
...
..
...
...
..
(|x| − 1)(1 + x) = (x − 1)(x + 1) > 0 ..
..
. ..
..
.
..

if x > 1 or x < −1. Therefore the .


....
......−1
. ..
.
..... . ........
..
.... .
..
..
. 
... x
...
... ... .... .. 1
solution is {x : x > 1}. ..
.
... .. ...
..... .... ..........
.. ............
...
... ...
... .
.. ...
(ii) If x < 0, |x| = −x and .. ..−(x+1)2
...
(|x| − 1)(1 + x) = −(x + 1)2 ≤ 0, for ..

all real x. Therefore


(|x| − 1)(1 + x) > 0 if and only if
x > 1.

hence (A).
Alternative 2
A product is positive if and only if both factors are positive or both are
negative. The first factor is positive if |x| > 1, i.e. if x > 1 or x < −1,
and negative if −1 < x < 1. The second factor is positive if x > −1, and
negative if x < −1. Therefore both factors are positive if x > 1, while
there is no value of x for which both factors are negative,
hence (A).
166 Algebra Solutions

4 1983 J.15 (14%), I.11 (27%), S.3 (75%)


The statement must be true for all p between 0 and 1. Thus it must be
1
true for p = . (We choose a value like this because it is then simple to
√ 4
evaluate p.) Testing
1 1
(A) > FALSE
4 2
1
(B) 4> TRUE
2
1
(C) >4 FALSE
4
1 1
(D) > FALSE
64 16
1 1
(E) > FALSE
64 4
hence (B).

5 1978 J.18 (16%)


If a > 0 and b < 0, then −b > 0 and a − b > 0,
hence (C).
Note: Alternatives A and B are sometimes false while Alternatives D
and E are always false.

6 1980 I.19 (19%)


Since a > b, then a + c > b + c,
hence (D).
Note: Counter examples can be found for each of the other propositions.
1 1
(A) > if b is positive (and not defined if a or b is zero).
a b
(B) ac > bc if c ≤ 0.
(C) a2 > b2 if b < 0 and |b| ≤ |a|.
1 1
(E) < if a is negative (and not defined if a or b is zero).
a b
Inequalities 167

7 1980 J.19 (35%), I.7 (52%)


Alternative 1
The question implies a unique answer. Choosing a particular value of x,
say x = 10,
5 5 5 5 5 5 x 10 x + 1 11
= , = , = , = , = .
x 10 x + 1 11 x − 1 9 5 5 5 5
5
Of these, is the smallest,
11
hence (B).
Alternative 2
Comparing the first three alternatives, x + 1 > x > x − 1, so
x+1 x x−1 5 5 5 5 5
> > , or < < . Also < (since
5 5 5 x+1 x x−1 x+1 6
x > 5). Now comparing the last two alternatives, 5 < x < x + 1, so
x x+1 5 x
1< < . Thus likely answers are or . But
5 5 x+1 5
5 5 x
< <1< ,
x+1 6 5
hence (B).

8 1980 J.21 (27%)


We know that 8 > 4 and that it is valid to add a number to both sides of
an inequality. Therefore 8 + c > 4 + c,
hence (D).
Note:

(A) breaks down when c ≤ 0.


(B) breaks down when c ≥ 0.

(C) breaks down when c = 0.

(E) breaks down when c ≤ −1.

9 1979 S.8 (54%)


x − y > x implies −y > 0, i.e. y < 0. x + y < y implies x < 0,
hence (D).
168 Algebra Solutions

10 1979 S.17 (53%)


Alternative 1
From the sketch it can be seen that (x + 1)(x − 3)(x − 5) > 0 when
−1 < x < 3 or x > 5.
y

................................
...... ......
..... ......
..... ......y=(x+1)(x−3)(x−5)
....
. .....
.....
...
. ..... .
...
..
. ..... ..
..... ...
−1...
...
..
. .....
......
...... .
....
.
....
. 
... 3 .............................................. 5 x
...
...
...
...
...

Of the given alternatives only −1 < x < 2 does not violate the complete
solution,
hence (E).
Alternative 2
y = (x + 1)(x − 3)(x − 5) can change sign only at x = −1, +3 or +5. For
x = −2, say, y = negative × negative × negative = negative. So y < 0 for
x < −1. Similarly we have y > 0 for −1 < x < 3, y < 0 for 3 < x < 5
and y > 0 for x > 5. Thus y > 0 if −1 < x < 2, as before,
hence (E).

11 1978 I.21 (14%), S.14 (39%)


2x2 − 3x + 4 y
> 1. 
.. y=(x−2)(x−1) .
x2 + 2 ...
...
...
...
...
...
Therefore 2x − 3x + 4 > x2 + 2 (since
2 ...
...
... .
...
...
... ...
x2 + 2 > 0 for all real x). Thus ...
...
... ..
..
..
...
x2 − 3x + 2 > 0, i.e. (x − 2)(x − 1) > 0, ...
...
... ..
...
... ....
i.e. x < 1 or x > 2, ...
... ..
...
.

hence (A).
...
....
.....
....... .
.
...
.....
...

..
1 .................... 2
. x
Inequalities 169

12 1980 I.27 (10%), S.21 (47%)


...
y

..
.........
..
....... y= 1
.
4 .............................................. x−3
1 .... ......
(i) When x = 3, is not defined. .. ........
....
x−3 .. ... ...
.. ..
..... .... ......
.. ..........
(ii) If x > 3, x − 3 > 0, thus ....................................
.. ....
.. ..
................

.........................
.........
....... 3 .... 3 1 x
.... . 4
1 ...
.. ...
.. .
< 4 ⇒ 1 < 4x − 12 ..
.. ..
x−3 .. ..
..
...
... ..
⇒ 4x > 13 .. ..
...
......
1 ....
...
⇒ x>3 , ......
.
4

1
giving a partial solution of {x : x > 3 }.
4
(iii) If x < 3, x − 3 < 0, thus
1
<4 ⇒ 1 > 4x − 12
x−3
⇒ 13 > 4x
1
⇒ x<3 ,
4
giving a partial solution of {x : x < 3}.
 
1
Thus the complete solution is {x : x < 3} ∪ x : x > 3 , i.e. all values
4
1
of x except 3 ≤ x ≤ 3 ,
4
hence (B).
170 Algebra Solutions

LOGARITHMS
1 1978 S.1 (40%)
 
(log 125)
−a = log2 (log16 2) 5
  
3
1
= log2
4
 −6 
= log2 2
= −6,

hence (D).

2 1978 S.27 (41%)

99 999x + 1 1
y= ⇒ ≤ y ≤ 100 (if 0 ≤ x ≤ 1)
1000 1000
⇒ −3 ≤ log10 y ≤ 2
2
⇒ (log10 y) ≤ 9,

hence (B).
Note: Maximum value achieved when x = 0.

3 1979 S.14 (14%)


The expression can be rearranged and factorised as a quadratic,

9(log x)2 − 12(log x)(log y) + 4(log y)2 = 0


2
thus (3 log x − 2 log y) = 0
thus 3 log x = 2 log y
thus log x3 = log y 2
thus x3 = y2 ,

hence (A).

4 1983 S.16 (17%)


Note: f (x) = log2 x and g(x) = 2x are inverse functions, so
log2 (2x ) = x. The key is thus to arrange the argument of log2 in the
form 2x for some x.
Operations 171

Alternative 1
p = 14 = 2−2 , thus from the definition, log2 p = −2. Therefore

1 1
−p log2 p = − (−2) = ,
4 2
hence (E).
Alternative 2
 
1 1 1   1 1
− log2 = − log2 2−2 = − (−2) = ,
4 4 4 4 2

hence (E).

OPERATIONS
1 1978 J.20 (7%), I.9 (16%), S.6 (62%)
 
1 bc
a ∗ (b ∗ c) = a ∗ = ,
bc a
hence (C).

2 1979 J.26 (11%), I.18 (42%)


1 1 1 1 1 1
∗6= 1 + + ×6=3+ +2=5 ,
3 3
6 3 6 6
hence (C).

3 1980 J.27 (18%), I.18 (41%)


Using the definition of the operator ∗, 5 ∗ x = 5x − 5 + x = 6x − 5. Now
22 2
5 ∗ x = 17, so 6x − 5 = 17, i.e. 6x = 22, i.e. x = or 3 ,
6 3
hence (E).
172 Algebra Solutions

PERCENTAGES
1 1978 J.14 (9%), I.5 (21%)
Suppose the first square has side length 2l, so that the second square has
side length 3l. Then the increase in area is (3l)2 − (2l)2 = 5l2 . Therefore

increase in area 5l2


percentage increase = × 100 = 2 × 100 = 125,
original area 4l

hence (D).

2 1981 J.16 (17%)


Let the original square have size 10x cm × 10x cm. Thus the original
area is 10x × 10x = 100x2 cm2 . The new area is 9x × 11x = 99x2 cm2 .
This is a 1% decrease,
hence (C).

3 1978 S.9 (35%)


Suppose the cube initially has side length 5l, so that the second cube
has side length 8l. Then the increase in surface area is 6(8l)2 − 6(5l)2 ,
i.e. 6 × 39l2 . The percentage increase is

6 × 39l2
× 100 = 39 × 4 = 156,
6 × 25l2
hence (C).

4 1978 J.19 (19%)


The
 new production
 is (100 + n)% of m, i.e.
100 + n  n 
×m=m 1+ ,
100 100
hence (D).

5 1980 I.15 (10%)


 
100 + a
Suppose the initial wage is x dollars. Then x = 250, i.e.
  100
100
x= × 250,
100 + a
hence (D).
Polynomials 173

6 1979 S.15 (51%)


If the original price is x, then the

discount = 0.95 × 0.9 × 0.8x (1)


= 0.684x.

So the discount is 0.316x, equal to 31.6%,


hence (A).
Note: From (1) it can be seen that the price is independent of the order
of discounts.

POLYNOMIALS
1 1978 S.23 (18%)
Alternative 1
We note that

(1 + x)5 = 1 + 5x + 10x2 + 10x3 + 5x4 + x5

Then
(a0 + a1 )(a1 + a2 )(a2 + a3 )(a3 + a4 )(a4 + a5 )
a0 a1 a2 a3 a4 a5

6 × 15 × 20 × 15 × 6
=
1.5.10.10.5.1
6×3×3×6
=
5
6 × 3 × 3 × 6 × (4 × 3 × 2 × 1)
=
5 × (4 × 3 × 2 × 1)

65
= .
5!

hence (C).
174 Algebra Solutions

Alternative 2
Consider the expression
     
1 a0 a1 a2 a3 a4
E= +1 +1 +1 +1 +1 .
a0 a1 a2 a3 a4 a5
Now, using binomial coefficients,
ai−1 C5 5! i!(5 − i)! i
= i−1
5 = . = .
ai Ci (i − 1)!(6 − i)! 5! 6−i
Therefore
     
1 2 3 4 5
E = +1 +1 +1 +1 +1 (since a0 = 1)
5 4 3 2 1

6 6 6 6 6
= · · · ·
5 4 3 2 1
65
= .
5!

hence (C).
Note: The above method generalises to the expansion of (1 + x)n . Here
ai−1 i
= ,
ai n+1−i
and hence the expression corresponding to E is
(n + 1)n
.
n!

2 1978 S.30 (6%)


Let Q(x) be a polynomial of degree 98. Then
x100 = (x2 + 3x + 2)Q(x) + Ax + B where A and B are real
= (x + 2)(x + 1)Q(x) + Ax + B.

Substituting,
(i) x = −1, (−1)100 = −A + B, that is, − A + B = 1,
(ii) x = −2, (−2)100 = −2A + B, that is, − 2A + B = 2100 .
Progressions 175

Solving, B = 2 − 2100 ,
hence (B).

3 1984 S.16 (9%)


99
There will be 100 individual terms in x , having as their coefficients
−1, −2, . . . , −100. Grouping them together to give one term, the
required answer is −1 − 2 − · · · − 100 (an arithmetic progression, 100
terms, first term = −1, last term = −100), giving
100
(−1 − 100) = 50(−101) = −5050,
2
hence (A).

PROGRESSIONS
1 1979 S.16 (26%)
The weights, in kilograms, constitute an arithmetic progression of first
term (a) equal to 1 and last term () equal to 0.1. Letting Sn denote the
sum of n terms, the formula for the sum is
n
Sn = (a + ) .
2
Hence
n
11 = (1 + 0.1),
2
i.e. n = 20,
hence (B).

2 1980 S.25 (26%)


Recall that the formula for the sum Sn of n terms of an arithmetic series
is
n(a + )
Sn = ,
2
where a is the first term and  is the last. Thus
     
1 1 2 1 2 3 1 2 3 4
+ + + + + + + + + +
2 3 3 4 4 4 5 5 5 5
 
1 99
··· + + ··· +
100 100
176 Algebra Solutions

1 1 2.3 1 3.4 1 4.5 1 99.100


= + . + . + . + ··· + .
2 2 3 2 4 2 5 2 100
1
= (1 + 2 + 3 + 4 + · · · + 99)
2
 
1 99.100
=
2 2

= 2475,

hence (B).
Note: This question was also set as 1984 I.21 and S.18, with different
alternatives, yielding correct response rates of 10% and 30% respectively.

3 1981 J.4 (71%)


Alternative 1
1 + 3 + 5 + 7 + 9 + 11 + 13 + 15 + 17 + 19 = 100,
hence (A).
Alternative 2
The addition can be simplified by grouping

1 + 3 + 5 + 7 + 9 + 11 + 13 + 15 + 17 + 19
= (1 + 19) + (3 + 17) + (5 + 15) + (7 + 13) + (9 + 11)
= 5 × 20
= 100.

hence (A).
Alternative 3
This is the sum of an arithmetic progression of n = 10 terms, first term
a = 1 and last term  = 19. The formula for the sum gives
n 10
Sn = (a + ) = (1 + 19) = 5 × 20 = 100,
2 2

hence (A).
Ratio 177

4 1983 S.11 (39%)


This is an arithmetic series with first term a to be determined. The
number of terms is n = 101. The common difference d equals 2, since
the terms are consecutive odd numbers. The formula for the sum is
n
Sn = (2a + (n − 1)d) ,
2
which translates, in this case, to
101
12 827 = (2a + 200),
2
i.e.
2a + 200 = 2 × 127 = 254,
giving a = 27,
hence (B).

5 1984 J.20 (46%), I.14 (57%), S.8 (78%)


For row n,

sum = 5 × the middle term


= 5(3 + 5(n − 1))
= 5(5n − 2)
= 25n − 10.

The sum nearest 150 is when n = 6,


hence (B).

RATIO
1 1978 J.17 (8%)
If we denote distance by d, average speed by v and time by t, we have
d = vt, so
d = vA tA = vB tB .
Since vA = 32 vB we have

3
v B tA = v B tB .
2
178 Algebra Solutions

Therefore
tA 2
= ,
tB 3

hence (B).

2 1984 J.11 (47%), I.6 (78%)


Alternative 1
Let Barabara’s share be $x. Then
x
+ x + 2x = 60.
3
10x
This gives = 60 or x = 18,
3
hence (E).
Alternative 2
3 3
Barbara receives = of the total,
1+3+6 10
hence (E).

RECURSION RELATIONS
1 1980 S.15 (34%)
Using the ‘recurrence formula’ f (x + 1) = 2f (x),

f (7) = 2f (6) = 2(2f (5)) = 4(2f (4)) = 8(2f (3)) = 16(2f (2))
= 32(2f (1)) = 64f (1) = 320

(since f (1) = 5),


hence (E).

2 1979 S.19 (16%)


x2n n
− xn−1 × xn+1 = (−2) , n ≥ 1, x0 = x1 = 1.
When n = 1, x21 − x0 × x2 = (−2)1 , i.e. 1 − x2 = −2, i.e x2 = 3.
When n = 2, x22 − x1 × x3 = (−2)2 , i.e. 9 − x3 = 4, i.e. x3 = 5,
hence (D).
Recursion Relations 179

3 1981 S.21 (17%)


Considering the five suggested formulae in order to find f (2):

If f (2n) = 2n2 , f (2) = 2.


If f (2n) = 4n2 , f (2) = 4.
If f (2n) = n2 , f (2) = 1.
If f (2n) = n2 + 1, f (2) = 2.
If f (2n) = 2n, f (2) = 2.

But f (n) = f (n − 1) + 2n − 1. So f (2) = f (1) + 4 − 1 = 1 + 3 = 4. The


only formula consistent with this is f (2n) = 4n2 ,
hence (B).
Note: Had ‘none of these’ been a distractor in this question it would
have been necessary to verify the formula f (2n) = 4n2 by induction.

4 1980 S.28 (50%)


We note that

f (4) = f (2 × 2)
= f (2) × f (2) (by (iii))
= 2×2 (by (ii))
= 4.

Now,

4 > 3 > 2 ⇒ f (4) > f (3) > f (2) (by (iv))


⇒ 4 > f (3) > 2
⇒ f (3) = 3, (by (i))

hence (B).
Note: It can also be shown that f (1) = 1, and that f is the identity
function, i.e. f (n) = n for all natural numbers n.
180 Algebra Solutions

SUBSTITUTION
1 1978 I.1 (77%)
a − 2b 2 − 2(−3)
= = −8,
a+b 2 + (−3)
hence (C).

2 1981 J.10 (78%)


2
If y = x + 2x + 3 and x = 3 then y = 9 + 6 + 3 = 18,
hence (D).

3 1981 I.3 (78%)


1 1
If a = 10 and t = 3 then at2 = × 10 × 9 = 45,
2 2
hence (A).

4 1982 J.11 (47%)


xy 0.4 × 1.1 0.44 4.4
If x = 0.4, y = 1.1, z = 0.2, then = = = = 2.2,
z 0.2 0.2 2
hence (C).

SURDS
1 1982 S.4 (92%)
Alternative 1
√ √ √ √
(3 + 5)(6 − 2 5) = (3 + 5)(3 − 5)2 = 2(9 − 5) = 8,
hence (D).
Alternative 2
√ √ √ √ √
(3 + 5)(6 − 2 5) = 3(6 − 2 5) + 5(6 − 2 5)
√ √
= 18 − 6 5 + 6 5 − 10
= 8,

hence (D).
Surds 181

2 1978 J.26 (6%)


To find the order of magnitude raise all numbers to the sixth power:
 √ 6 √ 6
16 = 1, 26 = 64, 5 = 125, 36 = 243,
3
9 = 81,

hence (A).
GEOMETRY SOLUTIONS

ANGLES
1 1981 J.6 (75%)
Size of remaining angle = 180◦ − (45◦ + 55◦ ) = 80◦ ,
hence (C).

2 1984 J.5 (45%)

The other internal angles of the triangle P QR . P


......
are 60◦ and 50◦ . Therefore ... ....
... ...
.. ◦ ....
.
... x...
x = 180 − 60 − 50 = 70, ...
...
...
...
...
... ...
hence (E). .
.
..
.
.. ...
...
...
... ...
... ...
..
. ...
.
. ...
◦ ....
130◦
. ...
120 ...
...
.
Q R

3 1982 J.8 (73%)


◦ ◦ ◦ ◦
 P QR = 180 − (75 + 90 ) = 15 ,
hence (E).

4 1982 J.5 (82%), I.2 (95%)


 P T S = 180◦ − (70◦ + 30◦ ) = 80◦ ,
hence (E).

5 1981 J.8 (57%), I.2 (84%)


x + x + 140 = 360. Therefore 2x = 220, i.e. x = 110,
hence (C).

6 1983 J.9 (35%)


...
◦ ◦ ◦ ...
The exterior angle x = 35 + y and y = ..
x.......◦..................
180 − 107 = 73. Therefore x = 35 + 73 = 108, ....
.....
...
...
...... ...
......
hence (D). .
.....
.
..
......
...
...
...
..... .
...... ◦......
.....
.....
...... 35 ◦ y ...
. 107◦
Angles 183

7 1979 J.4 (44%)



4x + 5x = 180 (adjacent angles on a straight line). Therefore
9x = 180◦ , i.e. x = 20◦ , i.e. 5x = 100◦ ,
hence (D).

8 1978 J.4 (56%)


y = x = 180 − 150 = 30. Thus x + y = 60,
hence (C).

9 1980, J.4 (84%), I.2 (92%)


Since QOR and P OR are adjacent angles on a straight line,
 
 QOR +  P OR = 180◦ , i.e.  P OR = 180◦ − 38◦ = 142◦ ,
hence (B).

10 1983 I.2 (92%)


Since lines l and m are parallel, opposite and corresponding angles give
x = 40,
hence (E).

11 1984 J.14 (44%), I.9 (53%), S.4 (71%)


Alternative 1
The internal angles of the quadrilateral are (180 − x)◦ , 105◦ , 90◦ and
65◦ . We obtain 180 − x + 105 + 90 + 65 = 360, i.e. x = 440 − 360 = 80,
hence (C).
Alternative 2
A line rotating through all the exterior angles of any (convex) polygon
would turn one full revolution, or 360◦ . Hence adding the external
angles of the given figure gives x + 90 + 115 + 75 = 360, i.e. x = 80,
hence (C).

12 1982 J.17 (24%)


Using the result that the exterior angle of a triangle is equal to the sum
of the two opposite interior angles, 5x = x + 72, i.e. 4x = 72, i.e. x = 18.
Therefore  QP S = 5x◦ = 90◦ ,
hence (A).
184 Geometry Solutions

13 1981 I.16 (20%), S.11 (33%)



On a clock the minute hand travels 360 per hour and the hour hand
360◦
= 30◦ per hour. At 12:35 the angle between the hands is
12
35 35 35 1◦
× 360◦ − × 30◦ = × 330◦ = 192 .
60 60 60 2
1◦ 1◦
Thus the smaller angle is 360◦ − 192 = 167 ,
2 2
hence (A).

14 1980 J.23 (17%)


One full revolution of the back wheel occurs when the chain passes over
18 rear sprocket teeth. Since there are 48 teeth on the front sprocket
18 3
this chain movement corresponds to or of a revolution of the front
48 8
3
sprocket. Thus, the angle through which the pedals turn is of 360◦ or
8
135◦ ,
hence (A).

15 1980 I.20 (9%)

The triangles P QO and QOR are both ..........


......................... R
...... .............
......... ... .....
.....
isosceles. Let  QP O = x◦ . Since an ...... . .
... ......... .. .......
. . 2x....
.. ...
Q ......................
........ ..
..
.. r
...
...
exterior angle of a triangle is equal to r...................... .......2x
.......x
..........
..r
...
......... ..... 60◦
...
...
...
...
.. .. x ....
.
the sum of the opposite interior angles, P ... ...
... ... S
...
... O ..
.
...
 RQO = x◦ + x◦ = 2x◦ ; and  SOR = ...
... .
..
.
..... ....
...... .....
....... .....
 ORP +  OP R. Therefore 60 = 2x + x, ...................................

i.e. x = 20,
hence (E).
Angles 185

16 1982 S.10 (50%)

Let A, B, C be points as shown, where B is the


....
.....
point of contact with the ground. Produce CQ ...
....
....
through Q. The required angle is α. Now, OB = ... ..
... .. O
..... .. .
OQ = 10 and AB = QC = 5. Therefore OA = ... .......α 10................
... ...................α...
..... ......... .
.
OA ............. ....... ..........
. ........A ....
.
OB − AB = 10 − 5 = 5. Thus cos α = = Q ........... ... .....
..5 ..... ............ .... .....
OQ .
.
...........................
...
...

5 1 C B
= , i.e. α = 60◦ ,
10 2
hence (D).
17 1980 J.28 (17%)
......
◦ ....... .........
....... c
Let the remaining angle be y . Since the ............
.......
.....
.....
.....
.
.
....... .....
angle sum at a point is 360◦ , y = 360 − x. ........................ .............
..
........
.....
.....
.....
.
Since the interior angles of a quadrilateral .....x y
...
............
.......
...... ................
.....
.....
.....
.....
.......... ....... ..
sum to 360◦ , .......
....... b ...
...
.......
....... . ...
.
....... ..
....... ...
....... ...
...
y = 360 − (a + b + c)
.......
a
....... .....
.........

i.e. 360 − x = 360 − (a + b + c)


i.e. x = a + b + c,

hence (C).
18 1978 J.28 (19%), I.20 (38%)

In ABC, θ + (180 − 2α) + (180 − 2β) = B ............... X


..................
............... .... ..... ◦ α
180 so θ = 2(α + β) − 180. From BCZ, ............. .......
................... .. ..
..
............... ... ◦.....
α + β + 80 = 180, so α + β = 100. Hence A .......................
θ
..........
α
...
...
...
...
.......... ....
...........
..........
..
. ◦ ..............
.
β Z
BAC = θ = 2(100) − 180 = 20, .......... .
.
.......... .................. ◦
.
............... β
..........
...........
hence (B). C ...........
.....
Y
186 Geometry Solutions

19 1983 I.28 (5%), S.21 (16%)

Note that x + y + z = 180◦ and (x + x ) + ......


......
...
(y + y  ) + (z + z  ) = 3 × 180◦ = 540◦ . Since z  ........ z.......................
. ......
... ......
......
4 + 5 + 6 = 15 it follows that ...
....
......
......
......
.. ......
... ......
......
...
4 ....
x y
......
......
y
x (360◦ ) = 96◦
......
= ..
...
.....
15 ...
.. x
5
y = (360◦ ) = 120◦
15
6
and z = (360◦ ) = 144◦ .
15

Therefore x = 180◦ − 96◦ = 84◦ , y = 180◦ − 120◦ = 60◦ and


z = 180◦ − 144◦ = 36◦ . The desired ratio is 84 : 60 : 36 or 7 : 5 : 3,
hence (A).

AREA
1 1984 J.9 (32%), I.4 (64%)
The total area is the area of the 10 × 5 rectangle minus the area of a
1
triangle with base 3, altitude 6, i.e. 10 × 5 − × 3 × 6 = 50 − 9 = 41,
2
hence (C).

2 1978 J.9 (12%)


150 cm equals 1.5 m and 50 cm equals 0.5 m. Therefore the area of the
rectangle is (1.5 × 0.5) m2 = 0.75 m2 ,
hence (D).

3 1979 J.8 (25%)


250 cm equals 2.5 metres. Therefore the area of the square is
(2.5 × 2.5) m2 = 6.25 m2 ,
hence (A).

4 1980 S.6 (61%)


The unshaded triangle has base length 5 cm, height 5 cm and hence area:
1
× 5 × 5 = 12.5 cm2 . The area of the rectangle is 5 × 10 = 50 cm2 . Thus
2
Area 187

the shaded area is 50 − 12.5 = 37.5 cm2 ,


hence (A).

5 1978 S.9 (35%)


Suppose that the cube initially has side length of 5l, so that the second
cube has side length 8l. The increase in surface area is
6(8l)2 − 6(5l)2 = 6 × 39l2 . The percentage increase is
6 × 39l2
× 100 = 156,
6 × 25l2
hence (C).

6 1981 I.6 (22%)

P M Q
Area P M W Y = x2 , W
Y ..........................................................
.......................................
V
.......................................
therefore M W = x. .......................................
.......................................
.......................................
Area M QV W = 5x = x × W V .......................................
.......................................
.......................................
.......................................
therefore W V = 5. .......................................

S T R
Area Y W T S = 3x = x × W T
therefore W T = 3.

Therefore shaded area equals 5 × 3 = 15,


hence (A).

7 1983 I.9 (54%), S.4 (78%)

1
AreaP T W = AreaRV U = (2)(9) = 9 cm2 .
2
1
AreaQT U = AreaSV W = (3)(6) = 9 cm2 .
2
Therefore area TUV W = (12)(8) − (4)(9) = 96 − 36 = 60 cm2 .

hence (E).
188 Geometry Solutions

8 1982 J.13 (30%), I.9 (56%)

After placing the information on the dia- P 2 T.......... . . . . . . . . . . . . . ..... Q


. .
.. ...........................................
gram we note T Q = 7 − 2 = 5. Therefore ...
...
................................
....... . . . . . . . . . . .
... ...... .....................
1 ...
. ...... . . . . . . . . . .
........................
areaQRT = × 3 × 5 = 7 12 , .
..
.
.
.... ....... . . . . . . .
...... . . . . . . .
.................. 3
2 .
.
...
. ....... . . . .
...............
...... . . .
hence (E). .
..
.
..
.
.
. ..........
.......
.......
.. ...
..
S 7 R

9 1982 I.14 (21%), S.9 (60%)


By Pythagoras, 10 = y + (2y) = y + 4y . Therefore 100 = 5y 2 , i.e.
2 2 2 2 2
1 1
y 2 = 20. Therefore area = × base × height = × y × 2y = y 2 = 20,
2 2
hence (E).

10 1984 J.16 (17%), I.13 (33%)


. ......
.. ...... ......
... ...... ......
...... P
Enclosing the region by a 9 × 7 rectangle it Q P
..
... .
... ............
..
........
.
......
......
......
......
.. ..... ......
can be seen that the total area is .......................
..
......
.
R .
...
...
...
63 − (area of 3 triangles of base 2, .
...
..
...
...
height 3 (P )) .. ..
......
......
......
.......
−(area of 2 × 1 rectangle (Q)) ......
.
......
..
.
P
......
......
−(area of triangle of base 1,
height 3 (R))
 
1 1
= 63 − (3) (2)(3) − (2)(1) − (3)
2 2
= 63 − 9 − 2 − 1.5 = 50.5.

hence (B).
Area 189

11 1982 J.21 (30%), I.11 (51%)


z
Adding areas in the (x, y), (y, z) 
and (x, z) planes, respectively,
3

Area = (5 × 10 + 5 × 7 + 5 × 3) ....  3
..... 7

 
+2(3 × 3 + 7 × 10) ..
..
....

+(10 × 5 + 5 × 7 10 ...
7
..
+5 × 3) ...
....... ....... ....... ....... ....... ....... ....... ....... ....... ....... .. 
= 100 + 158 + 100 ..
..
....

5
y
....
x
= 358 cm2 , 
 10

hence (D).
12 1981 J.15 (35%)
 2  8  2 
Consider T U as the base of T U V .
The height is equal to QR (no P T
. . . . . . . ....
U.... . . . . . . . Q
. .. . . . . . . .
........................... ....................
.................... ....................
matter where V is placed on SR). ....................
......................
.... . . . . . . . .
..............................
.
...................... ......................
The shaded area is AreaP QRS = ......................
........................
........................
... . . . . . . . . . .
1 ........................
.........................
....................................
..........................
AreaT U V = 12 × 8 − × 8 × 8 = .......................... .......................... 8
2 ........................... .... . . . . . . . . . . .
........................... ........................................
96 − 32 = 64 cm2 , ............................
.............................
.............................
..............................
............................. ... . . . . . . . . . . . . .
............................... .............................................
hence (D). ............................... ................................
................................ ................................
. . . . . . . . . . . . . . ....... . . . . . . . . . . . . . .

S V R

13 1981 I.14 (31%)


 20 
Let the height of the trapezium be
h cm, as shown. The area of RST P  T ..Q

..... ..
..
1    ....
is 60 cm2 . Therefore × 12 × h = 60.     h .....
2     .
Therefore h = 10.  ...
  
.
.......
....... ....... ........
S 12 R
Now

AreaP QRS = AreaP RQ + AreaP RS


1 1
= × h × P Q + × h × RS
2 2
190 Geometry Solutions

1
= h(P Q + RS) (1)
2
1
= × 10 × 32
2
= 160 cm2 ,
hence (D).
Note: The expression (1) is the standard formula for the area of a
trapezium.

14 1979 I.19 (22%), S.12 (40%)


Alternative 1
If the square P QRS were rotated clock- A B
wise about P making P Q parallel to AB,
then one-quarter of the squares would X................................ Q
P ......................
...................................... ...
overlap. Each square has area 100 cm2 ...
...
..
. .... ...
...
...
... ...
so one-quarter of the square has area .....
....
...
...
... ...
25 cm2 . Since the shaded areas are con- ..........
. ...
. .
...
...
...

gruent (ASA), area of region P XCY is D Y ...


... C ...
...
...
... ...
...
also 25 cm2 , ...
... ..................
...
...
...
...
. . . . . .
.
...
R
.....................................
hence (B). S
Alternative 2
P QRS could be rotated anticlockwise about P so that the overlapping
1
region is P BC. The area of P BC is × 100 cm2 = 25 cm2 . As in
4
Alternative 1 a congruency argument shows this to be equivalent to the
area of P XCY ,
hence (B).
Alternative 3
We will find the required area by find- A B
ing the areas of XY P and XY C: 4
CY = BX = 4 cm, by symmetry and .Q
P ..............................................X ...................................
...
CX = 10 − 4 = 6 cm. ...
...
.
.....
...
...
1 ...
...
..
.....
...
...
Therefore AreaXY C = ×4×6 = ... ...
... ...
...
...
2 ... ...
......
...
...
2 2 2 2 ... ...
12 cm . Now XY = CY + CX = D ...Y
...
C ...
...
2 .
... ...
36 + 16 = 52 cm . P Y = P X, by symme- ...
...
...
.
...
2 2 2
try. Also XY = P X + P Y = 2P X . 2 .
... .......................................
..
.
........................ . . . . . .
R
1 S
So P X 2 = (52) = 26 cm2 .
2
Area 191

1 1 1
So AreaXY P = P X × P Y = P X 2 = × 26 = 13 cm2 . Thus
2 2 2
AreaP XCY = areaXY C + areaXY P = 12 + 13 = 25 cm2 ,
hence (B).

15 1980 I.24 (8%), S.17 (39%)

By Pythagoras, P R = 10 cm. Since P RS P ...... .. Q


......
...... .......
is equilateral P S = SR = 10 cm. If M ......
...... 5
◦ .........
60 ......
is the foot of the perpendicular from S to ......
......
...... M
...
P R, P M S and RM S are congruent and .. .....
.. ...........
......
.. ...... 5
P M = M R = 5 cm. √ Therefore
√ M S2 = ...
..
. . ......
......
......
...
100 − 25, and M S = 75 = 5 3 cm, .. ◦...................
.
..
...........
60
.
hence (A). .
.. ...
.. ..
...........
..........
.. ...
.. ...
R
. ...
.. ◦ ..................
. .
..
60
.. .............
...........
S

16 1980 J.17 (18%)


The radius of the circle is x cm, giving it an area of πx2 cm2 . The area
of the square is (3x)(3x) = 9x2 cm 2 . Therefore the shaded area equals
9x2 − πx2 = (9 − π)x2 cm2 ,
hence (B).

17 1982 I.20 (21%), S.13 (29%)

Suppose the radius of the quadrant OP Q is P ..


............................
...................
....... .........
2r and the radii of the semicircles are r. We .. . ...... . ........
...... . . ......
..................
.... . . .......
... ..............
observe that the area of the quadrant equals the .... . . . .......
..................
....................
area of the semicircles on OP and OQ +b − a. b
... . . . . . ...
.. . . . . . ....
. .......................................................
1 ...
.
..... .
.................
. ........ . ...
.............
Thus π(2r)2 = 12 πr2 + 12 πr2 +b−a, i.e. πr2 = . .
............... ... ...... ....
.
... ...... ...
................. ... .........
4 ..................... ....
.
a ... ..
......
a ... . . . . . . ....
.. ................. ......
πr2 + b − a, i.e. a = b, i.e. = 1, . . . . . . ..
.... ....................
.....
....
...
b ......................
.......................  ...
hence (D). O Q
192 Geometry Solutions

18 1980 J.25 (4%), I.25 (6%), S.14 (22%)


...... .................
............. . .................
...............
The accompanying diagram shows that the en- ..... . . . . .
..... . . . . .
... .................. 1
.................
.......... .....
. ............ ...
closed area consists of five 1 metre squares (i.e. ... ............. ............ ....
... . . . . . . . .................
.. .............. . . . . . . . ...
... . . . . . . .
5 m2 ) together with the four shaded portions. 1
These make up four quarters of a circle of ra- 1
dius 1m, i.e. have area π12 = π m2 . The total ... . . . . . . . .
.................. ........................
area is (5 + π) m2 , ... . . . . . . .
... .............
... ............
1 ............. ...
........... ...
.... . . . . . ...............
hence (D). ................
....... . . . .
..............
................
...............
...
............ ..................

19 1983 I.22 (5%), S.14 (17%)

Clearly P QR is equilateral and  P QR = R ..


..............
 QP R = 60◦ . The area under the archway ...... . ... ......
..... .. .. .....
.
..
...... . . ..... .........
. . ...
..... ... ... ... ........
.... .
... ... .. ...
... .. .. ...
...
... ...
= Area sector P QR (P as centre) .
..
.
..
. ...
.
.
..
..
...
...
...
...
. . ..
... ..
. . ..
.. ... ....
+Area sector QP R (Q as centre) ....
... ...
..
..
.. ...
... . ... .....
.. .. .... .. ..
−Area P QR (otherwise ... ..
..... ........
.
... ....
.. ...
..... .. .. ..
counted twice) P S Q

This equals
1   1  1 √ 4 √
π(2)2 + π(2)2 − (2) 3, = π − 3,
6    6 2   3
Area of a Area of P QR
full circle √
3 being the length
radius 2 of the perpendicular RS

hence (B).
Area 193

20 1982 S.16 (5%)

Let the lengths of the diagonals of the rhom- A


bus be 2a and 2b, as shown√(a > b). By ..... √
Pythagoras, the side length is a2 + b2 . Since  .... a a2 + b2
the area of the square is double the area of  ..... 
 .. .....
....... ....... .......... ........ .....b
.......
the rhombus, AC 2 = 2 × (2 × ABC). Thus B  C
..

2  
..... 
a
a2 + b2 = 2(2ab), i.e. 2 + 1 = 4
a
, on  ...  .

 a 2 b a b  .... 
....
division by b2 . So −4 + 1 = 0, and
b b
√ √
a 4 ± 16 − 4 4±2 3 √
= = = 2 ± 3.
b 2 2
a a √
Since a > b, it follows that > 1 and the required ratio, = 2 + 3,
b b
hence (A).

21 1978 S.17 (6%)

Suppose x cm is removed at each corner, A.. C


  
......
as shown. Then clearly BC has length x 
(n − x) cm. Consider the right-angled isosce- n−2x 
 
les triangle, XY Z as shown. XY Z and B
n−2x
ACB are similar. Therefore

x 1 n  
=√ ⇒ x= √ 
n − 2x 2 2+ 2  

n(2 − 2) X .. 1 Z
=

......
2 √ 
n2 6 − 4 2 
⇒ x2 = 1  √
2
2
 4√  45◦
n 3−2 2 
= . Y
2
 √ 
The total area removed is 2x2 cm2 = n2 3 − 2 2 cm2 ,
hence (D).
194 Geometry Solutions

22
................................................
........ .......
Consider an equilateral triangle OAB of ........ ...
........ .. ... .........
.......... ..... ... .. ......
.
..
... ... ... .. ..... ........
.
side length r. If P is the midpoint of ..
.
.
.. ....
.
... ....
...
....
... ...
..... ... ...
... ...
... ...
... ... ... ... ... ...
its base AB, then its height  OP is given .... .....
... ...
...
... ..
.
.
... ... ...
... ..
... ...
... .. ... ...
√ r 2 .
.
.........
...
O
.... ...
...... .
.
. .....
......
2
by OP = OA − P A = r − 2 2 = ...........
....
.
.
........ . ..
....
.
.. .... .......
√ 4 ... ...
... ..
.
. .
.
..
. ... ....
. . . ..
..
. ...
. .
... ... ... . ..... ... ..
3r 1 ... ....
... ...
...
..
... ...
..
...
...
... ...
... ...
. Thus its area is × base × height = ... ...
... ..
..
.
..
...
...
... .
... .....
. .
2 ... ....
√ √ 22 ..... ...
..... ... .....
..... .. ..
..........
..
... P
...
... ..............
... .........
1 3r 3r ........
..... ...... . ........
×r× = . A ............................................ B
2 2 4
The √ area of the
√ hexagon inscribed in the circle of radius r is
3r2 3 3r2
6× = . But since the area of the circle is πr2 = 2π, we
4 2 √
3 3×2 √
have r2 = 2. Thus the area of the hexagon is = 3 3,
2
hence (B).

23 1982 S.25 (9%)


Alternative 1
Produce T S to meet QP in S  . Then the Q ..
............
...........
area of RQT equals .
...
...
..  ............
............
.
............
T ...........
.........  S ......
...
AreaT SQ + AreaT SR ....
...
4 S
1 1
..
... 
...
= T S × QS  + T S × S  P, ....
..

2 2 ...
... 
....
 
..
...

since QS and S P are the respective heights 
...
..
........
.
of T SQ and T SR, and this equals R P
1 1 1
T S(QS  + S  P ) = T S × QP = × 4 × 6 = 12 cm2 ,
2 2 2
hence (A).
Circles 195

Alternative 2
The position of S is not specified in the T... 4 Q, S
....
question and so may be assumed to coin-
.
...  .....
......
......
.
...
..  .
......
..
......
cide with Q. Thus the area of RQT equals ...
...  .....
......
1 1 ...
..
.  ...
.
......
.....
6
T Q × QP = × 4 × 6 = 12 cm2 , ...
...  ......
.....
......
2 2 ...
..  ......
.....
.

. .....
hence (A). ... ......
.. ......
 .........
.......  ........
.
R P

24 1983 J.19 (20%), I.12 (28%)

Labelling the diagram further as shown R Q S

.....

.
.....
it can be seen that .
 . . . . . .
................
.
........................
1 ..............................
 .
. . . . . . . . . ..
Area P QR = Area RST U,  ...........................
.
. . . . . . . . . . . ....
.

8 ...............................................
P ...................................................


.
..................................
.

and  .............................
 ............................
...................
........................
 
...................
.
.............. .
Area P T U = Area QST  ............
....
.........
1  ..

...
= Area RST U.
4 U T

Therefore
 
1 1 1 3
Area P QT = 1− − − Area RST U = Area RST U,
8 4 4 8

hence (B).

CIRCLES
1 1979 J.11 (24%)
It is clear that the width of the rectangle is twice the radius of the circle,
or 2 cm, and the length is four times the radius, or 4 cm. The shaded
area is the area of the rectangle minus 2 times the area of the circle, i.e.
(4 × 2 − 2 × π × 12 ) cm2 = (8 − 2π) cm2 ,
hence (B).
196 Geometry Solutions

2 1981 S.8 (53%)

After making the constructions shown P...


......
.
and denoting the circle radius as r, we ...
4 ...

have that QOS and QOT are con- ...
.
.............................
gruent (RHS), therefore QT = QS = 4.  .
....
.
......
.
........
...
...  V ......
.....
T 
.. ... ....
.
....... ... ...........
Similarly, P V = P T = 4 and V R = .... ... .... O ... .. ...
 
..
... ... ... ... .. ...
...
.... ............
...............
 
..
SR = 7. Therefore the perimeter is 30, ...
...
... ..........
.
.... .... .
.... ... ...........
....... ... . ..
.
 
........ . .
. .
........
hence (A). ... . .. r .
. . ..
. .............
.
...... ......... . ..... .......
...... .....
 
....... .......
...... ....................................... .......
...... ..
Q 4 S 7 R

3 1983 S.8 (52%)


..............................................
......... .......
....... ......
Let y and z be the angles as shown in the .......... .....
. .....
..... .....
.....
diagram. Since y ◦ is subtended from the .
. ...
....
.
...
.......... ....
. ◦ ◦.............. .......
same chord as that which subtends 44◦ , y = ..
.
.. ........
. y
......
......
x... ...... ...
...
.... ...... ..
...
.. ...... ...... ...
44. Clearly, z = 180 − 96 = 84. Thus x = ....
...
......
......
z
......

................ ...
.
...
... ...... ........ ..
180 − 44 − 84 = 52, ......... ◦ ..
...
...
...
.
. .
.... .........
......
96 ...... ...
.
.
...... ...
hence (C). ...
...
... ....
.
......
.....
......
......
...... ..
..
..
... .... ◦ ......
...... ......
..... ......... 44 ..........
.........
..... .....
......
....... .........
......... ...........
...........................................

4 1981 I.10 (24%), S.4 (53%)

 P RQ is a right angle. Therefore (by R ..........................................


........................ .
....... .......... ........................ .............
Pythagoras) ......
............. .
.
.
..
.
1 ......................................
....
Q
.. .

... . ...
... ...
 √ ..
... ...
..
...
...

. . ...
P R = 22 − 12 = 3, ...
.... ..
.
.... 1
...
...

. ...
... . ...
... .... ...
...
...
.
..
. •
 ...
.
hence (D). ... .
.. O ..

. .
... ..
. ...
...
... ... ...
...
...
...
... ....
.. 1 ..
...
.
...


.... .. .
...
...... .....
..... .....
...... .....
P ......
........
............ ......
...
.......
...................................
Circles 197

5 1978 J.15 (7%)


...............................................
......... .......
C is the midpoint of the chord AB and the ....... ......
...... .....
......... .....
.....
....
length of OC (x cm) is the (perpendicu- ...
.
.
. ...
...
...
.... ...
..
lar) distance from the centre O to AB. ....
. ...
...
...
...
Using Pythagoras, x2 = 132 − 52 = 144, ...
... O
...
...
..
i.e. x = 12,
...
...
........
...........
... ........ x cm
.
. ....
...
..... B
... .... ......... .
. . .
.... .. .
... . .......... . .. . .
.. .
C
... ...
hence (B). ...
...
...
...
.............. ..
........ ...
... ... ... ..
... 13 cm ... .. ...
..... ... . .. ...
..... ... ................
..... ... .........
...... ............
.......
......... ......
...............................................
A

6 1978 S.11 ()26%

We will use the fact that an equilateral


√ triangle X
of side length 2 units has height 3 units, as 
shown. Let the radius of our circle be r. Clearly,  
3 2
30◦ 
r= cm.  √ 
2π ◦
3

60 ........
. 
Y 1 T Z
Alternative 1
Since the centre of the circle is the centroid of A
AB 2 ....
ABC, AP = 3r. = √ (XY T , ABP
AP 3  ..
.. 
...............................
2 √ ...
..
...... ..  .....
...
similar). Thus AB = √ (3r) = 2 3r. There-  ...
.
..... 
...
...
3 ..... .
 
... .. ..
..
√ √ 3 ...
....
.r
.
....
.
 
...... ..
fore the perimeter equals 3×2 3r = 6 3× = .......................................

√ 2π B P C
9 3
cm,
π
hence (C).
198 Geometry Solutions

Alternative 2
Triangles
√ BP O and XT Y are similar. Therefore A
BP 3 √ 
= , i.e. BP = 3r. Thus the perimeter  
r 1 √ ....................
√ √ 3 9 3 ... 
.....
........
 .....
....
...
...
is 6 × BP = 6 3r = 6 3 × = cm, ....
...  ...
O  ...
...
.
2π π  ... .. ... .
... .....
.. .
...r ....
..

hence (C).  ...... ...◦ .....
...... 30 ............................
....... ..... .
....

B P C

7 1978 S.2 (21%)


....... ............ ....... ....
....... ...
Let the circle have radius r. Then the .
......
......
. .....
.. ..........
..... ............. .....
distance between the two lines is 2r. . ... ... ..
.....................
... ......... .. ...
...
... ... .....
....
....
.. ...... ..
.................
The locus of points equidistant from .... ........................ ........
....... ..
.....
....
...
...
C
......
........... . .... ............. .... ..
.......
both lines is a straight line distant r ... .... .
...
. ... B ..... ..... ... .. .............
.
...
...
... ...
......................... • ............ .. .
...
from each. The points r units from the ..
.............
............ ....
.
.
...
.
A ............... ... ...
... .
..............
.............
.
circle are those on a circle of radius 2r ...
..
.....
......
.........
...................................
.... ............. .
.................... .
...
. ....
about B, the centre, together with B. ...
... .......................
...
.. ......
.
.
...
..................... ..
.....
These two loci intersect at A, B and C, .......
....... .. .... ... .
......
..... .... ...
hence (C).

8 1984 I.15 (14%), S.9 (39%)


The circumference of a circle is 2π × radius.
 So the
 circumference of the
r 2r 1 2
shaded region is πr + π + π = πr 1 + + = 2πr,
3 3 3 3
hence (A).
Circles 199

9 1978 J.24 (17%), I.18 (16%), S.20 (32%)


Alternative 1
. ..
... ...
The three separate circular sec- ..
.. ...
..
tions of wire together comprise the .
.
.......
.
. .............................
............. .......
.....
.. ..........
.... ..
circumference of one circle of diam- ... .... ..
.
.
. .....
.
....... ........
....... ....... . ..... ......
.. ... .. ...
eter 1 m, and thus have total length .
... ... ...
.
.. ...
.. ....
1m .
... ....
. ... ... ... ...
π m. The three straight sections of .
...
.
. ...
..
.. .
....
.. ...
..
.. ...................................... .....................................
............. ............. .................. ........
wire each clearly have length 1 m. ..
..
......
... ..
.
. .
.
.
. ...... ..... . .
... ..
. . .
... . ....
B
.............
........ .. . ... .. ... ...
Hence the total length is (3 + π) m, .... ..... . . . .
. ... . ...
. ..
... ....
....... .
.
. .....
..
..
O ..
....... ◦
......
.
.. .
... .... ..
hence (A). ..
... ....
...
.
.
.. .
..
....
.
.. ....
.
..
.
α .
.
..
.. .....
. .... .
.
. .. .... . .. ...
... ..... ..... ..... ... .....
...
◦ ..
.
...... ...... ......... ......
.............................................. ...60
..... ....... ....... ........................................ ... .... ....... ....
A C

Alternative 2
Construct an equilateral triangle as shown. Consider the interior angles
of the quadrilateral OACB: α + 90 + 60 + 90 = 360, so α = 120. Thus
120 1
the section of wire AB is = of the circumference of a circle and
360 3
the total length of the wire can be found as above,
hence (A).

10 1984 I.18 (5%)

Let the radii of the small and large circles R


......................................................
......... .......
....... . ......
be r1 and r2 respectively. OP R, where O is .........
...... ..
.. ....
.....
.....
.....
..... . ...
the mutual centre of both circles, is a right- ...
.
. 7 .. .... ...
.... . . r2 ...
...
... ... ............................. ...
angled triangle, right-angled at P . Thus, by ....
..
.
......
...... .
.
. .....
..... ...
P .
.... ....... ....
.... ...
...
Pythagoras’ Theorem r12 + 72 = r22 , i.e. ...
...
...
... r .......
... 1
...
..
...
...
.
..
... ..
r22 − r12 = 49. But
... .
O .
 the area of the annulus is
... ... ... .
...
... .... . .
. .
.
..... .. .
... ..... ...
πr22 − πr12 = π r22 − r12 = 49π cm2 , ...
...
...
........
.....................
..
...
... ....
.
hence (B). .....
.....
..... .....
.....
...
...... ......
.......
......... . .
...
........
............................................

Note: On the assumption that the answer is determined by the given


information, one can arrive at the correct answer by considering the
degenerate case. That is to say, the case where the unshaded circle has
zero radius. The diameter of the shaded circle is then 14 cm and the
answer is 49π cm2 .
200 Geometry Solutions

11 1981 I.22 (9%), S.15 (20%)

The diagram is a vertical section through ..........


M
.......................................
........
.......
...... ......
the top of the ball. QP is the radius of ...........
.
......
.....
.....
... 4
13 5 .....
.. ...
...
the required circle. OQ = −4 = . ......
........ ....... ....... ....... ....... .. Q
...
... P
................ .....
2 2 ....
. ....
........... ....... ....
...
By Pythagoras, ...
.. ........
...
........ 13
..... ...
...
...
..... ......... 2
...
  ...
...
...
O
.
....
169 25 144 ...
...
...
...
.

PQ = − = = 6. ...
... ..
...
4 4 4 ...
....
..... .....
....
..... .....
...... .....
.......
....
.......
Therefore the circumference of the cir- ........
.............
...............................
........

cle contacting water surface is 2π × 6 =


12π cm,
hence (A).
12 1980 J.26 (12%), I.23 (15%), S.10 (29%)
...
Let x cm be the radius of the cir- .
........
......... .....
P
.......... ...
cle. Then the length of OQ is (x − ........
.
...
... .
........
...
.
...
...
........... ...
10) cm. Using Pythagoras’ Theo- x ..............
..
......
...
...
10
...
...
...
.
.......
.
rem on OQP , (x − 10)2 + 502 = ..
.. .
.......
...
.. .
.......
...
.. ...
...
...
.
.......
.
x2 , i.e. x2 − 20x + 100 + 2500 = x2 , .
....
.........
..
...
.. .....
...
...
...
O x−10
.
... R
i.e. 2600 = 20x or x = 130, .... Q
hence (D).
13 1978 S.21 (57%)
If two circles have radii and circumferences of r1 , C1 and r2 , C2
respectively then C1 − C2 = 2π(r1 − r2 ). In this case,
|C1 − C2 | = 2π(1) km,
hence (A).

14 1979 S.18 (34%)


1 ......
..........
..........................
.......
......
BD = (length of common chord BE) ......
...... .....
....B .......
2 .
..
...
. .............. ...........
............
.....
....
= 8 m. ....
.
17.........................................10
.. ... ...
...
...
... . .
....... ...
... ... ...
.... . .
In BDC, DC = 6 m (Pythagoras). ...
... A
...
... .
...
...D .. ... .
C ..
.
.
..
... ..... . ..
... ... ...
In BDA, AD = 15 m (Pythagoras). ...
...
.......
.. .....
.
....
.
... ................................
..... ...
Therefore AC = AD + DC = 21 m, .....
......
......... ......
... .. .... E
.................................
hence (B).
Circles 201

Note: The question says ‘a possible value for the distance between
centres’. Since A and C could be on the same side of BE there is
another possible value, namely (15 − 6) m = 9 m.

15 1978 I.28 (11%)

(3, 4, 5) is a Pythagorean triad, so  B = A.



...
90◦ . Let the radius of the centre be
.
... .. 
r cm. ...
.. .....

............ ..................
.......... .....

. ..
Alternative 1 ..... ...
.... 5
...
3 .....

.. ...
.. ...
...O

...
Area ABC = 12 × 4 × 3 = 6 cm2 . But ...
... ..
..
. ...
... .... ....... ....
..

...
... .... .......
.
also ...
.... ....
......
..
...
...
. .......
.......

.......
.....
AreaABC = OAB + OBC + .....
.. ................ ...............
..........  .......
......
B 4 C
OCA

1 1 1
= × 3 × r + × 4 × r + × 5 × r,
2 2 2
(since r is the height of each)
= 6r cm2

Therefore 6r = 6, i.e. r = 1 and the area of the circle is πr2 cm2 , i.e.
π cm2 ,
hence (C).
Alternative 2

x
r+y = 4 (1) ......
x ..................... ......
...... ....
.....
. 
y+x = 5 (2) ....
.
... ........
...

... .. ..
. . ...
... ...
r+x = 3 (3) ..
y
.
.... ... ....... ....... ..... .
..
..... .O
...
...
.
.. ...
.
..

(1) + (3) gives 2r + (y + x) = 7, i.e.
r .....
.....
..
.....
.
...
...

.......
r
.
........................................
y

2r + 5 = 7 or r = 1 and the area of the
circle is as above,
hence (C).
202 Geometry Solutions

16 1983 I.23 (19%), S.17 (35%)

Construct horizontal and vertical S


................................................ ... V
........ ....... ..
lines through P , R and the mid- ....... ..... ......
y ...... y ..... ...
.
........ . ..... ..
....
....... ....... .............P ... .
point of P R as shown. The shaded ...................................................... ....... ....... ....... ............
......
...... 
....................................................
............................................ ......
... ....
.
....

........................................
right-angled triangles are similar to ...
.. ................................
............................ 3 .. ....

. ...
..
. ....................
...
................ ..... ....
P QR and since P R = 10 units .. 3
........
....
 ..
..... 6 .........................4 ..................................x .. ....

.
the shaded triangles have dimen- ...
... M ............................................................................................ ......
...
...
. . . . .
. . . .
................................... ..
............................. ..
. . . .
sions 3, 4, 5. The two shown radii 
... ....................... ..
..... ................. ...
...... ........... ...
emerging from M each have length
....... . ..
............... ...
...
...
8  ..... ...
...
...R
Q ..... ...
.....
5. Therefore 3 + y = 4 + x = 5, ...
...
... . .
.
.. ..
... 4 ... ..
i.e. y = 2 and x = 1. So the length ...
.....
..... ....
....
... ..
...... .. .. . ....
.....
of ST equals y + 6 + 4 = 12 and .......
...........
.............................
....... .

T xU
the length of T U is 3 + 8 + x = 12.
The area of ST U V is 12×12 = 144
square units,
hence (C).
17 1979 S.28 (8%)
...
...
Since 1 + 2 + 3 + 4 + 5 = 15, it is clear that .
.....
...
.................................. ....
.
..
...... .........
.......
there are 5 rows of billiard balls within ...
.
...... ....
...
.
.. .. .... .......
the frame. Hence the number of balls in ....
...
.
..
...
...
. ......
.......
... .......... .
... ..
contact with any one side of the frame is ...
... .
..
.
.......
.
.........
.......... . .
.
.. ...
.. .....
... . ...
.. ...
5. Let the radius of the billiard balls be ...
....
.....
r...
.
. .
.
.........
.... ............ .....
.
...... ..... . .
◦.................
...... ..............
r mm. ........
........................ 30
x
....

Let x mm be the distance from the corner of the frame to the nearest
point of contact with a ball. From the diagram it can be seen that
r 1 √
= tan 30◦ = √ . Therefore x = 3r. The length of one side of the
x 3 √
frame is (8r + 2x) mm, i.e. r(8 + 2 3)√mm. Thus, comparing with the
total perimeter of the frame, 3r(8 + 2 3) = 876. Therefore
876 146
r= √ = √ ,
3(8 + 2 3) 4+ 3

hence (B).
Circles 203

18 1981 I.27 (22%), S.25 (26%)


....................................
.................... ................
............... ..... .......
Let the radius of the third circle be r. ....... ....
...... ..... ..... ......
..
......... ..... ...
... ..........
.... . 1 ... .....
Since OS = 2, OT = 2 − r. Using .
.........
.
..
.
.
..
. ...
...
.....
...
... .... ... ...
Pythagoras in P OT , (2 − r)2 + 12 = .
..
..
...
...
...
Q ......
......
....... ..... . ...
...
...
. ... .
....... . ........... ..
...... .................. ............. ....
(r+1)2 . Thus 4−4r+r2 +1 = r2 +2r+1, ...
.... ...
...
..... 1 ........
. ......... r
.... ..
......
... .. .
. .....
2 .....
..
......
.......
.......O .......... ..
..... ..........
...... .. ...... r .....
.
i.e. 4 = 6r, i.e. r = , ... .... . .
.
......................................... ....
......... ...... T ........ ...
3 ... .
.. ....... .
...... ..... . . .... S
.
... .... ... r .....
... ... ............. .......
hence (A). ...
... ...
.
..
....
1 ........ ................................... ...
...
... .
... ... ...... ... ...
... .. ...... ..
... .... P ...... ... ...
... ... ... .
....
.... ... . .
..... ... ...
..... ...
... .....
..... ..
... .....
...... ...... 1 .....
....... ..... .... ... ...........
........ ...... . ..
................... ...... .........
......................................................

19 1983 S.23 (15%)

The northernmost point is where a ray North


Pole
of light from the star is tangential to the ...
...................•
.....................
.........
..
....... .......
....... ......
earth’s surface, i.e. point A in the di- ....
.
......
.
.
.. .....
.....
.....
... ..
agram. The object is to determine the ...
.. equator ........ A
.
.
.. ....• ...
.. ...... .....
value of x in the diagram. Consider the ..... .
.......
. ......
......
....... ... ....

..
..... x◦ ... ... .....
quadrilateral OABC in the diagram. .... ..
.
....
O ...... ◦ ..
.
. ..... .. .. .... . . . .. ... .... .
.. .. ........
.. .....•
... . 35 19 ... ...... B

Because the angles at A and C are 90 , ...
...
......
.
...... ... ... .....
.
.
.
.......
...
... . . ........ ........
the remaining two angles must add to ...
...
..
. .... ..
...
...........
.. ...
.
...
Canberra......• ..
.... C
180◦ . Therefore x◦ + 35◦ 19 +  ABC = ....
.....
.....
...... .........
...... .....
...
....... ..... ... ...
180◦ . Because the rays reaching points .........

....... ...
............................................... ..... ◦
.. 62 20 .....
...
.
..
.. .....
A and C are parallel it is clear that South
..
....
.. .........
...
...

 ABC = 180 − 62 20 . ◦  Pole ...
Therefore ...
...
.
◦ ◦  ◦ ◦  ◦
x + 35 19 + 180 − 62 20 = 180 , giv-
ing x◦ = 62◦ 20 − 35◦ 19 = 27◦ 01 ,
hence (C).
Note: We have been assured that students familiar with French
literature were readily able to reduce the options to (C) and (D), i.e. the
two answers with northern latitudes. The well-known French novel Vol
de Nuit (Night Flight), by Antoine de Saint-Exupéry tells the story of
the adventure of the first intercontinental link between Europe and
America (Paris–Dakar(Senegal)–Natal(Brazil)). The author was a pilot
who pioneered the link. There are frequent references in the book to the
use of the Southern Cross in navigation over the Sahara, which is in the
northern hemisphere.
204 Geometry Solutions

20 1984 S.28 (6%)

The centre of the right-hand circle must be di-


.....P
...
.......... ....
rectly below the tangential point P . Let T be . .........
.
......
...
.
.....
. ..450
the point where both circles meet, having a ....
T ........... ........600
... ....... .........R
.
...... ... ....... .. x−450
common tangent, and let T R be the perpen- .
...
.
..
.
x ......
........ O
.....
dicular line from T meeting OP at R. .......
................

If the radius of each circle is x, then ORT is a right-angled triangle,


right-angled at R, with hypotenuse of length x and other sides of length
600 and x − 450. So (x − 450)2 + 6002 = x2 , i.e.
x2 − 900x + 4502 + 6002 = x2 , i.e. 900x = 4502 + 6002 , i.e.
(30)2 x = (15 × 30)2 + (20 × 30)2 or x = 152 + 202 = 625,
hence (B).

COORDINATE GEOMETRY
1 1978 I.4 (48%)
Substituting (a, 2) into y = 3x − 6 gives 2 = 3a − 6, i.e. 8 = 3a, i.e.
8
a= ,
3
hence (C).

2 1978 I.11 (34%)


3+x y+4
= 4 ⇒ x = 5 and = 2 ⇒ y = 0. •(x, 4)
2 2 
Thus x + y = 5, 
hence (A). •
 (4, 2)

•
(3, y)
3 1979 I.8 (33%)
3
Since the gradient of AB is − and the y-intercept is 3 the equation of
2
3
AB is y = − x + 3, i.e. 2y = −3x + 6. Thus 3x + 2y − 6 = 0,
2
hence (B).
Coordinate Geometry 205

4 1980 S.5 (71%)

A diagram shows that the radius of the circle 


....................................
....... ......
..... .....
is 6, ..
...
.
..... ...
...
...
.
hence (D). .... ...
...
... (−3,4) ...
....
...
...
• . ..
.
...
.
... . ..
.

... . ..
.
... .
..... ...
...... . ....
....... . .....
.......
. . . . . . ................................. . . . . .
(−3,−2)

5 1983 S.5 (82%)


3 3
Since the slope is − the equation is y = − x + c, where c is chosen so
2 2
 x = 4 ⇒ y = 0. Substituting (x, y) = (4, 0),
as toensure
3 3
0= − (4) + c, i.e. c = 6. Thus the equation is y = − x + 6, or
2 2
2y = −3x + 12,
hence (B).

6 1978 S.18 (29%)


y
Image of (x, y) in x = 0 is (−x, y). 
Image of (−x, y) in y = 1 is (−x, −y + 2),
(−x, −y) (x, y)
• •
hence (D). y=1

• x
(−x, −y + 2)
206 Geometry Solutions

7 1980 S.20 (15%)


y

R(k, ........
5) (4, 5)
20 = Area(I + II + III + P QR) I.................. ...........
.. ...
..... .
1 1 P (0, 3) ...............8..... cm2 .......... II
= × 2(k) + (4 − k) × 5 ..
......
......
......
. ...
...
...
2 2 ......
......
......
...
...
...
1 III
......
......
...... .....
...
+ ×3×4+8 ...... ...
...... ...
2 ...... ..
.........

.........
......
5k .

= k + 10 − +6+8 O Q(4, 0)
2
3k
= 24 −
2

8
Therefore k = ,
3
hence (B).

8 1979 I.29 (10%)


y
Alternative 1 
Position the coordinate axes as shown. Let 

length of OC, the perpendicular distance  A
between the parallel lines, be x cm. By  .... C
Pythagoras, the length of AB is 5 cm. Tri-  4 x....
..

  B
angles OAB, CAO are similar (AAA). Thus 
AB AO 5 4 O3  x
= . Thus = . 
OB CO 3 x
12
Thus x = ,
5
hence (B).
Alternative 2
1 1
The area of AOB equals × (OB × OA) = × (4 × 3) = 6 cm2 and
2 2
1 1 2 5 12
also equals × (AB × OC) = × 5 × x cm . So x = 6, i.e. x = ,
2 2 2 5
hence (B).
Coordinate Geometry 207

Alternative 3
The equation of the line AB is 4x + 3y − 12 = 0. Using the formula for
the distance
 of a point (the
 origin (0,0)) from the line (AB) we have
 (4)(0) + (3)(0) − 12  12
x =  √ =
 ,
4 2 + 32 5
hence (B).

9 1984 S.21 (11%)

C(x, 12)
12 .... ................
.. . ...
... .. ...
... . ...
AreaABC = AreaACDO ..
... .... ....
...
.. ... ....
.
...
+AreaDBC ...
... . ...
...
8 A(0, 8)
.......
...
...
...
...
−AreaOAB ...
...
...
.
. ...
  ... .
... ..
...
...
...
8 + 12 ... .
....
....
...
...
i.e. 20 = x .....
...
...
...
2 ... ..... ...
4 .... .
..
...
...
...
...
...
1 .. ...
...
... .....
...
..
+ (6 − x)12 − 24 ... ... ..
2 .
..
... ...
... ..
... ...
.. ... ..
= 10x + 36 − 6x − 24, ...
......
B(6, 0)
.....
...
. .
i.e. 8 = 4x, 0 D(x, 0) 6
or x = 2,

hence (A).
10 1979 I.26 (9%)
Since (0, a) lies on the graph of y = 2x2 − 8x + 9, it satisfies the
equation. Therefore a = 9. Arguing similarly for (b, 1), 1 = 2b2 − 8b + 9,
which on rearrangement gives 2(b − 2)2 = 0, i.e. b = 2. The gradient of
1−a 1−9
the required line is therefore = = −4,
b−0 2−0
hence (D).
208 Geometry Solutions

11 1981 S.20 (26%)


y
Let AB be the given line of slope 2. If α is  C...
... B
the angle at which it meets the x-axis, then ...
...

tan α = 2. Suppose AD and AC, as shown,
...
...
...

 ...
...
meet AB at 45◦ . The slope of AD is thus 45  ...
...

...

...

tan α − tan 45◦ 45 ...
... .............
.................................................. D
tan(α − 45◦ ) = A
1 + tan α tan 45◦ 
2−1 1 .
.
....
.
.α 
= = . .... x
1 + (2)(1) 3

As AC ⊥ AD, slope AC = −3,


hence (C).

12 1984 S.30 (7%)


Take origin O at the centre of cube and edges parallel to OX, OY and
OZ. The equation of the sphere is x2 + y 2 + z 2 = 9. We consider plane
sections at levels z = 0, 1, 2. Plane z = 1 cuts the sphere in circle
x2 + y 2 = 8 with points (±2, ±2) on the circle so there are 2 × 16 = 32
unit cubes contained within the sphere between planes z = ±1. The
plane z = 2 cuts the sphere in circle x2 + y 2 = 5 with points (±2, ±1)
and (±1, ±2) on the circle. There are 12 unit cubes within the sphere
and between the planes z = 1 and z = 2. So the total number of unit
cubes within the sphere is 32 + 2 × 12 = 56,
hence (B).

CUBES
1 1980 J.9 (55%)

Six 2 cm cubes placed together as  4 cm 


shown form the given rectangular  
  
 
block, 6 cm   
hence (C).    


   

2 cm 

Cubes 209

2 1984 J.19 (5%), I.11 (6%), S.7 (16%)


Note that P QR is equilateral, hence  P QR = 60◦ ,
hence (C).

3 1983 J.26 (4%), I.18 (7%)

There are three planes of symmetry as .. .....


....... ...
.......

shown in the first diagram (parallel to  ....... ............ .......  ....... ............ ... ..
.
..
.....
..
.... . .. .. ...
.....
.... .
........ ... .
..
the edges). There are six planes of sym- .... ......
. .
.....
..
..
.... .
........
... ..
....... ........ ....... ..... .... .........
. ..
metry demonstrated by the six dotted .
.
.
 ..... ..... ....... ....
..
.. 
.
.

lines in the second diagram. Total equals ....  . .


Figure 1 Figure 2
3 + 6 = 9,
hence (D).
Note: The following argument establishes that all cases have been
exhausted. The 8 vertices of the cube are not in a single plane.
Therefore given any plane of symmetry there must be one vertex not in
it. But because it is a plane of symmetry there will be a pair of distinct
vertices which are symmetric in this plane. Note that the plane of
symmetry is the perpendicular bisector of the segment joining the two
vertices. Consider the following possibilities:
Case 1: Pairs of vertices in which the connecting segment is an edge.
Here we observe that the perpendicular bisector of an edge is also the
perpendicular bisector of three other edges parallel to the edge
considered. There are 12 edges consisting of 3 sets of 4 parallel edges.
Each such set has a common perpendicular bisector. Thus there exist 3
planes of symmetry of this kind (those shown in Figure 1).
Case 2: Pairs of vertices in which the connecting segment is a face
diagonal.
The 6 faces of the cube have altogether 12 face diagonals which fall into
6 parallel pairs. Thus there exist 6 planes of symmetry of this kind
(those shown in Figure 2).
Case 3: Pairs of vertices in which the connecting segment is a body
diagonal.
The perpendicular bisector of a body diagonal is not a plane of
symmetry because the reflection of any vertex off the body diagonal is
not a vertex. Thus there are no planes of symmetry of this kind.
The total number of planes of symmetry is thus 3 + 6 + 0 = 9.
210 Geometry Solutions

4 1982 J.15 (43%)


For any face of the cube the only other face which may be painted the
same colour is the one directly opposite. A cube consists of three such
pairs of opposite faces. Thus at least three different colours are needed,
hence (D).

5 1984 S.15 (10%)


...
.
.
Consider the edges of the cube to be of length
 .... 
1. Each face of the tetrahedron is an √ equi-  ..
.. 
2  √2
lateral triangle with sides of length 2, and √ .... √
√ 3
3  .. 2
.. 
thus altitude
2
. Therefore the required ra-  ...
.. 
tio is  ..
.. 
√  ..........
√ .. 
1 3 √ √ √ 2
6:4× × × 2 = 6 : 2 3 = 3 : 1,
2 2
hence (D).
6 1982 I.16 (11%)

Label the points R and S as shown. Suppose P .....


..... ..... 
...
the side length of the cube is x units. By
Pythagoras,  ...
..
...
.. 
...
... ..
.. ...
2 2 2 2 2 2 ... ..
SQ = RS + RQ = x + x = 2x .. ...
..
...
.. ...
S .......
.... ...
.
....... ....... ........ ....... .......
and ..
.....
.......
.......
...
..

....... ....

..
..... ........

P Q2 = P S 2 + SQ2 = x2 + 2x2 = 3x2 = 12 R Q

(given).
Thus x2 = 4, x = 2 and the volume of the cube x3 is 8,
hence (A).
Cubes 211

7 1979 J.20 (29%), I.10 (46%), S7 (63%)

On each face of the original cube there are   ..........................  .......................... 


.......................... ..........................

exactly four 1 cm cubes with one face only     
painted red. As the original cube has six ....
........... ............
faces there are 4 × 6 = 24 of the required ..... .....
........
..... .....
........ ....
.
....... ....... ...
..... .... ..... ....
cubes, ........... .............
....
..... ..... ..... ..... .
hence (E). ........
.......
........
.......
..... .... ..... ....



8 1981 J.20 (19%), I.17 (22%), S.10 (27%)
.......................... ..........................
A block with exactly four faces in contact  ..........................
..........................
  ............................
.......................... ......
.

with neighbouring blocks will have two     .................... ......................... ..........................


.
.. .
..... ..... ..... ..... .. ..
........ ........ .....
faces exposed. On each of the 12 edges .......
..... ....
.......
..... ....
.
.....
 .
.. .
of the cube there are 2 such blocks. Thus .
....... .... 
..... ..... ..... ..... ...... .
........ ........ ... ......
....... .
..... .... ..... .... . ......
the total number is 12 × 2 = 24,  .
..
..... ..... .......
....... .... 
..... .....
hence (C). ........
.......
........ ... .
..... .... ..... .... . . ......
.......... .........
... 
..... ..... ..... ..... ..
........ ........
....... .......
..... .... ..... .....

9 1984 J.25 (8%)
Suppose the face numbers are a, b, c, d, e and f . There are 8 vertices,
each vertex number being a sum of these face numbers, i.e. 24 numbers
altogether. The 6 face numbers must occur equally often, in the sum of
vertex numbers. The sum of the vertex numbers will be
4(a + b + c + d + e + f ), i.e. a number divisible by at least 4,
hence (B).

10 1982 J.24 (32%)


Alternative 1
First note that the cube has 6 faces and 8 vertices. After cutting there
are 8 triangular faces (corresponding to the 8 original vertices) and 6
square faces (corresponding to the original faces), that is, 14 faces
altogether. Also note that every edge belongs to exactly one triangle
and the 8 triangles have 8 × 3 = 24 edges,
hence (A).
212 Geometry Solutions

Alternative 2
Make use of the famous Euler’s Relation connecting vertices (V ), edges
(E) and faces (F ) of convex polyhedra, namely V − E + F = 2, i.e.
E − F = V − 2. Now the resulting solid in the question has V = 12, i.e.
E − F = 10. The only possible given choice is E = 24 and F = 14,
hence (A).

11 1979 J.29 (23%)


A cube has 8 vertices and 12 edges. As each corner is cut off 3 new
edges are formed and all original edges are retained. Hence the total
number of edges is 12 + (8 × 3) = 36,
hence (D).

12 1981 J.30 (12%), I.30 (14%), S.30 (13%)


z
Alternative 1  • • •y
 
Parallel to the x-axis there are nine lines. •
 • •
 •
Considering the y- and z-axes also gives • • •
• •
3 × 9 = 27 lines. There are 2 diagonals each
• • • 
in the three planes parallel to the xy-plane. •

Considering the yz and zx planes also gives • • • x
3 × 3 × 2 = 18 lines.
The cube has 4 diagonals. The total number of lines equals
27 + 18 + 4 = 49,
hence (C).
Cubes 213

Alternative 2 A B C
Consider one face of the cube, labelled as  
 
shown. Also let X denote the centre of the  
cube. We count the number of lines on this D  F
face, or starting from this face and passing E
through X, and also the number of faces on  
which these lines will be counted:  
 
G H I

Times
Number of lines Lines counted
4 AC, CI, IG, GA 2
4 AE, BH, CG, F D 1
1 Through X from E 2
4 Through X from B, F , H, D 4
4 Through X from A, C, I, G 6

Thus, considering all 6 faces, the total number of lines is


 
1 1 1 1
6 4× +4×1+1× +4× +4× = 49,
2 2 4 6

hence (C).
Alternative 3
Following a brilliant solution by Leo Moser, University of Manitoba, to a
similar problem (American Mathematical Monthly, 1948, p 99, Problem
E773), consider a 5 × 5 × 5 cube which encases the given 3 × 3 × 3 cube
with a shell of unit thickness.
The two-way extension of a winning line in the inner 3 × 3 × 3 cube
pierces two of the unit cubes in the shell and each unit cube in the shell
is pierced by only one winning line. Thus each winning line corresponds
to a unique pair of unit cubes in the outer shell, and the number of
winning lines is simply one-half of the unit cubes in the shell, namely
53 − 33
= 49,
2
hence (C).
Note: This approach is perfectly general. The number of winning lines
(k + 2)n − k n
for a cube of edge k in n-dimensional space is .
2
214 Geometry Solutions

13 1983 J.30 (14%), I.30 (12%), S.30 (10%)


Note that the larger cube must be either 4 × 4 × 4 or 5 × 5 × 5, for a
3 × 3 × 3 cube contains too few (27) unit cubes, while in a 6 × 6 × 6 cube
none of the 64 unit cubes of the inner 4 × 4 × 4 cube will receive any
paint and 64 > 45. Note furthermore, that any time any of the faces of
the larger cube are painted, upon removing these painted unit cubes, the
remaining (unpainted) unit cubes form a k × m × n block, so the key to
the solution is found in the factorisation of 45 into the product of three
positive integers, each less than or equal to 5 since the larger cube is
either 4 × 4 × 4 or 5 × 5 × 5. This can be done in only one way, resulting
in a 3 × 3 × 5 block. This can be embedded only in a 5 × 5 × 5 cube, and
for that matter, only in one way, by adding a layer of painted cubes
around the four 3 × 5 sides,
hence (D).

POLYGONS
1 1984 J.28 (6%), I.26 (3%), S.22 (4%)
Alternative 1
Each vertex of an n-sided convex polygon has a diagonal connecting it
to n − 3 other vertices (i.e. to all vertices excepting itself and the two
adjacent). Considering all n vertices and noting that each diagonal will
n(n − 3)
be encountered twice, the number of diagonals equals . This
2
gives rise to the following table:

n 4 5 6 7 8 9 10 11 12
diagonals 2 5 9 14 20 27 35 44 54

Of the given options, only 45 is not feasible,


hence (C).
Polygons 215

Alternative 2
In a convex polygon, each line adjoining
non-adjacent vertices is a diagonal. Let A
....... ....... ....
 .. ....... ....... .....C
.
the number of diagonals of an n-sided con-
  ...
vex polygon be D(n). Suppose that AB is  
..
...
..
an edge of an n-sided convex polygon and   .....
that an (n + 1)-sided polygon is formed   .....
using a new vertex C, as in the diagram. ..B


There will be (n − 2) new diagonals, linking C with all the other vertices
except the adjacent vertices A and B. The line AB also becomes an
internal diagonal of the larger polygon. Therefore

D(n + 1) = D(n) + (n − 2) + 1 = D(n) + n − 1.

Now D(3) = 0. This leads to a table

n 3 4 5 6 7 8 9 10 11 12
D(n) 0 2 5 9 14 20 27 35 44 54

Of the options given, 45 is not a value of D(n),


hence (C).

2 1984 I.22 (5%)


P QRST U is a regular hexagon. Each side is of length
   2
2
1 1 1
+ =√ .
2 2 2

The hexagon can then be √ considered to be composed


√ of six triangles,
1 1 3 3
each of area × √ × √ square metres, i.e. square metres. Thus
2 √2 2 2 √ 8
6 3 3 3
the total area is square metres, i.e. square metres,
8 4
hence (A).
216 Geometry Solutions

PYTHAGORAS’ THEOREM
1 1978 I.2 (53%)

Using Pythagoras, x2 = 102 + 242 = 676, C


i.e. x = 26, 
x km
 
hence (D). N

A  

24 km


10 km .......

 ....

2 1980 I.9 (24%)

The required length is equal to the length of D


.
.....
the diagonal of the box, AD. This is found .... .
.
.....
.....
by applying Pythagoras’ Theorem twice:  ..
..
.
 .
..... 4
.... .....
..
...
... ..
.
................... ....... ....... ....... ............
C
AC 2 = AB 2 + BC 2 = 122 + 32 = 153 .. ....
..... ...
........ ....... .
...... ..
..
..... ...

.
.
.
... . ......
. 3

. ..
...
AD2 AC 2 + CD2 = 153 + 42 = 169. ..
.
... ..... . .
= ......... .
12
A B
Therefore AD = 13,
hence (C).

3 1981 I.18 (31%), S.6 (63%)

P XQ is congruent to QY R (ASA). Thus P


 β .......
... X
XQ = RY = 4 cm. By Pythagoras, QY = 
 
3 cm. The height of P above the floor is thus S
 
XQ + QY = 7 cm, 5
hence (B).  α
 
  Q
5 β
 
α .......
.
R 4 Y
Pythagoras’ Theorem 217

4 1980 I.16 (30%)

Suppose AB is the initial position of the B


pole and CD the final position. Using 
Pythagoras, 
 ................D
2 2 2 2 2  ...........
.
....
.........
OB = AB − OA = 25 − 20 , 
.........
...
.........
.........
.

.........
.
...
.

......... .......
A 
.
..
....... ..
thus OB = 15. Also C 20 O
 24 
OD2 = CD2 − OC 2 = 252 − 242 .

Thus OD = 7. Therefore the distance slipped is 15 − 7 = 8 m,


hence (B).

5 1982 I.22 (11%), S.18 (36%)

On reaching the top, the ant will have travelled on .......


.........
B
.........
the cube a horizontal distance of 4 m and a vertical ...
...
...........
.... 1m
.........
.........
distance√of 1 m. By√Pythagoras, the length of its .........
.........
A 4m
path is 42 + 12 = 17 m,
hence (E).
6 1980 I.28 (31%)

Alternative 1 R S
 
By Pythagoras, U S = 10 cm. Letting U P = x

10−2x 
Q .......
x cm it can be seen that QS = x cm and  .  .....

P Q = (10−2x) cm. Since U P R and U RS ...... 


..

.... 
are similar,
UP
=
UR
. Thus
x
=
6
, so 
x
P

UR US 6 10  
x = 3.6. U T

Thus P Q = 10 − 2(3.6) = 2.8 cm,


hence (D).
Alternative 2
Using the diagram above, together with Pythagoras,
RP 2 = U R2 − U P 2 = 36 − x2 and RS 2 = RP 2 + P S 2 , i.e.
64 = 36 − x2 + (10 − x)2 = 36 − x + 100 − 20x + x2 , i.e. x = 3.6. So
P Q = 10 − 2(3.6) = 2.8 cm,
hence (D).
218 Geometry Solutions

7 1979 S.22 (13%)

Construct the points F , G, J, K as shown. A... J... ... B


...
... ....... .....
... .....
Then, by Pythagoras, ...
... ...
.. .....
.....
.....
...
...
...a ... b .....
.
.....
.
..
... .....
... .. ......
x2 + b 2 = (DG2 + GE 2 ) + (EK 2 + KB 2 ) .......
... ..
... .. ..........
... . ......
.....
......
. .
........ ....... ................................ ....... ....... ....... ..........
= F E 2 + GE 2 + GC 2 + AF 2 F x .
....
.
...... .. ................
..
.........c K
.
...
..... .
. .....
...... ... .........
...... ....... .........
= (F E 2 + AF 2 ) + (GE 2 + GC 2 ) ...... .. .........

D G C
Thus x2 + b2 = a2 + c2 , i.e. x2 = a2 − b2 + c2 ,
hence (D).
Note: The multiple-choice form of this question allows us to find the
solution by using a special case. Some judgement in choosing a special
case is required. For instance, putting E at the intersection of the
diagonals will not eliminate any of the possible answers. However, by
locating E at a corner all incorrect choices can be eliminated. Suppose
E is taken at D. Then x = 0, and by Pythagoras, a2 + c2 = b2 . Thus
a2 − b2 + c2 = 0 = x2 . However, before concluding that (D) is the
correct choice it is necessary to check that no other choice is satisfied by
this special case.

QUADRILATERALS
1 1981 J.24 (18%), S.12 (60%)
2 2
The area of the rectangle is 16 × 9 = 4 × 3 . This area is given by a
square of side length 12, and thus of perimeter 48,
hence (D).
Note: At first sight it is tempting to solve this problem by rearranging
the pieces of the rectangle into the required square. Readers may find
the rearrangement simplified once the size of the square has been
deduced.
Ratio 219

2 1979 S.25 (7%)

Construct BY parallel to AD. Let A a ..B


..... ....... ....... .......
the heights of BXN and BY C   
be h cm and H cm respectively. Let   h
  
 ...N....... 
M N have length x cm, so that XN a X H
M
  
 .......
has length (x − a) cm. Since BXN  x−a  
h x−a  a Y  b−a  ....... ..
and BY C are similar, = . D C
.....
H b−a
H(x − a)
Therefore h = . Now 2(areaABN M ) = areaABCD. So
b−a
(x + a) (a + b)
2 h= H, using the formula for area of a trapezium. Thus
2 2
H(x − a) (a + b)
(x + a) = H. So 2H(x2 − a2 ) = H(b2 − a2 ). Thus
b−a 2  
2 2 2 2 2 a 2 + b2 a 2 + b2
2x − 2a = b − a , i.e. x = or x = ,
2 2
hence (A).

RATIO
1 1978 S.24 (33%)

Alternative 1 A
Construct KD  LC, as shown. 
Since M LC and M KD are sim-  
1 K ..
...........
........


ilar, LC = KD. Since ABC and  ..................................

 ........... L
2   .............................................
KBD are similar, and AC = AB,  α  α
...........
...........
.
1 B D C M
KD = KB. Hence LC = KB,
2
hence (B).
(Alternatively, construct a line through L, parallel to AB, and argue
similarly.)
Alternative 2
A careful scale drawing will distinguish between the distractors.
220 Geometry Solutions

2 1980 S.26 (25%)

Triangles T U S and T QR are similar. Q  R


........
..........
TS TU 3 1  ..........
.......... 
Therefore = = = (also show-  V...............................
...
...
...

TR TQ 6 2 
 ....
.......... 

..   S
..
...
............
ing that S is the midpoint of T R). QP V .......
.

P U 
and T RV are similar, therefore
 
QV QP TS 
= = (QP = RS = T S) T
TV TR TR
1
= .
2

1
Therefore QV = QT = 2,
3
hence (D).

3 1983 S.28 (9%)

Let P

kr

QS
=
RT  
QR RP U ....
.......
.
 (1−k)q
 
... ........
... .......
.......
PU ... .......
 
... .......
.......
= ...
...
.......
.......
PQ (1−k)r  
... .......
.......
... ....T
... ......
.......
areaST U  ...
...
...
 .
.......
...........
kq
=  
......
... .......
... .......
areaP QR ... ............
θ ...........
. 
= k. Q kp S (1−k)p R
 p 
We note
1
AreaQSU = k(1 − k)pr sin θ = k(1 − k)AreaP QR
2
and similarly

AreaP U T = AreaT RS = k(1 − k)AreaP QR.

Therefore

AreaQSU + AreaP U T + AreaT RS = 3k(k − 1)AreaP QR.


Ratio 221

Therefore

AreaST U = AreaP QR − 3k(k − 1)AreaP QR.

Thus
AreaST U
(k =) = 1 − 3k + 3k 2 ,
AreaP QR
i.e. 3k 2 − 4k + 1 = 0, i.e. (3k − 1)(k − 1) = 0. The only acceptable
1
answer is k = (given that S, T , U are points other than vertices of the
3
triangle P QR),
hence (A).

4 1984 S.26 (4%)

Alternative 1 P....
.....

Construct the lines P T and SR. Be- ..... ..........


 .
...
...
...3x
...
cause P QT and QST have a com- y

..
...
...
...
...
...

..
mon vertex (and therefore equal al- ...U
... ...

...

.. ...
titude) and equal bases, their areas Q......................... ...

...2x
...

..................... ...

....................
........................
are equal. Let this area be A. Let T .....
....... R
  .......
.......

 
areaP U T = 3B. Then areaU RT = y
.......
.......
.......
2B since it has the same altitude as  ......
.......
2 .
.....
.
.
 .......
P U T and times the base. S
.....
3
Let area ST R = C. Then since P QR and SQR have equal
altitudes and bases, they are equal. Thus A + 3B + 2B = A + C, i.e.
C = 5B. Now 3 × areaU RS = 2 × areaP U S. Therefore
15 5
3(2B + 5B) = 2(3B + 2A), i.e. 15B = 4A or A = B = (3B). Thus
4 4
5 5 3 3
areaQT P = × areaP U T = × × areaT RP = × areaT RP.
4 4 5 4
3 QT 3
Therefore lengthQT = × lengthT R, giving = ,
4 QR 7
hence (B).
222 Geometry Solutions

Alternative 2 P...
...

Construct the line QX where X is on  ..........


 .......X
P U and QX  SU . Then P QX and    .........
...

...
P SU are similar. Thus 
 ...U
 ..........


Q.. ....................
.................... ...
...
PX PQ 1  T...................................................R
..
= = .  
PU PS 2 

 
3 
In particular P X = XU = RU (since 

4 S
3
P U = U R).
2
Also, RT U and RQX are similar. Therefore
RT RU RU RU 1 4
= = = = = .
RQ RX RU + U X 3 7 7
RU + RU
4 4
QT 3
Therefore = ,
RQ 7
hence (B).
Alternative 3
Using Menelaus’ Theorem,
PS QT RU
× × = 1.
QS RT PU
Therefore
QT 2
2×× = 1.
RT 3
QT 3 QT 3
Therefore = , i.e. = ,
RT 4 RQ 7
hence (B).

SOLID GEOMETRY
1 1978 J.29 (22%)
S lies in plane ABR, so BS does. S lies in the plane BCP , so does BS.
So BS lies in both planes,
hence (B).
Solid Geometry 223

2 1978 I.25 (30%)

Using Pythagoras twice, C


..... 
OD2 = OA2 + AD2  ..
.. 
  O  ... B
= 12 + 22 + 32 ....... ....... . .
...... ....... ..
..
.
..... ....... ...
.... .....
D
....... ....... ....... ....... .......... ......
= 14. 1 ...
...
.
..... .......
...

√ .
..
.....
.. .....
.......
.
3
Thus OD = 14, ..... ... .......
.. ....
......... 
A 2
hence (D).
3 1978 S.29 (12%)

From DBC,
√ √ D L
DL = 22 − 12 cm = 3 cm.  √ 
From LDA, 2 2 2  3 
√ LM = 3 − 1 cm ,    
i.e. LM = 2 cm,
 .......
..   .......
.. 
hence (A). B 1 L C D 1 A

4 1979 I.28 (13%)

Let F be the centre of the square base. E will E ....


..............
... ... .....
lie perpendicularly above it. Let x cm be the .
.. ... .....
.. ......... .........
.. . .....
... ... .....
length of F B and of F A. In AF B, ... .. ...
.....
.....
.....
... . .....
... ... ....
x2 + x2 = 202 (by Pythagoras). Thus x2 = ...
.
D .. ...
.....
.....
.
.. ...............................................................................
200. In AF E, ..
.
. ..... ... ... ... ..... ..... C
.... ........ .... . .... .... ....... ........
.. .. ....... .. ..... ....
.. ..... ...... .. ....
........ ....... ..... .... ........
....... ...
......... ...... .... F .. ... ....
AE 2 AF 2 + EF 2 . . ..
= ..........
............
..
 2  A B
= x + 100 cm2
=300 cm2 .

Therefore AE = 10 3 cm,
hence (A).
224 Geometry Solutions

5 1980 J.18 (8%), I.8 (12%), S.4 (24%)


.. ..
........ ......
...... . ......
Consider the vertices O, X, Y , Z as shown. ..... .
...... .. .....
......
...... ......
There are three other edges parallel to OX. Z .
.....

These four edges form six pairs of parallel .....


.
..
. ....... ....... ....... ....... ....... ....... .........
....
. Y ......
edges. Arguing similarly for the edges OY .....
.
.
.....
. .....
......
......
and OX gives 6 + 6 + 6 = 18 parallel pairs of O X
edges,
hence (B).

6 1980 J.29 (65%), I14 (75%), S.8 (83%)

When figure D, illustrated at right, is folded,


squares (i) and (ii) will overlap, leaving one (i) (ii)
side of the cube uncovered, ....... ....... ....... ..
..
....... ....... .......
..
..
...
hence (D). ..
...
...
.
.. ...
....... ....... ....... .

7 1980 S.22 (29%)

If the cylinder is cut at P and laid flat ... 6


..
it can be seen that P Q2 = 32 + 42 and ...
..
...
P Q = 5 cm, ..
...
..
hence (C). 4
...
.. 4
...
..
...
..
...
..
...
..

3 Q

8 1983 J.7 (85%)


The bottom face of an open box is the only face to contact every other
face at at least one vertex (in fact exactly two vertices). Thus face V is
the bottom of the box,
hence (B).
Solid Geometry 225

9 1983 S.26 (8%)


Alternative 1
Let the centres of the faces T U V W and U QRV be Y and Z
respectively. Let P X meet Y Z in A.

S ....... ....... ....... ....... ....... ............. R


.. .... ..
.... ..
.. ..
.....
. ..... ....
P Q R P .. .
.
............................ ........ ....... ....... ....... ..........
.... .
...
.. ........................
  ... ..... .......
........... .............
. .. . ...
. .............
..
.
.
Q ..
...
. .
. . .................
 ............... 
.. ..... ....... ............. ..
.............
... ..
.... .. .......
..... . ......
.... ...... .. ............. Z ...
.. . ..
   •Z . . ..
. . ..
.......
.
.
......
.. ........ ...
.
.
......
.... ....
.... A
...... ........ ..... ....

Y 
A 
Z   W ... ....... ........... ..
........ ......... ...................................... ................
.
.

..... 
...
A
.. .... ....... .. ..
. ..... .... .... ....
. .
........................................................... V
 ...
.....  ... ... ......
.. .. . X
 
.
.......... ....... ....... ....... ....... ....... .. Y .. .....

X U Y X V T U

Considering the congruent triangles contained in the quadrilateral


P Y XZ it may be deduced that A is the midpoint of Y Z. Now let Y 
and A be points such that Q, Y  , X, Z and A are the projections of P ,
Y , X, Z and A onto the face QRV U . Since QU = 2ZY  and
QU
ZY  = 2A Y  (recall that A is the midpoint of Y Z),   = 4. Also
AY
UX QU
triangles QU X, A Y  X are similar so  =   = 4. Therefore
Y X A Y
U X = 4Y  X. Thus U Y  = U X − Y  X = 3Y  X, and
XV = V Y  − Y  X = U Y  − Y  X = 2Y  X.
Therefore
UX 4Y  X
= = 2,
XV 2Y  X
hence (A).
Alternative 2
Without loss of generality, we can let

P = (0, 0, 0) Q = (2, 0, 0) R = (2, 0, 2) S = (0, 0, 2)


T = (0, 2, 0) U = (2, 2, 0) V = (2, 2, 2) W = (0, 2, 2)
226 Geometry Solutions

z
Then the midpoint Y of face T U V W is S...  ....... ....... ....... ....... ........W
...
(1, 2, 1). The midpoint Z of U QRV is .......
.
....... ...

(2, 1, 1). So the plane P Y Z has equa- R ... ............. ....... ....... ....... V........ ....... .... .. ...
..
... .
tion x + y − 3z = 0. This cuts U V at . .... ...
..
.... ...
4 4 . .
...
..
y
x = 2, y = 2, z = . Thus U X = ,
P
... .
.. .... .......
3 3 ...
...
T

.
.... .. ....
4 2 UX x
 . ....... ....... ....... ....... ....... .. ...
XV = 2 − = , and = 2, Q U
3 3 XV
hence (A).

10 1984 I.24 (7%), S.19 (12%)


First consider three spheres, each of radius 10 cm, touching each other as
in the first diagram. If their centres are at P √
, Q and R, and S is the
midpoint of QR, then the length of P S is 10 3 cm, applying
Pythagoras’ Theorem to the right-angled triangle P SR.

P .
....
. ..
... .

........
................................
......
Q ..
.... ....... ....... ....... ........ ........ ....... ....... ....... ....... ........
.. U
...... ..... . .... .. ..
..... .... ... .. ... .... ....
..... ... ... .....
. . ..
..
. ... .. .. ..... ... ..... ..
.... ...
... .. . .. ..
... P ...
... ..
.....
... . .. .....
. .....
....
... • ....
... ..
.
.. ...
.. . ... .
....
.....
.
.
..
..
.
...
..
...
... .. .. ..
. . .. .
.. .... ..
. . ... . .. ..... . .. ..
...
...
.. ...
.. .
.. .. . .. .....
..... ... . . .
. . .....
.
.. ... ..................................... .
... ....... ....... ....... ....... ................. .
.......
......................................
.......... . ................. ....... S .. . ..... V
.....
...... ............................ ..
.... .. .... ...
......
.....
... .
...
10 ..
..... ..
.....
..
..
... ... .. ... ..... ..
..
. ... . .. . ... ... .....
.... ......... . ... ... ....... .....
..
... ...... ... .. ..... ..
... ... ....
....
... • ....... ....... ....... ........ ....... ....... .
.... • .
..
..
...
...
.. ..
.....
.. ..
.....
..
..
..
...
...
...
Q .
.
....
.
. ..
. ..
S R ...
.
. ... ......
.. .
..... ...
..
... ... ..... ... . . ..
.....
..... ....
... .....
..... ....
... .......... ....... ....... ....... ....... ....... ....... ....... ....... ....... .......
....... ..... ....... ......
............ ..................
.........
............ .................
.......... R T

In the second diagram, P , Q, R, T and U represent the centres of the


five spheres, while S again represents the midpoint of QR. If V is the
midpoint of the square QRT U , it can be seen, applying√Pythagoras’
Theorem to triangle P SV , that the length of P V is 10 2 cm. Since the
point V√is a further 10 cm above the table, the total height of P is
10(1 + 2) cm,
hence (B).
Triangles 227

TRIANGLES
1 1983 J.5 (86%)
x = 180 − 50 − 60 = 70,
hence (A).

2 1978 I.6 (22%)


..
Two such set-squares placed together as shown  ... ....
form an equilateral triangle. Thus the longest side  .
...
30◦ ..

has length 2 × 12 = 24 cm,  ...


..
 ...
..
hence (E). 60◦ ........
....... .......60
◦ ..
....... ...

3 1979 J.14 (44%)


Regarding QR as the base of P QR, then the perpendicular height is
the side length of the square ABCD, i.e. 10 cm. Thus
1
AreaP QR = × 5 × 10 cm2 = 25 cm2 ,
2
hence (C).

4 1982 J.19 (13%), I.13 (17%), S.8 (24%)

The seven matches can be grouped into ....•


... •............
1........... .....1
.....
..
the following lengths: (1,1,5), (1,2,4), •...... • • • • ..•...
5
(1,3,3) or (2,2,3).
The first two do not form triangles as the sum of the lengths of the two
smaller sides is not greater than the length of the third side. Thus only
two different triangles exist,
hence (C).

5 1979 J.19 (17%), I.9 (23%), S.6 (46%)


Noting that the interior angles of the triangle are 180◦ − a, 180◦ − b and
c we have
180◦ − a + 180◦ − b + c = 180◦ ,
i.e. 180◦ = a + b − c,
hence (C).
228 Geometry Solutions

6 1981 I.21 (25%), S.14 (29%)

Let h, x, y be the distances as shown on T


the diagram. Triangles P RQ and T RV 
are similar, 
y
x 20 1  80
= = . 
y 80 4 R
P
.. 
20 x .. 

Therefore  .h
.... 
Q S V
QR x 1 1
= = = .
QT x+y 1+4 5

h QR 1
Triangles QRS and QT V are similar. Therefore = = , i.e.
80 QT 5
80
h= = 16,
5
hence (C).

7 1984 J.27 (5%), I.27 (6%), S.24 (7%)


There are triangles of side length:

3: PV Y
√2: P SU , QV X, RW Y
3: QU W , SRX
1: P QR, QST , QRT , RT U , SV W , SW T , T W X, T U X, U XY

Thus the total is 1 + 3 + 2 + 9 = 15,


hence (E).
Triangles 229

8 1981 S.29 (6%)

Construct the lines T A, T D, SB, SE as P


shown. Let EQ and ES have lengths x cm and 

... ..
y cm respectively. Triangles P AT , T CS and
SEQ are congruent (ASA). Therefore AT =
.....

A....... ....... ...........T

..
... ...
CS = EQ = x. Therefore D and E tri- 9...... ... .....
... .
. 

..
sect RQ. Similarly, A and B trisect P R. In ..... ..
...
....... .
B ....... ........... ........C....... ................S
SRE, RE 2 + ES 2 = 72 , i.e. .
.
..
.. ... .........
.....
.
...
. 
. 7 ....
.
...
... .......... ..
... .......
........ ...
.

y ......

4x2 + y 2 = 49. (1) .


..
............. .......
. ..
. . ...
.
..
x

R D E Q

Similarly, from AT R,


x2 + 4y 2 = 81. (2)
2 2 2 2
(1) + (2) gives 5x + 5y = 130, or x +√y = 26. But
ST 2 = SQ2 = x2 + y 2 = 26, i.e. ST = 26,
hence (C).

9 1982 S.28 (6%)

Alternative 1 Q
. ...
◦ .....
.
Let X and Y be points as shown. In QXS, X.. 45 ...
 XQS = 45◦ (given), so  QSX = 45◦ . ........

..
 ...
..
Thus the triangle is isosceles and  ....... ....... S .....
. ......

 . .......
........ ......
 .....
QX = SX. (1) .. .
.. ........ 45◦ ....
..... ... .
P ◦ Y R
45

Also P XR is isosceles, so

RX = P X. (2)

Further,
 P XS = 90◦ =  RXQ. (3)
From (1), (2) and (3) it follows that P XS and RQX are congruent
(SAS). Thus RQ = P S = 20,
hence (A).
230 Geometry Solutions

Alternative 2 Q,S
... ...

The position of S is not specified in the ....


 
question and so may be assumed to coin-  
cide with Q. The P SR is isosceles, so 45◦ 45◦
P R
QR = SR = SP = 20,
hence (A).

TRIGONOMETRY
1 1978 S.5 (39%)
Alternative 1
2rs
A triangle with tan A = is as

r 2 − s2

shown. Using Pythagoras,

 2rs
2
(r2 − s2 ) + (2rs)2 A  ........

 r 2 − s2
= r4 + s4 + 2r2 s2
= r 2 + s2 ,
r 2 − s2
i.e. cos A = .
r 2 + s2
hence (E).
Alternative 2
 2
2 2 4r2 s2 r 2 + s2
Since sec A = 1 + tan A = 1 + 2 = ,
(r2 − s2 ) r 2 − s2
 2 2
r − s2
cos2 A = , and since A acute requires cos A > 0,
r 2 + s2
r 2 − s2
cos A = 2 ,
r + s2
hence (E).
Volume 231

2 1983 S.12 (33%)


 


 
 y
y = 50 tan 45◦ metres,  
45◦ ..  x
. 
= 50 metres.
.........

...


...

x = 50 tan 30◦ metres,  30◦ ..... 


50m
50
= √ metres.
3
 
1
Thus the height of the cliff is (y − x) metres, i.e. 50 1 − √ metres,
3
hence (E).

3 1984 S.13 (10%)


 
◦ 1 1
tan 45 − tan arctan 1−
 ◦ 1
 2 2 1
tan θ = tan 45 − arctan 2 =  = = ,
1 1 3
1 + tan 45◦ tan arctan 1+
2 2
hence (D).

VOLUME
1 1983 J.13 (19%)
If the perimeter of the base is 18, the length and width must add to 9,
giving only the choices (1,8), (2,7), (3,6) and (4,5). The only choice of
these for length and breadth which have a product which is still a factor
of 42 is (2,7), leaving the third factor (and height) as 3,
hence (A).
232 Geometry Solutions

2 1982 I.18 (10%), S.11 (41%)

If r is the radius of the tennis balls, the required ......


2r
............................
.....
.... ...
cylinder must have height 6r and radius r. Its volume ... ...
.... ...
.... ...
will be πr2 (6r) = 6πr3 . The volume of the 3 balls is ...
...
... ...
..
.
.
4 ....
.....
.....
.....
3 × πr3 = 4πr3 . The required ratio of volumes is ........
........
............................
......
3 .....
..... .....
...
...
4πr3 2 ....
... ...
..
= , 6r .... ..
..
6πr3 3 ...
...
..... ..... ....

hence (B). ......


..............................
... ........
.
....... ......
..... ....
.... ...
... ...
.. ...
.... ...
... ...
... ..
... .. .
..... .....
......
............................

3 1981 S.18 (27%)


2 2
The cylinder has a base area of 1 m = 10 000 cm . If the water level
rises h cm after the immersion of the cube, the volume of the cube is
10 000h cm2 . Thus 10 000h = (20)3 = 8000 or h = 0.8,
hence (C).
OTHER TOPICS SOLUTIONS

COUNTING TECHNIQUES
1 1978 J.21 (23%)
Alternative 1
Each team plays each of the eight other teams twice and then plays 16
matches. There are nine such teams, but because each match is played
9 × 16
between two teams, the total number of matches played is = 72,
2
hence (D).
Alternative 2
Regard each team as playing each other team once at home and once
away. Then each team plays exactly 8 home matches (and every match
is at home for exactly one of the teams). Thus the total number of
matches is the total number of home matches, i.e. 8 × 9 = 72,
hence (D).
Alternative 3  
9 2×9×8
The number of matches equals 2 × = = 72,
2 2×1
hence (D).

2 1979 J.22 (47%), I.21 (67%)


Alternative 1
The 127 players other than the winner each lose precisely one match.
Thus 127 matches will be played,
hence (E).
Alternative 2
To reduce the field to 64 players, 64 matches are required. To further
reduce it to 32 players, 32 more are required, and so on until only one
player remains. Hence the total number of matches is
64 + 32 + 16 + 8 + 4 + 2 + 1 = 127,
hence (E).
234 Other Topics Solutions

3 1979 J.23 (6%), I.24 (6%)


In a triangle the length of the largest side, call it , is less than the sum
of the other two sides, or equivalently less than half the perimeter.
Hence we have  < 6 12 , or  ≤ 6, as side lengths are integers. If  = 6 cm
the other possible side lengths are 5 cm and 2 cm or 4 cm and 3 cm. If
 < 6 cm, that is,  ≤ 5 cm, the other sides must sum to 8 cm or more.
The second largest side then has length 5 cm or more and so is not
second largest. Therefore (6,5,2) and (6,4,3) are the only solutions,
hence (B).

4 1979 S.26 (34%)


Pages 1 to 9 use 9 digits. Pages 10 to 99 use 90 × 2 = 180 digits. The
remaining pages (from 100 onwards) use 852 − 180 − 9 = 663 digits.
Hence the number of further pages is 663 ÷ 3 = 221, and the total
number of pages is 99 + 221 = 320,
hence (A).

5 1980 J.30 (30%), I.29 (39%), S.27 (54%)

The numbers at each vertex on the di- P


agram represent the number of routes 
to that vertex from P : A 
1 
B 1
 
• To each of A and B there is only C 
1 
D 
2 1
one route.   

1 
E 
3 3 1
• To C there is only one route (via A    
4 6 4
to which there is only one route).   
10 10
• D can be reached in 1+1 = 2 ways,  
either via A or via B (to each of 20
which there is only one route). Q

• E can be reached in 2 + 1 = 3 ways (2 via D and one via C).

Continuing in this manner shows that 20 distinct paths lead to Q,


hence (D).
Note: The method demonstrated here can be applied to any diagram, of
arbitrary complexity, subject to the same restriction of downward
Counting Techniques 235

motion only. The similarity of construction of these numbers and the


numbers of Pascal’s triangle should be noted.

6 1980 I.12 (21%)


Alternative 1
Each of the 20 people took part in 19 handshakes. As each handshake
involves two people, the total number of handshakes is
20.19
= 190,
2

hence (A).
Alternative 2
The number of handshakes is equal to the number of combinations of 20
objects, taken two at a time,
20.19
C220 = = 190,
2.1

hence (A).

7 1981 J.13, (27%), I.8 (37%)

In the diagram squares of side length 4, 3, 2 and a c g


1 cm can be found. Clearly there are sixteen 1 cm
squares. Specifying the others by the letter in the b d h
top left-hand corner gives:
e f i
nine 2 cm squares (a, b, c, d, e, f, g, h, i),
four 3 cm squares (a, b, c, d), and
one 4 cm square (a).
Thus the total number of squares = 16 + 9 + 4 + 1 = 30,
hence (E).
Note: Systematically describing the squares in this way makes it easy to
see that the number of squares in an n cm×n cm square is
12 + 22 + · · · + n2 .
236 Other Topics Solutions

8 1981 J.17 (8%), I.13 (17%)


Any one of the three numerals (4, 7, 8) can be used to make up each of
the hundreds, tens and units digits of the house number. This gives
3 × 3 × 3 = 27 different possibilities,
hence (E).

9 1981 J.18 (35%)

The dominoes can be systematically listed on a 0 1 2 3 4 5 6


grid as shown. The crosses represent dominoes. 0 ×
There are 28 of them, 1 ××
hence (E). 2 ×××
3 ××××
4 ×××××
5 ××××××
6 ×××××××

10 1981 J.23 (47%), I.19 (67%)


Alternative 1
Suppose n people are present. Each of these takes part in n − 1
handshakes. As each handshake involves two people the total number of
n(n − 1)
handshakes is = 28, i.e. n(n − 1) = 56. As n is an integer this
2
can be solved by inspection to give n = 8,
hence (B).
Alternative 2
If the first of the n people shakes hands with each of the others and then
stands aside, there will be (n − 1) handshakes. If another repeats the
process there will be (n − 2) handshakes. The total is
(n − 1) + (n − 2) + · · · + 2 + 1. By trial, 1 + 2 + 3 + 4 + 5 + 6 + 7 = 28, so
8 are present,
hence (B).

11 1981 J.27 (7%), I.28 (9%), S.27 (11%)


Alternative 1
If we consider the 24 arrangements of the numbers 1, 2, 3, 4 and cross
off all those with 1 in the first place (i.e. the first letter in its proper
Counting Techniques 237

envelope), 2 in the second place, 3 in the third place, or 4 in the fourth


place, we have left these nine arrangements:

2 1 4 3 2 4 1 3 2 3 4 1
3 1 4 2 3 4 1 2 3 4 2 1
4 1 2 3 4 3 1 2 4 3 2 1

Therefore there are nine different ways,


hence (B).
Alternative 2
The problem is a special case of the topic called derangements. A
derangement is an arrangement of the numbers 1, 2, . . . , n such that no
number appears in its original position. For example, 23514 is a
derangement of 12345 whereas 23541 is not. The following formula gives
the number of derangements D(n), of the numbers 1, 2, 3, . . . , n.
 
1 1 1 1
D(n) = n! 1 − + − + · · · + (−1)n ,
1! 2! 3! n!

where n!, known as n factorial, equals n(n − 1)(n − 2) × · · · × 3 × 2 × 1


(for example, 4! = 4 × 3 × 2 × 1 = 24). For a simple explanation of this
formula see Niven [23].
So the ‘careless office boy with four letters’ problem is a special case of
the above formula with n = 4, where
 
1 1 1 1
D(4) = 4! 1 − + − +
1! 2! 3! 4!
 
1 1 1
= 24 1 − 1 + − +
2 6 24
= 9,

hence (B).

12 1981 J.29 (5%), I.29 (6%), S.28 (7%)


There are six possible orders of preference. The dissenting judge may be
one of 3 and has 6 different ways to vote. The other pair has 5 ways to
vote. Therefore the total number of ways to vote is 3 × 6 × 5 = 90,
hence (B).
238 Other Topics Solutions

13 1983 J.16 (26%)


Alternative 1 (by listing)
With 1 coin we can get four amounts, i.e. 1c, 2c, 5c, 10c.
With two coins we can get 6 amounts, i.e. 1c + 2c = 3c, 1c + 5c = 6c,
1c + 10c = 11c, 2c + 5c = 7c, 2c + 10c = 12c and 5c + 10c = 15c.
With 3 coins we can get 4 amounts (one with each coin missing), i.e.
1c + 2c + 5c = 8c, 1c + 2c + 10c = 13c, 1c + 5c + 10c = 16c,
2c + 5c + 10c = 17c.
With 4 coins we can only get 1c + 2c + 5c + 10c = 18c.
All of these amounts are different. The answer is 4 + 6 + 4 + 1 = 15,
hence (B).
Alternative 2
Each of the 4 coins can either be chosen or not chosen (i.e. there are two
choices). This gives 2 × 2 × 2 × 2 = 24 possibilities. But we must remove
the one possibility in which no coin is chosen. Therefore the answer is
24 − 1 = 15,
hence (B).
Note: It is necessary to be satisfied that all of the possible amounts are
different.

14 1983 J.24 (7%)


Suppose the students taking home the wrong trumpet are called A, B
and C. These can take the wrong trumpets in two ways, e.g. A takes
trumpet B, B takes trumpet C and C takes trumpet A, or A takes
trumpet C, B takes trumpet A and C takes trumpet B. We need also to
know how many ways A, B and C can be chosen from the five. This is
the same as the number of ways in which the two with the right
trumpets can be chosen, this being ten (e.g. if the students are called A,
B, C, D and E these are
A and B, A and C, A and D, A and E,
B and C, B and D, B and E,
C and D, C and E, and
D and E.
Thus the answer is 10 × 2 = 20,
hence (D).
Counting Techniques 239

Generalisation to this question:


In the following discussion,
n factorial, means n(n − 1)(n − 2) · · · 3 × 2 × 1,
n!, 

n n!
= is the number of combinations possible from choosing
r r!(n − r)!
r objects from a set of n (without regard to order),
D(r) is the number of derangements of the numbers 1, 2, . . . , r, that is,
arrangements of the numbers 1, 2, . . . , r so that no number appears in its
original position. It has been established that
 
1 1 1 r 1
D(r) = r! 1 − + − + · · · + (−1)
1! 2! 3! r!

(see Niven).
It is now easily established that if there are n children, each owning
their own trumpets, the  number of ways in which exactly r take home
the wrong trumpet is nr × D(r). For n = 5, the solutions can be
tabulated as
   
5 5
r D(r) × D(r)
r r
0 1 1 1
1 5 0 0
2 10 1 10
3 10 2 20 (the solution to our problem)
4 5 9 45
5 1 44 44

Note: The sum of the solutions, 1 + 0 + 10 + 20 + 45 + 44 = 120 or 5!, is


the total number of ways of allocating 5 trumpets to 5 children.

15 1983 J.28 (5%)


The number of questions earning the maximum of 3 marks must be 3, 4
or 5. (With only 1 or 2 the total would be less than 15.) Consider each
of these cases.

3: This makes up 9 points and the other three questions must each
earn 2 points. There are 20 combinations of questions earning 3
240 Other Topics Solutions

points each, namely

123 134 145 156 234 245 256


124 135 146 235 246
125 136 236
126

345 356 456


346

4: The other two questions must yield 1 and 2 points. Any one of 6
questions could have earned 1 point and any one of the 5 could
have earned 2 points giving 6 × 5 = 30 such combinations.

5: The remaining questions earn no points. There are 6 ways of scoring


in this way.

Thus the total number of ways is 20 + 30 + 6 = 56,


hence (C).

16 1983 S.29 (9%)


Alternative 1
Call the socks aa, bb, cc, dd.
There are 4 × 3 × 2 = 24 ways of selecting the first three as abc (i.e. the
first three different).
Counting Techniques 241

These can be arranged as shown:

abc a bd
cdbd
dbdc
cd
cdb
bd 6
b symmetric
with a 6
da bcd
dc
cbd
db
dbc
cb 6
db symmetric
with da 6
dc symmetric
with da 6 30

So there are 24 × 30 = 720 patterns commencing with abc.


There are 4 × 3 = 12 ways of selecting the first three as aba.
These can be arranged as shown:

aba b cdcd
dcdc 2
c bdcd
dbcd
bdc
cbd
cdb 5
d symmetric
with c 5 12

So there are 12 × 12 = 144 patterns commencing with aba.


Finally, 720 + 144 = 864,
hence (D).
242 Other Topics Solutions

Alternative 2
(Uses inclusion-exclusion principle)
This solution has the advantage of generalising the problem to n pairs of
socks when n = 4.
Consider the problem for 2 pairs of socks, as illustrated in the Venn
diagram.

.............................................. ..............................................
......... ....... ......... .......
....... ...... ....... ......
A1................... ..... ...........
..... ....
........
..... A
..... 2
.....
.... .. . ... ...
.
... .
... .... ...
...
... .. ... ...
. . .
...
... ... ... ...
.... ...
.... ... ...
... ... ... ...
... ... ... ...
.... .... .. .
... ... .... ...
... ... ... ....
... ... .. ..
...
...
...
... ... ...
... ... ... ...
... ..
...
...
...
... ..... ....
.
.
...
..... ... ....
... ...
..... ......... .....
..... ..... ......... .....
......
....... . . .. ...... ...... .
..........
..... . ....... .
.........
.............................................. .......... .......
..........................................

The set A1 , for instance, is the set of arrangements in which pair 1 is


together.
We wish to compute
2

N (A1 ∪ A2 ) = N () − N (Ai ) + N (A1 ∩ A2 ).
i=1

In this problem
4!
N () = .
22
(We divide by 22 because the socks within each pair can be changed
without changing the arrangement.)
Now
3!
N (A1 ) = N (A2 ) = 1
2
and
2!
N (A1 ∩ A2 ) = 0 ,
2
Counting Techniques 243

so the solution is
4! 3!
− 2 + 2 = 6 − 6 + 2 = 2.
4 2
The inclusion-exclusion principle generalises to give, in the n−pair case
 n  n
  
N Ai = N () − N (Ai ) + N (Ai ∩ Aj )
i=1 i=1 i=j

− N (Ai ∩ Aj ∩ Ak )
i=j=k
 n


n
+ · · · + (−1) N Ai .
i=1

In the case of 3 pairs, this gives


6! 5! 4!
3
− 3 2 + 3 1 − 3! = 90 − 90 + 36 − 6 = 30.
2 2 2

In the case of 4 pairs, this gives


8! 7! 6! 5!
− 4 3 + 6 2 − 4 + 4! = 2520 − 2520 + 1080 − 240 + 24
24 2 2 2
= 864,

hence (D).
In the case of n pairs this gives
       
n (2n)! n (2n − 1)! n (2n − 2)! n n n!
− + + · · · + (−1) .
0 2n 1 2n−1 2 2n−2 n 2n

Note that the first two terms always cancel each other in this particular
problem.
244 Other Topics Solutions

17 1984 J.13 (13%), I.7 (18%)


Let the members of couple A be A1 , A2 . Let the members of couple B
be B1 , B2 . The arrangements are

A1 A2 B1 B2 B1 B2 A1 A2
A1 A2 B2 B1 B1 B2 A2 A1
A2 A1 B1 B2 B2 B1 A1 A2
A2 A1 B2 B1 B2 B1 A2 A1

i.e. 8 arrangements altogether,


hence (E).

18 1984 J.30 (8%), I.30 (8%), S.29 (5%)


 
4
Since , the number of ways of selecting 2 objects from an available
2
4×3
4, is = 6, the 4 × 6 array can be coloured as shown to give no
2×1
rectangle with all four corners the same colour.

Y Y Y G G G
Y G G Y Y G
G Y G Y G Y
G G Y G Y Y

No larger rectangle can be so coloured. Suppose it could. If it has one


column of 4 green points, then no other column can contain more than
one green point. Therefore more than 2 columns with more than 3
yellow points implies a yellow rectangle for n ≥ 3, a contradiction.
Now suppose that no column has 4 points coloured the same, but that
one contains 3 green points. No other column can contain 3 green
points; at most 3 other columns can contain 2 green points; no other
column is possible without forming a yellow rectangle. Hence n ≤ 4, a
contradiction. Hence each column has two points of each colour (as
shown),
hence (D).
Logic 245

LOGIC
1 1978 J.25 (55%), I.19 (59%), S.10 (67%)
The statement ‘All citizens have two legs’ is false. Therefore ‘Not all
citizens have two legs’ is true. Statements I, III and IV need not be true,
hence (A).

2 1984 J.29 (9%), I.28 (13%), S.23 (20%)


Let a single-headed arrow represent ‘. . . says . . . is a kangaroo’ and a
double-headed arrow represent ‘. . . says . . . is a frog’. Then we have

 A
E 





B


D  C 
 

Assume Ellie is a kangaroo, and hence that his statement is true.


Thus: Albert is a kangaroo,
Bernard is a kangaroo,
Charles is a frog and
Daniel is a kangaroo.
But this is not possible since Ellie and Albert are then both kangaroos,
contrary to Daniel’s statement. This proves that Ellie is not a kangaroo,
but a frog instead. Being a frog, Ellie’s statement is false.
Thus: Albert is a frog,
Bernard is a frog,
Charles is a kangaroo and
Daniel is a frog.
There are 4 frogs,
hence (D).
246 Other Topics Solutions

PROBABILITY
1 1984 S.25 (5%)
The optimal way of distributing the balls is as follows:

16W
1W 1W
6B

Box 1 Box 2 Box 3

The probability of drawing a white ball is


   
1 1 1 16 1 8 10
+ + = 2+ = ,
3 3 3 22 3 11 11

hence (C).
Note: The following argument demonstrates that the above distribution
of balls is in fact optimal. The chance of drawing a particular ball must
be
1 1
≥ × .
3
 22

probability of probability of choosing
choosing the the ball within the
correct box box if the box
contains the maximum
possible number of balls

Since there are 6 black balls, in any distribution the chance of drawing a
black ball in the manner described must be
1 1 1
≥6× × = .
3 22 11
Since this is in fact the probability of drawing a black ball in the above
distribution, that distribution must, indeed, be optimal.

2 1982 I.29 (3%), S.29 (3%)


Let C1 be card 1 (drawn first) and C2 be card 2 (drawn second).

N1 = total number of pairs which include exactly one ace


Sets 247

= number of pairs for which (C1 = one of four aces, C2 = one of


12 non-aces) or (C1 = one of 12 non-aces,
C2 = one of four aces)
= (4 × 12) + (12 × 4) = 96.
N2 = total number of pairs for which both cards are aces
= number of pairs for which (C1 = one of four aces, C2 = one of
three remaining aces)
= 4 × 3 = 12.

Therefore the probability of 2 aces given that at least one is an ace


N2 1
= = ,
N1 + N 2 9

hence (E).

SETS
1 1978 I.16 (7%)
........................................... ........
..........................................
........ ......
...... ...........
......
Suppose x% read all 3. Then the A ...... ......
B
..... .......... .....
........ .
.... ....
... ... ..... ...
given data allows the values 15, x, ...
...
.
...
..
.
.
...
...
...
...
...
...
.... .... ...
19 − x, 20 − x and 17 − x to be en- ... 15 .
.
...
....
...
...
.... .....
...17−x.... ...
tered as shown. Since 34% read A, ...
...
...
...
. . ..... .. ... ... .....
...
. ...
..
... ...........
. .. ..
................ .
.
..... ... x .. .. ..
... ...... ...... .
... ..... ... ... ..... ...
... ..... ... ... .....
.... ...
15 + (17 − x) + x + (19 − x) = 34. .....
..... ..
.
....
..
....
............. ...
. .
......
...
... 19−x ...... 20−x ..
...... . .. . ..
....... ...... ... ............
.
.............. ...... .......
................................ ........... .
................................
... ...
....
Hence x = 17. ...
...
..
... ...
... ...
..
Suppose c% read C only. Since 52% ...
...
... ...
.
... ..
...
read C, ....
.....
..... ........
...... .....
........ ...... C
............... .....................
........
(19 − x) + x + (20 − x) + c = 52.

Thus
2 + 17 + 3 + c = 52.
Therefore c = 30,
hence (B).
248 Other Topics Solutions

TRIGONOMETRIC IDENTITIES
1 1979 S.9 (23%)

For all y sin y ◦ + sin(x − y)◦ = sin x◦


⇔ for all y sin y ◦ + sin x◦ cos y ◦ − cos x◦ sin y ◦ = sin x◦
⇔ for all y (1 − cos x◦ ) sin y ◦ + sin x◦ (cos y ◦ − 1) = 0.
The statement is true for all y provided 1 − cos x◦ = 0 and sin x◦ = 0.
These equations are satisfied by x = 0, 360, . . .,
hence (E).
Note: Substitution shows that (E) is correct, for
sin y ◦ + sin(360 − y)◦ = sin y ◦ − sin y ◦ = 0 = sin 360◦ .

2 1980 S.16 (23%)


 2
cos θ sin θ
(cot θ + tan θ)2 = +
sin θ cos θ
 2
cos2 θ + sin2 θ
=
sin θ cos θ
 2
1
=
sin θ cos θ
= cosec2 sec2 θ,
hence (A).

3 1982 S.23 (10%)


Alternative 1
Making use of the trigonometric identities:
cos θ = sin(90◦ − θ)
C +D C −D
and sin C + sin D = 2 sin cos ,
2 2
we have
sin 6x + cos 4x = sin 6x + sin(90◦ − 4x)
= 2 sin(45◦ + x) cos(5x − 45◦ )
= 0
Miscellaneous 249

when
sin(45◦ + x) = 0 or cos(5x − 45◦ ) = 0.
Looking for the minimum positive x value: 45 + x = 180 or
5x − 45 = 90, i.e. x = 135◦ or x = 27◦ ,
hence (A).
Alternative 2
Trialling the various alternatives, starting with the smallest:

If x = 9◦ , sin 6x + cos 4x = sin 54◦ + cos 36◦ = 0.


If x = 18◦ , sin 6x + cos 4x = sin 108◦ + cos 72◦
= sin 72◦ + cos 72◦ = 0.
If x = 27◦ , sin 6x + cos 4x = sin 162◦ + cos 108◦
= sin 18◦ − cos 72◦
= cos 72◦ − cos 72◦ = 0,

hence (A).

4 1984 S.17 (10%)


cos 12 − sin 18 = cos 12 − cos 72 = 2 sin 42 sin 30 = sin 42◦ ,
◦ ◦ ◦ ◦ ◦ ◦

hence (A).

MISCELLANEOUS
1 1983 J.17 (58%), I.7 (75%)

A shortest route is shown. It travels P .... ...................................... .............................


......... ................................................
. ............................. ................................................ ......................................
along 10 edges. Answer = (10×10) cm = ... ...............................................
.. ................................................
......................................
......................................
......................................
......................................
. . . . .. . . . . . . . .. . . . . . . . . . . . . . . . . . . . . . . . ...................
100 cm, ..... ....... ... ...................
.. .............................
......................................
................................................
......................................
.... ............................................... ......................................
hence (E). .............................
. ...................
......................................
...................
.................................................. ....... ..... ................................................ .. ....... ..... ................................................
...................................... ...................................... .. ............................
...................................... ... ...................................... . ... ................................................
...................................... . ............................ ...
............................ .
. ............................
...................
................... .
..... ....... ....... ............................
Q

2 1980 S.11 (50%)


There are nine 2-digit palindromic numbers (11, 22, . . . , 99). Further,
any 2-digit number uniquely corresponds to a 3-digit palindromic
number by repeating the first digit as the last digit (e.g. 36 gives 363).
There are ninety 2-digit numbers (10, 11, 12, . . . , 99). Thus there are
250 Other Topics Solutions

9 + 90 = 99 palindromic numbers from 10 to 1000,


hence (D).
Note: The number of 3-digit numbers can also be found by noting that
there are 9 possibilities for the first digit (1, 2, . . . , 9) and 10 possibilities
for the middle digit (0, 1, 2, . . . , 9). Thus there are 9 × 10 = 90 such
numbers.

3 1982 J.27 (8%), I.19 (10%)


Let us number the days of the week from Day 1 to Day 7. We will
require 14 different calendars to cater for each of the seven possible
starting days of leap and non-leap years.
A cycle of 4 years contains (366 + 3 × 365) days or 208 weeks and 5
days. If 1972 (the first leap year of the collection) starts with Day 1
then the starting days of subsequent leap years are 1976—Day 6,
1980—Day 4, 1984—Day 2, 1988—Day 7, 1992—Day 5, 1996—Day 3. It
is now enough to note that the calendars of the years immediately
preceding the leap years will provide the remaining calendars: 1971
begins with Day 7, 1975—Day 5, 1979—Day 3, . . ., 1995—Day 2. (In
fact all non-leap year calendars are collected by 1978.) Thus the
collection of calendars can cease in 1996,
hence (D).

4 1980 J.24 (54%)


Tabulating the possibilities gives:

Initial Annual Annual


height growth measurements
(m) (m) (m)
x y
(A) 8 2 8, 10, 12, 14, 16, 18, 20, 22, 24, . . .
(B) 5 7 5, 12, 19, 26, . . .
(C) 6 5 6, 11, 16, 21, 26, . . .
(D) 5 3 5, 8, 11, 14, 17, 20, 23∗, . . .
(E) 2 6 2, 8, 14, 20, 26, . . .

hence (D).
Miscellaneous 251

5 1979 J.28 (5%), I.13 (9%)


Alternative 1
In t hours the minute hand of a clock travels 360t◦ while the hour hand
travels 30t◦ . If T hours is the time between one coincident position of
the hands and the next such position, then in this interval the minute
hand will have travelled precisely one revolution, or 360◦ , further than
12
the hour hand. Hence 360T − 30T = 360, or 330T = 360, giving T = .
11
12
The number of such intervals in 24 hours is 24 ÷ = 22,
11
hence (E).
Alternative 2
The hands pass once during every hour (i.e. 3pm–4 pm, 4 pm–5 pm, and
so on) except during the two-hour periods 11 pm–1 am and 11 am–1 pm
when they only pass at the 12.00 position. Hence the hands pass 11
times in 12 hours, or 22 times in 24 hours,
hence (E).

6 1980 J.16 (67%)


The elapsed time from 1 pm to 10 am the next day is 21 hours. The lost
time over this period is (21 × 3) = 63 min =1 hour and 3 minutes. Thus
the clock will read 8:57 am,
hence (D).

7 1983 J.23 (13%)


One electronic device ‘bips’ each minute exactly. The other electronic
device ‘bips’ at 10:01:02, 10:02:04, . . ., 10:29:58, 10:31:00,
hence (B).

8 1982 J.30 (9%), I.28 (9%), S.27 (10%)


Note that 12 hours is 720 minutes. The first watch will tell the correct
time after it has gained 720 minutes, or multiples of this. At 1 minute
720
per day this takes = 720, or 1440, or 2160, or . . . days. Similarly,
1
the second watch shows the correct time after it has lost 720 minutes, or
1 720
multiples of this. At 1 minutes per day this takes = 480, or 960,
2 1
1
2
252 Other Topics Solutions

or 1440, or . . . days. Thus the first time they together give the correct
time is after 1440 days,
hence (E).

9 1980 I.30 (12%), S.29 (13%)


Let the period between changes be n minutes. As the light is off at 9:09,
n ≤ 9. Further
(a) light off at 9:09 ⇒ n = 2, 4

(b) light on at 9:17 ⇒ n = 1, 3, 5, 9

(c) light on at 9:58 ⇒ n = 6, 8

Thus n = 7, and the light is on at the intervals 9:00–9:07, 9:14–9:21,


9:28–9:35, 9:42–9:49, 9:56–10:03, 10:10–10:17, 10:24–10:31, 10:38–10:45,
10:52–10:59, 11:06–11:13, 11:20–11:27, . . .,
hence (A).

10 1979 S.24 (55%)


The given information can be displayed in the following table:

Person Correct time Time shown on watch Time estimated


by owner
Bill 6 pm 6.10 pm 6.15 pm
Joanna 6 pm 5.55 pm 5.45 pm
Harriet 6 pm 6.05 pm 6.15 pm
John 6 pm 5.50 pm 5.40 pm

Thus at 6 pm Joanna and John will not have reached the station and
will miss the train,
hence (C).

11 1982 J.29 (11%), I.26 (13%), S.20 (12%)


Alternative 1
Denote the number of outcomes where Gregory leads on the first lap
and Michael on the second as GM ; and similarly for GG, M G and
M M . Now note that each of the first two sentences imply that
Miscellaneous 253

M M = 0. From the other information we have

M G − GM = 9 (1)
GM + GG = 7 (2)
MG + GG = 6 (3)
(1) − (2), M G − GG = 2 (4)
(3) + (4), 2M G = 8
MG = 4
From (1), GM = 5
From (2), GG = 2

The number of challenge races was


M G + GM + GG + M M = 4 + 5 + 2 + 0 = 11,
hence (C).
Alternative 2
Use the same notation as in Alternative 1. It is not necessary to compute
each of M M , M G, GM and GG. As before we note that each of the
first sentences implies that M M = 0. The third sentence then says that
the number of laps won by Michael equals 9. The fourth sentence says
that the number of laps won by Gregory equals 13. The total number of
22
laps is thus 9 + 13 = 22, so the number of challenge races is = 11,
2
hence (C).

12 1981 J.28 (18%), I.26 (21%), S.24 (33%)


Alternative 1
This is a round robin situation in which each team plays each other
team, and the question can be answered by constructing a win-loss-draw
table as follows, showing the details of each individual match.
First, since Falcons had two draws, and Condors and Eagles had one
draw each, the draws took place in the Falcons v Condors and Falcons v
Eagles matches. The Falcons’ remaining match was a win, so this must
have been against Hawks. Also, we are given that Hawks defeated
254 Other Topics Solutions

Condors. The information can be represented in the following table:

F C E H
.............................
......................................
......................................
Falcons ......................................
......................................
D D W
.............................
......................................
......................................
Condors D ......................................
......................................
L
.............................
......................................
......................................
Eagles D ......................................
......................................
.............................
......................................
......................................
Hawks L W ......................................
......................................

The remaining information can now be filled in by noting the final


league table. Condors had one win so this must have been against
Eagles. Hawks had a win and two losses, so must have lost to Eagles.
This also gives us the two missing Eagles results,
hence (E).
Alternative 2
The table shows that Falcons drew against Condors and Eagles. We are
given that Hawks win against Condors. This is displayed on the diagram
with a double line denoting a draw and a directed line from loser to
winner.

F C F C

 

 


 
   
E H E  H

Working down the table again it is easy to complete the diagram. Thus
Eagles were undefeated except against Condors,
hence (E).

13 1983 I.27 (6%)


There are 25 presidential elections during the 20th century, that is, a
Tuesday in each of 1904, 1908, . . ., 2000. These dates can range from 2
Miscellaneous 255

November to 8 November and all will coincide, except the 8th, since
then the Melbourne Cup is run on the 1st. In 1983 the Melbourne Cup
is run on 1 November, thus in 1984 this will be held on 6 November (it
would have been 7 November if 1984 were not a leap year).
This will coincide with a US presidential election date. It is easy to
establish a cycle of dates for US presidential elections, since for the
range of years considered each is separated by 4 years including exactly
one leap year.
To obtain the date for the following election either add 2 or subtract 5,
whichever gives a number in the range 2 ≤ x ≤ 8. (Going backwards
subtract 2 or add 5.) This gives the following table for presidential
election dates:

Year Date Year Date Year Date Year Date


1904 8 1932 8 1960 8 1988 8
1908 3 1936 3 1964 3 1992 3
1912 5 1940 5 1968 5 1996 5
1916 7 1944 7 1972 7 2000 7
1920 2 1948 2 1976 2
1924 4 1952 4 1980 4
1928 6 1956 6 1984 6

21 of these 25 dates are in the range 2 ≤ x ≤ 7,


hence (E).

14 1981 J.25 (17%), I.24(18%), S.23 (19%)


From 1 January 1901 to 1 January 1981 there are 80 years. Of these 20
are leap years and 60 are not. The number of elapsed days is:
20 × 366 + 60 × 365 = 29 220 = (4 147 × 7) + 2. Thus 4 147 weeks and 2
days of the century elapsed before Thursday 1 January 1981, making a
Tuesday the first day of the century,
hence (A).
GENERAL REFERENCES

1. Andreescu T and Feng Z, 101 Problems in Algebra: From the


Training of the USA IMO Team, AMT Publishing, Canberra,
2001.
2. Artino RA, Gaglione AM and Shell N, The Contest Problem Book
IV, Mathematical Association of America, 1983.
3. Atkins WJ, Problem Solving via the AMC, AMT Publishing,
Canberra, 1992.
4. Atkins WJ, Edwards JD, King DJ, O’Halloran PJ and Taylor PJ,
Australian Mathematics Competition, Book 1, 1978–1984, AMT
Publishing, Canberra, 1986 and 2000.
5. Atkins WJ, Munro JEM and Taylor PJ, Australian Mathematics
Competition, Book 3, 1992–1998, AMT Publishing, Canberra,
1998.
6. Atkins WJ and Taylor PJ, Australian Mathematics Competition,
Book 4, 1999–2005, AMT Publishing, Canberra, 2006.
7. Barbeau EJ, Polynomials, Springer-Verlag, New York, 1989.
8. Beckenbach EF and Bellman R, An Introduction to Inequalities,
Mathematical Association of America, 1961.
9. Berzsenyi G and Maurer SB, The Contest Problem Book V,
Mathematical Association of America, 1996.
10. Burns JC, Seeking Solutions, AMT Publishing, Canberra, 2000.
11. Chinn WG and Steenrod NE, First Concepts of Topology,
Mathematical Association of America, 1966.
12. Coxeter HSM, Introduction to Geometry, John Wiley & Sons, New
York, 1961 and 1969.
13. Coxeter HSM and Greitzer SL, Geometry Revisited, Mathematical
Association of America, 1967.
General References 257

14. Fomin D and Kirichenko A, Leningrad Mathematical Olympiads,


Mathpro Press, Westford, Massachusetts, 1994.
15. Greitzer SL, International Mathematical Olympiads 1959–1977,
Mathematical Association of America, 1978.
16. Grossman I and Magnus W, Groups and Their Graphs,
Mathematical Association of America, 1964.
17. Gueron S, Hungary-Israeli Mathematics Competition: The First
Twelve Years, AMT Publishing, Canberra, 2004.
18. Henry JB, Dowsey J, Edwards AR, Mottershead LJ, Nakos A,
Vardaro G, Taylor PJ, Challenge! Book 1, 1991–1998, AMT
Publishing, Canberra, 2008.
19. Henry JB and Taylor PJ, Challenge! Book 2, 1999–2006, AMT
Publishing, Canberra, 2009.
20. Holton D, Let’s Solve Some Math Problems, Canadian
Mathematics Competition, Waterloo, Ontario, 1993.
21. Honsberger R, From Erdös to Kiev, Mathematical Association of
America, 1996.
22. Honsberger R, Ingenuity in Mathematics, Mathematical
Association of America, 1970.
23. Honsberger R, Mathematical Gems I, Mathematical Association of
America, 1973.
24. Honsberger R, Mathematical Gems II, Mathematical Association
of America, 1976.
25. Honsberger R, Mathematical Gems III, Mathematical Association
of America, 1985.
26. Honsberger R, Mathematical Morsels, Mathematical Association of
America, 1978.
27. Honsberger R, Mathematical Plums, Mathematical Association of
America, 1989.
28. Honsberger R, Mathematical Chestnuts from Around the World,
Mathematical Association of America, 2001.
258 General References

29. Kazarinoff ND, Geometric Inequalities, New Mathematical


Association of America, 1961.
30. Klamkin MS, International Mathematical Olympiads 1978–1985,
Mathematical Association of America, 1986.

31. Klamkin MS, USA Mathematical Olympiads, 1972–1986,


Mathematical Association of America, 1988.

32. Kuczma ME, 144 Problems of the Austrian-Polish Mathematics


Competition 1978–1993, The Academic Distribution Center,
Freeland, Maryland, 1994.

33. Kuczma ME and Windisbacher E, Polish and Austrian


Mathematical Olympiads 1981–1995: Selected Problems with
Multiple Solutions, AMT Publishing, Canberra, 1998.
34. Larson LC, Problem Solving Through Problems, Springer-Verlag,
New York, 1983.

35. Lausch H and Bosch Giral C, Asian Pacific Mathematics


Olympiad, 1989–2000, AMT Publishing, Canberra, 2000.

36. Lausch H and Taylor PJ, Australian Mathematical Olympiads


1979-1995, AMT Publishing, Canberra, 1997.

37. Lazarov B, Tabov JB, Taylor PJ and Storozhev AM, Bulgarian


Mathematics Competition, 1992–2001, AMT Publishing,
Canberra, 2004.
38. Liu A, Chinese Mathematics Competitions and Olympiads Book 1,
1981–1993, AMT Publishing, Canberra, 1997.

39. Liu A, Chinese Mathematics Competitions and Olympiads, Book 2,


1993–2001, AMT Publishing, Canberra, 2005.

40. Liu A, Hungarian Problem Book III: Based on the Eötvos


Competitions, 1929–1943, Mathematical Association of America,
2001.

41. Liu A and Taylor PJ, International Mathematics Tournament of


Towns, Problems and Solutions, Book 6, 2002–2007, AMT
Publishing, Canberra, 2009.
General References 259

42. Niven Ivan, Mathematics of Choice - How to count without


counting, Mathematical Association of America, 1965.
43. O’Halloran PJ, Pollard GH and Taylor PJ, Australian
Mathematics Competition, Book 2, 1985–1991, AMT Publishing,
Canberra, 1992 and 1998.
44. Ore O, Graphs and Their Uses, Mathematical Association of
America, 1963, revised and updated by R Wilson, 1990.
45. Ore O, Invitation to Number Theory, Mathematical Association of
America, 1967.
46. Plank AW and Williams NH, Mathematical Toolchest, AMT
Publishing, Canberra, 1992 and 1996.
47. Rabinowitz S, Index to Mathematical Problems 1980–1984,
Mathpro Press, Westford, Massachusetts, 1992.
48. Rapaport E, Hungarian Problem Book I, Mathematical
Association of America, 1963.
49. Rapaport E, Hungarian Problem Book II, Mathematical
Association of America, 1963.
50. Salkind CT, The Contest Problem Book, Random House, 1961.
51. Salkind CT, The Contest Problem Book II, Random House, 1966.
52. Salkind CT and Earl JM, The Contest Problem Book III, Random
House, 1973.
53. Schneider LJ, The Contest Problem Book VI, Mathematical
Association of America, 1998.
54. Sharygin IF, Problems in Plane Geometry, Mir, Moscow, 1988.
55. Sharygin IF, Problems in Solid Geometry, Mir, Moscow, 1986.
56. Slinko AM, USSR Mathematical Olympiads 1989–1992, AMT
Publishing, Canberra, 1997.
57. Storozhev A, International Mathematics Tournament of Towns,
Problems and Solutions, Book 5, 1997–2002, AMT Publishing,
Canberra, 2005.
260 General References

58. Tabov JB and Taylor PJ, Methods of Problem Solving, Book 1,


AMT Publishing, Canberra, 1996.
59. Tabov JB and Taylor PJ, Methods of Problem Solving, Book 2,
AMT Publishing, Canberra, 2002.

60. Taylor PJ, International Mathematics Tournament of Towns,


Problems and Solutions, Book 1, 1980–1984, AMT Publishing,
Canberra, 1993 and 2006.

61. Taylor PJ, International Mathematics Tournament of Towns,


Problems and Solutions, Book 2, 1984–1989, AMT Publishing,
Canberra, 1992 and 2003.

62. Taylor PJ, International Mathematics Tournament of Towns,


Problems and Solutions, Book 3, 1989–1993, AMT Publishing,
Canberra, 1994.

63. Taylor PJ and Storozhev A, International Mathematics


Tournament of Towns, Problems and Solutions, Book 4,
1993–1997, AMT Publishing, Canberra, 1998.

64. Yaglom IM, Geometric Transformations, Mathematical


Association of America, 1962.

65. Yaglom IM, Geometric Transformations II, Mathematical


Association of America, 1968.

66. Yaglom IM, Geometric Transformations III, Mathematical


Association of America, 1973.
Australian Intermediate Mathematics Olympiad
1999-2013
The Australian Intermediate in years 7 and 8. It also provides
Mathematics Olympiad (AIMO) is a challenge for students who have
the third stage and culmination performed well in the AMC.
of the Mathematics Challenge for
This book is a compilation of
Young Australians (MCYA).
past AIMO papers, presented in
It is a four-hour exam based on their original form, together with
Gauss and Noether components of full solutions and success rates.
the MCYA Enrichment Stage. The This book now brings the AIMO
problems are designed for students to a wider audience and provides
in year 10, but are valuable students and teachers with a useful
practice for very good students in and stimulating resource.
year 9 and for excellent students

Visit the Australian Mathematics Trust website at www.amt.edu.au


to purchase this book and other publications.
The Australian Mathematics Competition
is one of the greatest mathematics
competitions of the world. This book
brings together hundreds of motivating
and exciting questions set in the years 1978
to 1984. The questions and their solutions
have been grouped within topics. The book
is an extremely useful aid for mathematics
teachers, a learning vehicle for students and
a source of pleasure for all with an interest
in mathematics and problem solving.
WARREN ATKINS was a Senior Lecturer in the
Faculty of Education at the University of
Canberra and an original member of the
Australian Mathematics Foundation. He has
been a member of the Problems Committee
since its inception and held the position of
Chairman from 1976 to 1979 and 1995
to 2012.
JO EDWARDS was a Senior Lecturer in
Mathematics at the Canberra College of
Advanced Education (later University of
Canberra). She was involved with the
Australian Mathematics Competition since
its inception in 1978.
DECLAN KING was a mathematics teacher
in Canberra. He was a member of the
Problems Committee and on the Editorial
Committee from 1977 to 1988.
PETER O’HALLORAN is acknowledged as
founder of the Australian Mathematics
Competition and was also the Foundation
Executive Director of the Trust.
PETER TAYLOR was the Executive Director
of the Australian Mathematics Trust from
1995 to 2012 and was also a founding
member of the Australian Mathematics
Foundation. He was Chairman of the
Problems Committee from 1989 to 1994.

A U S T R A L I A N M AT H E M AT I C S T R U S T M ISBN 9 7 81 8 7 6 4 2 0 0 6 2

A
E N R I C H M E N T S E R I E S
T
9 7 81876 4200 62

You might also like